You are on page 1of 111

12/25/2014

Dr Bhatia Medical Coaching Institute:Online Test Platform

Online Test
Dashboard

Take Test

Results

My Progress

Study Material

Welcome Chilaka
Gt 135 (AIPG Mock)

Welcome, Chilaka

Marking: +1 Marks for correct answer and no negative Marking for incorrect answer.
Test Duration (mins): 180
User ID : db1407034

Time Left (mins) : 180


Instructions

Roll No.: -Group :

PG ONLINE

Once the test has started, do not press the refresh button (or F5 on your keyboard)
It is advisable to save the test regularly to avoid losing your information, save test refers to storing the attempted part of the test.
In case of accidental failure of internet connectivity the system will save the attempted portion of the test automatically.
Once the test time is over, you will be awarded a grace duration to wrap up the test and "Submit" it.

Quick Link
Syllabus

Submit test refers to the final completion of test; once you submit the test you will not be able to edit / preview your answers.
Please ensure that you are connected to the internet, while submitting the test.

Section

Test Package
Grand Test Series
2014
Subject Test Series
2014
AIIMS Test Series
DNB Test Series
Grand Test Series
2013

Part
(Q.1) What is the mechanism of Empaglifozim?
(a)

Insulin secretogogue

(b)

Increase peripheral utilization of glucose

(c)

Decrease hepatic glucose outflow

(d)

Renal glucose transport inhibitor

Your Response :

Correct Answer :

Exp:

Renal glucose transport inhibitor


Sodium-glucose cotransporter types 1 (SGLT1) and 2 (SGLT2) receptors and epithelial transport of sodium
and glucose. The SGLT receptors couple the transport of sodium and glucose across epithelial cells found on
the luminal membrane of the renal proximal tubule, allowing for reabsorption of glucose. Inhibiting SGLT2
receptors results in an increase in glucosuria and subsequent reductions in plasma glucose concentrations.

Subject Test Series


2013
Special Visual Based
Test

(Q.2) The defense mechanism characteristic of obsessive compulsive disorder is :


(a)

Denial

(b)

Projection

(c)

Undoing
Conversion

(d)
Your Response :

Correct Answer :

Exp:

Undoing.
This is an ego defense mechanism used by OCD patients. By this mechanism anxiety arising out of distressful
obsession is removed by undoing the feared consequents of obsession ego if regarding dirt or contamination then
pt tries to undo this obsession by washing compulsion.

(Q.3) Which of the following is the mechanism of abortion sticks?


(a)

Stimulation of uterine contraction

(b)

Menstrual bleeding

(c)

Irritation of the GIT

(d)

Irritation of the genito-urinary tract

Your Response :

Correct Answer :

Exp:

Stimulation of uterine contraction


The main mechanism of the use of abortion sticks is to produce an irritant action on the uterus so as to induce
uterine contractions and expulsion of the fetus.
Abortion stick is an object used to procure illegal abortion by unskilled interference by either self or by someone
else (a dai/midwife).
It usually consists of a specially made wooden or bamboo stick about 1520 cm long or a twig of similar length

http://dbmci.examonair.com/Result/ShowAllQuestionInHtml.aspx?testid=5615

1/111

12/25/2014

Dr Bhatia Medical Coaching Institute:Online Test Platform


with some irritant plant such as Madar (Calotropis), Chitra (Plumbago zeylandica) or Kanner (Nerium odorum)
applied at one of its ends.
The irritant substance is applied to some cotton wool or a piece of rag at one end of the stick.
The abortion sick is introduced into the os of the uterus thus causing abortion with or without rupture of the
membranes.
Excoriation, bruising or perforation of the vagina or uterus may occur as a result of the irritant action of the
substance.
(Q.4) Most common cause of Vesicovaginal fistula in India is?
(a)

Obstructed labour

(b)

Iatrogenic

(c)

LSCS

(d)

Gestational diabetes

Your Response : a
Correct Answer : A

Obstructed labour

Exp:

MC Urinary fistula = Vesicovaginal fistula


MC cause of VVF in India = Obstructed labour
MC cause of uretero vaginal fistula = Ureteral injury post Wertheims
hysterectomy
MC cause of Recto-vaginal fistula = Complete perineal tear
MC cause of vesico-uterine fistula = LSCS
(Q.5) Which of the following methods should not be used for delivery in a case of shoulder dystocia?
(a)

McRoberts maneuver

(b)

Zavanelli maneuver

(c)

Iatrogenic clavicular fracture...

(d)

Simpson's maneuver

Your Response :

Correct Answer :

Exp:

Simpson's maneuver.
A number of labor positions and/or obstetrical maneuvers are sequentially performed in attempt to facilitate
delivery at this point, including :
McRoberts maneuver; The McRoberts maneuver is employed in case of shoulder dystocia during childbirth and
involves hyperflexing the mother's legs tightly to her abdomen. This widens the pelvis, and flattens the spine in the
lower back (lumbar spine). If this maneuver does not succeed, an assistant applies pressure on the lower abdomen
(suprapubic pressure), and the delivered head is also gently pulled. The technique is effective in about 42% of
cases
suprapubic pressure (or Rubin I)
Rubin II or posterior pressure on theanterior shoulder, which would bring the fetus in an oblique position with
head somewhat towards the vagina
Woods' screw maneuverwhich leads to turning the anterior shoulder to the posterior and vice versa (somewhat
the opposite of Rubin II maneuver)
Jacquemier's maneuver(also called Barnum's maneuver), or delivery of theposterior shoulderfirst, in which the
forearm and hand are identified in the birth canal, and gently pulled.
Gaskin maneuver, named after Certified ProfessionalMidwife,Ina May Gaskin, involves moving the mother to an all
fours position with the back arched, widening the pelvic outlet.[8][9]
More drastic maneuvers include
Zavanelli's maneuver, which involves pushing the fetal head back in with performing acesarean section.[10]or
internal cephalic replacement followed byCesarean section
intentional fetalclavicular fracture, which reduces the diameter of the shoulder girdle that requires to pass through
the birth canal.
maternalsymphysiotomy, which makes the opening of the birth canal laxer by breaking the connective tissue
between the twopubesbones facilitating the passage of the shoulders.
abdominal rescue, described by O'Shaughnessy, where ahysterotomyfacilitates vaginal delivery of the impacted
shoulder
Management
"ALARMER"
Management of shoulder dystocia has become a focus point for many obstetrical nursing units in North America.
Courses such as the CanadianMore-OB programencourage nursing units to do routine drills to prevent delays in
delivery which adversely affect both mother and fetus. A common treatment mnemonic is ALARMER
Ask for help. This involves preparing for the help of an obstetrician, for anesthesia, and for pediatrics for
subsequent resuscitation of the infant that may be needed if the methods below fail.
Leg hyperflexion (McRoberts' maneuver)
Anterior shoulder disimpaction (pressure)
Rubin maneuver
Manual delivery of posterior arm
Episiotomy
Roll over on all fours
The advantage of proceeding in the order of ALARMER is that it goes from least to most invasive, thereby reducing
harm to the mother in the event that the infant delivers with one of the earlier maneuvers. In the event that these
maneuvers are unsuccessful, a skilled obstetrician may attempt some of the additional procedures listed above.
Intentional clavicular fracture is a final attempt at nonoperative vaginal delivery prior toZavanelli's
maneuverorsymphysiotomy, both of which are considered extraordinary treatment measures.

http://dbmci.examonair.com/Result/ShowAllQuestionInHtml.aspx?testid=5615

2/111

12/25/2014

Dr Bhatia Medical Coaching Institute:Online Test Platform


(Q.6) Endosalpingitis is best diagnosed by
(a)

Hystero laparoscopy

(b)

X-Ray abdomen

(c)

Hysterosalpingography

(d)

Sono salpingography

Your Response :

Correct Answer :

Hystero laparoscopy

Exp:

Hystero-Laparoscopy also called laparo-hysteroscopy is a procedure in which the tubes can be examined under
direct vision with an endoscope and if the pelvis looks inflamed then the best place to take a biopsy for culture is
the tubal end. Same holds true for the diagnosis of endo-salpingitis.
X-Ray abdomen is too gross a procedure and informs mostly anatomical problems of the pelvis.
HSG is a procedure which will inform regarding the tubal patency and outlines the uterine anatomy along with
uterine polyps and fibroids.
There may be some filling defects in the tube recognized with a good HSG if there is Endosalpingitis but it does not
prove the diagnosis as is done by a direct tubal culture.
Sono salpingography is an assessment of the tubal patency by perfusing the uterus and therefore the tubes while
observing the uterus and the pouch of Douglas by an ultrasound. This procedure is good for the diagnosis of tubal
patency but not for infections.
(Q.7) Which of the following is not included in Active management of III Stage of Labor?
(a)

Uterotonic within 1 minute of delivery

(b)

Immediate clamping, cutting and ligation of cord

(c)

General Massage of the uterus

(d)

Controlled cord traction

Your Response :

Correct Answer :

Exp:

Immediate clamping, cutting and ligation of cord


Clamping and ligature of cord is done in the second stage of labor and not the third.
Active management III stage of labor involves active assistance in delivery of placenta, so as to reduce the chances
of post partum hemorrhage.
It includes:
Administration of uterotonic like Oxytocin
Controlled cord traction
General massage of the uterus
During active management III stage we have to rule out any incidence of multiple pregnancies and then administer
intramuscular oxytocin within one minute of the delivery.
To deliver the placenta, gentle traction on the umbilical cord is put, while the contracted uterus is being held back.
Uterine massage is done to make the uterus go into contraction and facilitate expulsion of any retained clots.

(Q.8) Which of the following is the most common predisposing factor for placenta accreta?
(a)

Myomectomy

(b)

Recent curettage

(c)

Previous cesarean section

(d)

Placenta previa

Your Response :

Correct Answer :

Exp:

Placenta previa
Placenta accreta is an extremely rare condition in which the placenta is directly anchored to the myometrium
partially or completely without any intervening deciduas. The probable cause is defective decidual formation.
This condition is usually seen when the placenta is implanted in the lower segment (placenta praevia) or over the
previously injured sites such as on caesarean section scar, dilatation and curettage operation, manual removal,
synaecolysis or myomectomy.
The diagnosis is made only during attempted manual removal when the plane of cleavage between the placenta
and the uterine wall cannot be made out. Ultrasound imaging, color Doppler and MRI have all been valuable in the
diagnosis of placenta accreta during pregnancy.
Pathological confirmation includes:
Absence of decidua basalis
Absence of Nitabuchs fibrinoid layer, and
Varying degree of penetration of the villi into the
muscle bundles (increta) or up to the serosal layer (percreta).
The risks include hemorrhage, shock, infection and rarely inversion of the uterus.
10% of cases of placenta previa are associated with placenta accreta, an abnormally firm attachment of the
placenta to the uterine wall.
Placenta accreta prevents the placenta from separating from the wall of the uterus at the time of delivery and can
cause severe bleeding that often necessitates a hysterectomy.
Placenta accreta is particularly common in women with placenta previa and one or more previous caesarean
sections and may complicate 1/3 to of all such cases.
More than 50% of the patients with Placenta Accreta require a blood transfusionWilliams obstetrics.

http://dbmci.examonair.com/Result/ShowAllQuestionInHtml.aspx?testid=5615

3/111

12/25/2014

Dr Bhatia Medical Coaching Institute:Online Test Platform


(Q.9) Intrapartum automated fetal electrocardiography has the advantage over conventional cardiotocography of:
(a)

. Improving neonatal outcome

(b)

Decreasing caesarean section rate

(c)

Decreasing the need for fetal blood sampling during labour.

(d)

All of the above

Your Response :

Correct Answer :

Exp:

Decreasing the need for fetal blood sampling during labour.


Intrapartum automated fetal electrocardiography decreases the need for fetal blood sampling during labour in
comparison with conventional cardiotocography, however it has no impact on neonatal out come and it does not
decreases caesarean section rate.

(Q.10) Bambuterolis a pro drug of which of the following?


(a)

Salbutamol

(b)

Aminophyllin

(c)

Terbutaline

(d)

None of the above

Your Response :

Correct Answer :

Exp:

Terbutaline
Bambuterolis along acting beta-adrenoceptor agonist(LABA) used in the treatmentofasthma; it also is
aprodrugofterbutaline.

(Q.11) A Hystero-salpingogram was performed for infertility evaluation and revealed the appearance shown here. The diagnosis is?

(a)

Septate uterus

(b)

Bicornuate uterus

(c)

Unicornuate uterus
Arcuate uterus

(d)
Your Response :

Correct Answer :

Exp:

Bicornuate uterus
Mllerian duct anomalies are an often treatable cause of infertility. Mllerian duct anomalies are estimated to
occur in 01-0.5% of women. However, the true prevalence is unknown because the anomalies usually are
discovered in patients presenting with infertility. Full-term pregnancies have occurred in patients with forms of
bicornuate, septate, or didelphys uteri; therefore, true prevalence may be slightly higher than currently
estimated. An association exists between mllerian duct anomalies and renal anomalies such as unilateral
agenesis.
The bicornuate uterus results from incomplete fusion of the uterovaginal horns at the level of the fundus and
accounts for approximately 10% ofmllerian duct anomalies. Patients with a bicornuate uterus and no
extrauterine infertility issues usually have little difficulty conceiving. A bicornuate uterus consists of two
symmetric cornua that are fused caudally, with communication of the endometrial cavitiesmost often at the
level of the uterine isthmus. The intervening cleft of the complete bicornuate uterus extends to the internal
cervical os (bicornuate unicollis), while the cleft of a partial bicornuate configuration is of variable length. A
bicornuate bicollis uterus is associated with a duplicatedcervix, although a degree of communication is
maintained between the two horns. The bicornuate uterus has been reported to have the highest associated
prevalence38%of cervical incompetence among mllerian duct anomalies.

(Q.12) Rohr's stria found in the placenta are due to:


(a)

Deposition of fibrin

(b)

Langhans cells accumulation

(c)

Hofbauer cells accumulation

(d)

Blood vessels

Your Response :

Correct Answer :

Exp:

Deposition of fibrin
There may be inconsistent deposition of fibrin called Rohr's stria at the bottom of the intervillous space and
surrounding the fastening villi.

http://dbmci.examonair.com/Result/ShowAllQuestionInHtml.aspx?testid=5615

4/111

12/25/2014

Dr Bhatia Medical Coaching Institute:Online Test Platform


(Q.13) Which of the following image represents a Dichorionic - Diamniotic gestation?

(a)
(b)
(c)
(d)
Your Response :

Correct Answer :

Exp:

Types of placenta in a twin pregnancy : 1.single (monochorionic) placenta. 2. two separate (dichorionic) placenta
.
In monochorionic twins there is one chorion around both babies which means they share one placenta and the
term monochorionic is used. Triplets and Quads can also be monochorionic. The inner membrane is known as the
amnion. If the babies each have their own amnion the pregnancy will be referred to as monochorionic diamniotic
(MCDA). These babies are always identical. In about 1% of monochoronic twin pregnancies, the babies will also
share one inner membrane (amnion). These babies will be referred to as monchorionic monamniotic (MCMA),
and are always identical. In dichorionic twins each baby will have its own placenta, with its own amnion and
chorion. For these twins there will be two chorions and two amnions the term dichorionic diamniotic (DCDA) is
used. If triplets each have their own placenta there will 4 be three chorions and three chorions they will be
referred to as trichorionic triamniotic.

(Q.14) Which of the following hormones does not cross the placenta?
(a)

Chorionic gonadotrophin

(b)

Growth hormone

(c)

Thyroxine

(d)

Calcitonin

Your
b
Response :
Correct
D
Answer :
Exp:
(Q.15) Amniotic membrane contains:
(a)

Blood vessels

(b)

Nerves

(c)

Glycerophospholipids

(d)

All of the above

Your Response :
Correct Answer :
Exp:

C
Glycerophospholipids
The amnion has got neither blood nor nerve supply nor any lymphatic system. It is a rich source of
glycerophospholipids' containing arachidonic acid precursor of prostaglandin E2 and F2 .

(Q.16) CHARGE Syndrome includes ?


(a)

Choanal atresia

(b)

Coloboma of eye

(c)

Ear anomalies

(d)

All of above

Your Response :

Correct Answer :
Exp:

D
All of above
Congenital anomalies are a major cause of stillbirths and neonatal deaths, but they are perhaps even more
important as causes of acute illness and long-term morbidity. Anomalies are discussed in general in Chapters 81
and 108 and specifically in the chapters on the various systems of the body. Early recognition of anomalies is
important for planning care; with some, such as tracheoesophageal fistula, diaphragmatic hernia, choanal atresia,
and intestinal obstruction, immediate medical and surgical therapy is essential for survival. Parents are likely to
feel anxious and guilty on learning of the existence of a congenital anomaly and require sensitive counseling.
Common Life-Threatening Congenital Anomalies
NAME

MANIFESTATIONS

Choanal atresia

Respiratory distress in delivery room, apnea, unable to pass nasogastric


tube through nares. Suspect CHARGE syndrome

Pierre Robin syndrome

Micrognathia, cleft palate, airway obstruction

http://dbmci.examonair.com/Result/ShowAllQuestionInHtml.aspx?testid=5615

5/111

12/25/2014

Dr Bhatia Medical Coaching Institute:Online Test Platform


Diaphragmatic hernia

Scaphoid abdomen, bowel sounds present in chest, respiratory distress

Tracheoesophageal fistula

Polyhydramnios, aspiration pneumonia, excessive salivation, unable to


place nasogastric tube in stomach. Suspect VATER syndrome

Intestinal obstruction: volvulus,


duodenal atresia, ileal atresia

Polyhydramnios, bile-stained emesis, abdominal distention. Suspect


trisomy 21, cystic fibrosis, cocaine

Gastroschisis, omphalocele

Polyhydramnios, intestinal obstruction

Renal agenesis, Potter syndrome

Oligohydramnios, anuria, pulmonary hypoplasia, pneumothorax

Neural tube defects: anencephalus,


meningomyelocele

Polyhydramnios, elevated -fetoprotein, decreased fetal activity

Ductal-dependent congenital heart


disease

Cyanosis, hypotension, murmur

CHARGE, coloboma of the eye, heart anomaly, choanal atresia, retardation, and genital and ear anomalies;
VATER, vertebral defects, imperforate anus, tracheoesophageal fistula, and radial and renal dysplasia.
(Q.17) Wrong match regarding Maternal disorder and its fetal effect is ?
(a)

Autoantibody against folate receptors : NTDs

(b)

Cyanotic heart disease : IUGR

(c)

Graves disease : Fetal hypothyroidism

(d)

Sickle cell anemia : Preterm birth

Your Response :

Correct Answer :

Exp:

Graves disease : Fetal hypothyroidism


Ref : Kliegman: Nelson Textbook of Pediatrics, 18th ed.Chap 95
Maternal Conditions Affecting the Fetus or Neonate
EFFECTS

MECHANISM

Autoantibody against
folate receptors

Neural tube defects

Block cellular uptake of folate

Cervical neoplasia

Preterm premature rupture of


membranes

Associated with loop electrosurgical


excision procedure or laser cone
therapy

Cholestasis

Preterm delivery

Unknown, possibly hepatitis E

Cyanotic heart disease

Intrauterine growth restriction

Low fetal oxygen delivery

Mild

Large for gestational age,


hypoglycemia

Fetal hyperglycemiaproduces
hyperinsulinemia; insulin promotes
growth

Severe

Growth restriction

Vascular disease, placental


insufficiency

Drug addiction

Intrauterine growth restriction,


neonatal withdrawal

Direct drug effect plus poor diet

Endemic goiter

Hypothyroidism

Iodine deficiency

Graves disease

Transient neonatal thyrotoxicosis Placental immunoglobin passage of


thyroid-stimulating antibody

Herpes gestationalis
(noninfectious)

Bullous rash

Unknown

Hyperparathyroidism

Neonatal hypocalcemia

Maternal calcium crosses to fetus


and suppresses fetal parathyroid
gland

Hypertension

Intrauterine growth restriction,


intrauterine fetal demise

Placental insufficiency, fetal hypoxia

DISORDER

Diabetes mellitus

Idiopathic
Thrombocytopenia
thrombocytopenic purpura

Nonspecific maternal platelet


antibodies cross placenta

Isoimmune neutropenia or Neutropenia or


thrombocytopenia
thrombocytopenia

Specific antifetal neutrophil or


platelet antibody crosses placenta
after sensitization of mother

Malignant melanoma

Placental or fetal tumor

Metastasis

Myasthenia gravis

Transient neonatal myasthenia

Immunoglobin to acetylcholine
receptor crosses placenta

Myotonic dystrophy

Neonatal myotonic dystrophy,


congenital contractures,
respiratory insufficiency

Genetic anticipation

Obesity

Macrosomia, hypoglycemia

Unknown

Phenylketonuria

Microcephaly, retardation

Elevated fetal phenylalanine levels

Poor nutrition

Intrauterine growth restriction,


adult insulin resistance,
schizophrenia (?)

Reduced fetal nutrients, nutritional


programming

Preeclampsia, eclampsia

Intrauterine growth restriction, Uteroplacental insufficiency, fetal


thrombocytopenia, neutropenia, hypoxia, vasoconstriction
fetal demise

Renal transplant

Intrauterine growth restriction

Rhesus or other blood


group sensitization

Fetal anemia, hypoalbuminemia, Antibody crosses placenta directed


hydrops, neonatal jaundice
to fetal cells with antigen

Sickle cell anemia

Preterm birth, intrauterine


growth restriction

http://dbmci.examonair.com/Result/ShowAllQuestionInHtml.aspx?testid=5615

Uteroplacental insufficiency

Maternal sickling producing fetal


hypoxia

6/111

12/25/2014

Dr Bhatia Medical Coaching Institute:Online Test Platform


Systemic lupus
erythematosus

Congenital heart block, rash,


anemia, thrombocytopenia,
neutropenia

Antibody directed to fetal heart, red


and white blood cells, and platelets

(Q.18) A 22 years old female Kamala, who had body mass index 18 before pregnancy, was pregnant for first time. She may be allowed to

weight gain upto :


(a)

7kg

(b)

13kg

(c)

18 kg
21 kg

(d)
Your Response :

Correct Answer :

Exp:

18 kg
Ideally weight gain should depend or pre pregnancy body mass index (BMI) level weight gain for a woman with
normal BMI (20-26) is 11 to 16 kg. An obese woman (BMI>29) should not gain more than 7 kg, whereas an under
weight woman (BMI<19) may be allowed to gain upto 18 kg.

(Q.19) Drug of choice for Infantile spasm is :


(a)

ACTH

(b)

Valproic Acid

(c)

Pyridoxine

(d)

Clonazepam

Your Response :

Correct Answer :

Exp:

ACTH
Though there are number of drugs which can be used in Infantile spasms,
ACTH is preferred.
Oral corticosteroids can be used; vigabatrin is emerging as a first choice.

(Q.20) The main source of production of relaxin is:


(a)

Ovary

(b)

Placenta

(c)

Decidua
Adrenals

(d)
Your Response :

Correct Answer :

Exp:

Ovary
The main source of production of relaxin is the corpus luteum of the ovary but part of it may also be produced by
the placenta and decidua.
Relaxin production refers to production and secretion of one of the well known peptide hormones, relaxin. Dustin
Ensign was the first scientist who described this hormone in the year 1926. The hormone is found both in human
as well as in animal bodies. The hormone is also present in both male and female. Relaxin production
considerably increases during pregnancy.
Relaxin is well known for having more than one function too. During pregnancy relaxin production gradually
increases to make delivery easier. At that time the hormone is produced from different sources of relaxin
production.
Sources of Relaxin production
The sources of relaxin production is different for both men and women. In each case the production takes
place in different organs of the body. The presence of the hormone is relatively higher in pregnant women
than in men.In female bodies relaxin production takes place in the corpus luteum in the ovary. The breasts too
are responsible for production of some amount of the hormone. At the time of pregnancy, adequate amount
of relaxin is also produced by placenta, chorion as well as deciduas. This adds to the increased level of relaxin
production. This additional production of the hormone during pregnancy also results to the increased level of
hormone present in the body of pregnant women.In the bodies of men testes is regarded as the primary
source of relaxin production. In other words, the production of the hormone named relaxin takes place in that
part of a male body which is known as testes. The hormone is generated by breaking an additional chain of
peptide. However, the amount of relaxin present in male body is significantly lesser than women, especially
those who are pregnant.
The level of relaxin production is different at different stages of pregnancy. At the time of ovulation the level of
relaxin production rises in female bodies.
At that time corpus luteum starts producing an increased amount of relaxin. If during that time pregnancy does
not occur, the level of relaxin production again drops to normal. A significant drop in the level of the hormone
takes place during the time of menstruation.
During the first trimester of pregnancy the level of production of relaxin also goes up. At that time an additional
amount of relaxin is also produced by deciduas. This results to an increased amount of relaxin production.
The increased amount of relaxin production facilitates the process of pregnancy in many ways. It softens cervix
and pubic symphysis, prevents contraction of uterine muscle by reducing collagen production. Thus the process of

http://dbmci.examonair.com/Result/ShowAllQuestionInHtml.aspx?testid=5615

7/111

12/25/2014

Dr Bhatia Medical Coaching Institute:Online Test Platform


delivery becomes less painful.
(Q.21) Most precise way of determining gestational age IS:
(a)

Crown rump length

(b)

Biparietal diameter

(c)

Femur Length

(d)

Head Circumference

Your Response :

Correct Answer :

Exp:

Crown rump length


Estimation of gestational age by Crown rump length is most precise (variation ::1:5 days).
Ultrasoundcan be used to determine gestational age. Measurement of a crown-rump length during the first
trimester (1-13 weeks) will give a gestational age that is usually accurate to within 3 days of the actual due date.
During the second trimester (14-28 weeks), measurement of the biparietal diameter will accurately predict the
due date within 10-14 days in most cases. In the third trimester, the accuracy of ultrasound in predicting the due
date is less, with a plus or minus confidence range of as much as 3 weeks

(Q.22) Fetal biophysical profile used for the assessment of fetal well being does not includes:
(a)

Fetal breathing movements

(b)

Fetal tone

(c)

Amniotic fluid volume

(d)

Fetal blood PH

Your Response : d
Correct Answer : D

Fetal blood PH

Exp:

Fetal biophysical profile considers several parameters, which includes:


Non Stress test
Fetal breathing movements
Gross body movements
Fetal muscle tone
Amniotic fluid volume
(Q.23) As per WHO recommendations, the number of antenatal visits by a pregnant women to the hospital for check up should be at least:
(a)

(b)

(c)

4
5

(d)
Your Response :

Correct Answer :

Exp:

4
How many visits? A recent multi-country randomized control trial led by the WHO and a systematic review
showed that essential interventions can be provided over four visits at specified intervals, at least for healthy
women with no underlying medical problems. The result of this review has prompted WHO to define a new
model of ANC based on four goal-oriented visits. This model has been further defined by what is done in each
visit, and is often called focused antenatal care. The optimum number of ANC visits for limited resource settings
depends not only on effectiveness, but also on costs and other barriers to ANC access and supply.
When? For many of the essential interventions in ANC, it is crucial to have early identification of underlying
conditions for example, prevention of congenital syphilis, control of anaemia, and prevention of malaria
complications. Hence the first ANC visit should be as early as possible in pregnancy, preferably in the first
trimester. The last visit should be at around 37 weeks or
near the expected date of birth to ensure that appropriate advice and care have been provided to prevent and
manage problems such as multiple births (e.g. twins), postmaturity (e.g. birth after 42 weeks of pregnancy, which
carries an increased risk of fetal death), and abnormal positions of the baby (e.g. breech, where the babys head is
not the presenting part at birth).

(Q.24) Fundal pressure method for delivery of placenta is used when baby is:
(a)

Premature

(b)

Large

(c)

Asphyxiated

(d)

Rh- Incompatible with mother

Your Response :

Correct Answer :

Exp:

Premature
If the baby is macerated or premature, fundal pressure method is preferable to cord traction as the tensile
strength of the cord is much reduced in both the instances.

(Q.25) A 30 year female presented fever pain abdomen & purulent vaginal discharge after 2 days of abortion. The doctor diagnosed her as a

case of septic abortion. On examination & investigations it was found that infection was localized to the uterus & parametrium.
From which grade of septic abortion she was suffering from?

http://dbmci.examonair.com/Result/ShowAllQuestionInHtml.aspx?testid=5615

8/111

12/25/2014

Dr Bhatia Medical Coaching Institute:Online Test Platform


(a)

Grade I

(b)

Grade II

(c)

Grade III

(d)

Grade IV

Your Response :

Correct Answer :

Exp:

Grade II
Clinically, the cases of septic' abortion are graded as:
(1) Grade I: The infection is localized to the uterus.
(2) Grade II: The infection spreads beyond the uterus to the parametrium, tubes and ovaries or pelvic
peritoneum.
(3) Grade III: Generalized peritonitis and/or endotoxic shock or jaundice or acute renal failure.

(Q.26) During resuscitation of newborn chest compressions are indicated when heart rate is below :
(a)

40bpm

(b)

60bpm

(c)

80 bpm
100bpm.

(d)
Your Response :

Correct Answer :

Exp:

60 bpm
External cardiac massage is indicated in babies in whom heart rate drops below 60/min despite effective
ventilation with 100% oxygen for 30 seconds. There should be 90 compressions and 30 ventilations (3: 1). Chest
compressions are discontinued when HR is above 60/min.
Chest Compressions
Chest compressions are indicated for a heart rate that is <60 per minute despite adequate ventilation with
supplementary oxygen for 30 seconds. Because ventilation is the most effective action in neonatal resuscitation
and because chest compressions are likely to compete with effective ventilation, rescuers should ensure that
assisted ventilation is being delivered optimally before starting chest compressions.
Compressions should be delivered on the lower third of the sternum to a depth of approximately one third of the
anterior-posterior diameter of the chest . Two techniques have been described: compression with 2 thumbs with
fingers encircling the chest and supporting the back (the 2 thumbencircling hands technique) or compression
with 2 fingers with a second hand supporting the back. Because the 2 thumbencircling hands technique may
generate higher peak systolic and coronary perfusion pressure than the 2-finger technique. the 2 thumb
encircling hands technique is recommended for performing chest compressions in newly born infants . The 2finger technique may be preferable when access to the umbilicus is required during insertion of an umbilical
catheter, although it is possible to administer the 2 thumbencircling hands technique in intubated infants with
the rescuer standing at the baby's head, thus permitting adequate access to the umbilicus.
Compressions and ventilations should be coordinated to avoid simultaneous deliveryThe chest should be
permitted to reexpand fully during relaxation, but the rescuer's thumbs should not leave the chest. There should
be a 3:1 ratio of compressions to ventilations with 90 compressions and 30 breaths to achieve approximately 120
events per minute to maximize ventilation at an achievable rate. Thus each event will be allotted approximately
1/2 second, with exhalation occurring during the first compression after each ventilation.
There is evidence from animals and non-neonatal studies that sustained compressions or a compression ratio of
15:2 or even 30:2 may be more effective when the arrest is of primary cardiac etiology. One study in children
suggests that CPR with rescue breathing is preferable to chest compressions alone when the arrest is of
noncardiac etiology.It is recommended that a 3:1 compression to ventilation ratio be used for neonatal
resuscitation where compromise of ventilation is nearly always the primary cause, but rescuers should consider
using higher ratios if the arrest is believed to be of cardiac origin.
Respirations, heart rate, and oxygenation should be reassessed periodically, and coordinated chest compressions
and ventilations should continue until the spontaneous heart rate is 60 per minute.However, frequent
interruptions of compressions should be avoided, as they will compromise artificial maintenance of systemic
perfusion and maintenance of coronary blood flow.

(Q.27) Which type of inheritance does the following pedigree chart denote?

(a)

Mitochondrial inheritance

(b)

Autosomal dominant

(c)

Autosomal recessive

(d)

X-linked dominant

Your Response :

Correct Answer :

Exp:

Mitochondrial inheritance
In contrast to the homologous pair recombination that takes place in the nucleus, mtDNA molecules do not
undergo recombination, and so mutational events represent the only source of mtDNA genetic diversification.
Moreover, with very rare exceptions, it is only the maternal DNA that is transmitted to the offspring. The fertilized
oocyte degrades mtDNA carried from the sperm in a complex process that involves the ubiquitin proteasome
system. Thus, whereas mothers transmit their mtDNA to both their sons and their daughters, only the daughters
are able to transmit the inherited mtDNA to future generations. Accordingly, mtDNA sequence variation and

http://dbmci.examonair.com/Result/ShowAllQuestionInHtml.aspx?testid=5615

9/111

12/25/2014

Dr Bhatia Medical Coaching Institute:Online Test Platform


associated phenotypic traits and diseases are inherited exclusively along maternal lines. As noted below, because
of the complex relationship between mtDNA mutations and disease expression, sometimes this maternal
inheritance is difficult to recognize at the clinical or pedigree level. However, evidence of paternal transmission
almost certainly rules out an mtDNA genetic origin of phenotypic variation or disease; conversely, a disease
affecting both sexes without evidence of paternal transmission strongly suggests a heritable mtDNA disorder
Some examples of mitochondrial myopathies are:
a. Lebers hereditary optic neuropathy
b. MELAS syndrome
c. Kearns-Sayre syndrome
d. Myoclonic epilepsy and ragged red fiber's syndrome
e. Maternally inherited myopathy and cardiomyopathy
f. Progressive external ophthalmoplegia
g. Pearson syndrome: bone marrow and pancreatic failure
h. Lactic acidosis
(Q.28) Marker for Paget's disease of breast is
(a)

CEA

(b)

HMB-45

(c)

S-100
Synaptophysin

(d)
Your Response :

Correct Answer :

Exp:

CEA
"CEA is elevated in adenocarcinoma of colon, pancreas, lung, BREAST and ovary" "S-100 protein is present in
virtually all primary and metastatic melanomas, including the amelanotic variety. However, S-100 positivity is also
found in other tumours of neuro endocrinal origin (e.g. small cell lung cancer, carcinoid, neuro-epithelioma); a
more specific marker for melanoma is the HMB45 (human melanoma black 45) antigen".
Synaptophysin is the marker for neuroendocrine tumours.

(Q.29) A patient has retinoblastoma. He is also likely to develop Mandibular osteosarcoma if the pathogenetic causes are the following

EXCEPT
(a)

P53 gene mutation

(b)

Radiation induced

(c)

Chemotherapy induced

(d)

Mutation of Rb gene

Your Response :

Correct Answer :

Exp:

Chemotherapy induced
Patients with hereditary retinoblastomas have up to 1,000 times greater risk of subsequently developing
osteosarcoma, attributed to germ-line mutations in the RB gene.
Loss of heterozygosity, structural rearrangements, or point mutations in the RB gene is also present in 60 70%of
sporadic tumors.
Abnormalities in genes that regulate cell cycling, such as p53 , CDK4,p16,INK4A,CYCLIN DI, and MDM2 have also
been implicated in the genesis of nonhereditary osteosarcomas
The risk of osteosarcoma developing in facial bones increases greatly if external beam irradiation has been used
to treat the retinoblastoma
Germ-line mutation of the Rb-1 locus (chromosome 13q14) in patients with inherited retinoblastoma is
associated with the development of osteosarcoma in those who survive the retinoblastoma and of soft tissue
sarcomas unrelated to radiation therapy.

(Q.30) Hemodialysis associated amyloidosis is due to the deposition of


(a)

2- Microglobulin

(b)

Transthyretin

(c)

SAA

(d)

APP

Your Response :

Correct Answer :

Exp:

2- Microglobulin
Hemodialysis-associated amyloidosis Patients on long-term hemodialysis for renal failure develop amyloidosis
owing to deposition of 2- Microglobulin. This protein is present in high concentrations in the serum of patients
with renal disease and is retained in circulation because it cannot be filtered through the cuprophane dialysis
membranes. In some series, as many as 60 80%of the patients on long-term dialysis developed amyloid deposits
in the synovium, joints and tendon sheaths.
Amyloid Fibril Proteins and Their Clinical Syndromes

Term

Precursor

Clinical Syndrome

Clinical Involvement

Primary or myeloma
associateda

Any

Systemic Amyloidoses
AL

Immunoglobulin light chain

http://dbmci.examonair.com/Result/ShowAllQuestionInHtml.aspx?testid=5615

10/111

12/25/2014

Dr Bhatia Medical Coaching Institute:Online Test Platform


AH

Immunoglobulin heavy chain Primary or myeloma associated Any


(rare)

AA

Serum amyloid A protein

2-Microglobulin

Secondary; reactiveb

Renal, any

Hemodialysis-associated

Synovial membrane, bone

Familial (mutant)
Senile systemic (wild type)

Cardiac, peripheral and


autonomic nerves

2M
ATTR

Transthyretin

AApoAI

Apolipoprotein AI

Familial

Hepatic, renal

AApoAII

Apolipoprotein AII

Familial

Renal

AGel

Gelsolin

Familial

Corneas, cranial nerves, renal

AFib

Fibrinogen A

Familial

Renal

ALys

Lysozyme

Familial

Renal

ALECT2

Leukocyte chemotactic factor ?


2

Renal

Localized Amyloidoses
A

Alzheimers disease;

Down syndrome

CNS

ACys
APrP

Cystatin C

Cerebral amyloid angiopathy

CNS, vascular

Prion protein

Spongiform encephalopathies

CNS

AIAPP

Islet amyloid polypeptide


(amylin)

Diabetes-associated

Pancreas

ACal

Calcitonin

Medullary carcinoma of the


thyroid

Thyroid

AANF

Atrial natriuretic factor

Age-related

Cardiac atria

APro

Prolactin

Endocrinopathy

Pituitary

Amyloid protein

a. Localized deposits can occur in skin, conjunctiva, urinary bladder, and tracheobronchial tree.
b. Secondary to chronic inflammation or infection, or to a hereditary periodic fever syndrome, e.g., familial
Mediterranean fever.
(Q.31) Delayed continuous vascular permeability is due to
(a)

Endothelia contraction

(b)

Retraction by endothelial venules and capillaries

(c)

Direct endothelial injury

(d)

Angiogenesis

Your Response :

Correct Answer :

Exp:

Direct endothelial injury


Several mechanisms may contribute to increased vascular permeability in acute inflammatory reactions.
Endothelial cell contraction leading to intercellular gaps in postcapillary venules is the most common cause of
increased vascular permeability. It is a reversible process elicited by histamine, bradykinin, leukotrienes, and
many other chemical mediators. Endothelial cell contraction occurs rapidly after binding of mediators to specific
receptors, is usually short-lived (15-30 minutes), and is called the immediate transient response. A slower and
more prolonged retraction of endothelial cells, resulting from changes in the cytoskeleton, may be induced by
cytokines such as tumor necrosis factor (TNF) and interleukin-1 (IL-1). This reaction may take 4 to 6 hours to
develop after the initial trigger and persist for 24 hours or more.Endothelial injury results in vascular leakage by
causing endothelial cell necrosis and detachment. Direct injury to endothelial cells is usually seen after severe
injuries (e.g., burns and some infections). In most cases leakage begins immediately after the injury and persists
for several hours (or days) until the damaged vessels are thrombosed or repaired. Therefore, this reaction is
known as the immediate sustained response. Venules, capillaries, and arterioles can all be affected, depending on
the site of the injury. Direct injury to endothelial cells may also induce a delayed prolonged leakage that begins
after a delay of 2 to 12 hours, lasts for several hours or even days, and involves venules and capillaries.
Examples include mild to moderate thermal injury, certain bacterial toxins, and x- or ultraviolet irradiation (i.e.,
the sunburn that appears the evening after a day in the sun).Leukocyte-mediated endothelial injury may occur as
a consequence of leukocyte accumulation along the vessel wall. As discussed later, activated leukocytes release
many toxic mediators that may cause endothelial injury or detachment.Increased transcytosis of proteins via an
intracellular vesicular pathway augments venular permeability, especially after exposure to certain mediators
such as vascular endothelial growth factor (VEGF). Transcytosis occurs via channels formed by fusion of
intracellular vesicles.Leakage from new blood vessels. Tissue repair involves new blood vessel formation
(angiogenesis). These vessel sprouts remain leaky until proliferating endothelial cells mature sufficiently to form
intercellular junctions. New endothelial cells also have increased expression of receptors for vasoactive
mediators, and some of the factors that induce angiogenesis (e.g., VEGF) directly induce increased vascular
permeability via transcytosis.

(Q.32) HMB-45 is immunohistochemistry marker for


(a)

Melanoma

(b)

Schwannoma

(c)

Neurofibroma

(d)

Rhabdomyosarcoma

Your Response :

Correct Answer :

http://dbmci.examonair.com/Result/ShowAllQuestionInHtml.aspx?testid=5615

11/111

12/25/2014

Dr Bhatia Medical Coaching Institute:Online Test Platform


Exp:

Melanoma
The cytologic diagnosis of metastatic melanoma can be challenging. Melanoma often manifests with a diverse
cytologic appearance that may include a dys Hesive single cell pattern or a cohesive cellular arrangement.
The cell shape varies from epithelioid to spindled, or a mixture of epithelioid and spindle cell patterns might be
seen. Furthermore, the cytologic features of melanoma often are shared with other poorly differentiated
malignant neoplasms, including carcinomas, lymphomas, and sarcomas.
In addition, metastatic melanoma can be found anywhere in the body and may manifest with a myriad of clinical
signs and symptoms. Nevertheless, it is important to differentiate melanoma from non-melanocytic malignant
neoplasms because prognosis and therapy differ radically among these entities.
Immunocytochemical studies often are used as an aid in the diagnosis of melanoma. The most frequently used
melanocytic markers in clinical practice are S-100 protein and HMB-45
Monoclonal antibody to S-100 protein, a calcium binding F-hand protein originally isolated from the brain, is a
sensitive marker that reacts with more than 90%of melanomas.
However, this protein also is present in adipocytes, chondrocytes, Schwann cells, and myoepithelial cells.
Tumors derived from these tissues usually retain immunoreactivity to S-100 protein.Thus,S-100 protein reacts
with a broad range of benign and malignant neoplasms, therefore limiting its specificity as a melanocytic marker.
In addition, certain epithelial neoplasms such as mammary carcinoma may be positive for S-100 protein.
Monoclonal antibody against HMB-45 antigen recognizes melanosome-specific gp100.Although it is quite specific
for melanocytic neoplasms,HMB-45 is less sensitive than S-100 protein for identifying melanoma.
Immunohistochemical Stains Useful in detecting neoplasms
Tissue Marker

Diagnosis

Estrogen and progesterone receptors

Breast cancer

BRST-1

Breast cancer

Gross cystic disease fibrous protein-15

Breast cancer

Thyroid transcription factor 1

Lung and thyroid cancer

Thyroglobulin

Thyroid cancer

Chromogranin, synaptophysin, neuron specific enolase

Neuroendocrine cancer

CDX-2

Gastrointestinal cancer

Calretinin, mesothelin

Mesothelioma

Leukocyte common antigen

Lymphoma

10

S-100, HMB-45

Melanoma

11

URO-III, thrombomodulin

Bladder cancer

12

-Fetoprotein

Hepatocellular cancer,
Germ cell cancer,
Hepatoblastoma

13

-Human chronic gonadotropin

Germ cell cancer

14

Prostate specific antigen

Prostate cancer

15

Cytokeratin

Carcinomas

(Q.33) Diphtheria-like colitis with thickening of ascending and transverse colon are seen due to which poisoning
(a)

Phenol

(b)

Arsenic

(c)

Mercury
Lead

(d)
Your Response :

Correct Answer :

Exp:

Mercury poisoning
Mercury poisoning has 2 stages of signs and symptoms.
The first phase is acute and lasts upto about a day.
The second phase is more protracted and starts from one to three days after many days of a patient suffering
from Hg poisoning; membranous colitis develops and produces dysentery, ulceration of the colonic mucosa and
hemorrhage.
On post-mortem examination, a very characteristic appearance seen in the small intestine is the fissuring of the
swollen mucous membrane. These fissures run in all directions, and break up the continuity of the surface,
forming numerous small islands. Such features are characteristic of Hg poisoning and are not observed in Phenol,
Arsenic or Lead poisoning.

(Q.34) If a person under oath gives false evidence in a court of law, then he may be punished with imprisonment upto :
(a)

Seven years

(b)

Nine years

(c)

One year
Three years

(d)
Your Response :

Correct Answer :

Exp:

Seven years
A witness who willfully or with some motive tells lie, suppresses facts partly or fully in a court as a witness, may
be declared a hostile witness. If a person under oath gives false evidence in a court of law, then he may be
punished with imprisonment upto 7 years. In other case for giving false evidence (other than under oath inside a
court) a person may be punished with imprisonment which may extend upto 3 years (per jury)

http://dbmci.examonair.com/Result/ShowAllQuestionInHtml.aspx?testid=5615

12/111

12/25/2014

Dr Bhatia Medical Coaching Institute:Online Test Platform


(Q.35) The following are rules of Helsinki declaration except :
(a)

The drug used for the experiment must not be a known harmful agent

(b)

There must be scientific justification

(c)

The experiment must be conducted by medical man.

(d)

The experiment should not be stopped if any untowards symptoms starts.

Your Response :

Correct Answer :

Exp:

The experiment should not be stopped if any unto wards symptoms starts.
The World Medical Association (WMA) has developed the Declaration of Helsinki as a statement of ethical
principles for medical research involving human subjects, including research on identifiable human material and
data.
1. The Declaration is intended to be read as a whole and each of its constituent paragraphs should be applied
with consideration of all other relevant paragraphs.
2. Consistent with the mandate of the WMA, the Declaration is addressed primarily to physiciAns: The WMA
encourages others who are involved in medical research involving human subjects to adopt these principles.
General Principles
3. The Declaration of Geneva of the WMA binds the physician with the words, The health of my patient will be
my first consideration, and the International Code of Medical Ethics declares that, A physician shall act in the
patient's best interest when providing medical care.
4. It is the duty of the physician to promote and safeguard the health, well-being and rights of patients,
including those who are involved in medical research. The physician's knowledge and conscience are dedicated to
the fulfilment of this duty.
5.

Medical progress is based on research that ultimately must include studies involving human subjects.

6. The primary purpose of medical research involving human subjects is to understand the causes,
development and effects of diseases and improve preventive, diagnostic and therapeutic interventions (methods,
procedures and treatments). Even the best proven interventions must be evaluated continually through research
for their safety, effectiveness, efficiency, accessibility and quality.
7. Medical research is subject to ethical standards that promote and ensure respect for all human subjects and
protect their health and rights.
8. While the primary purpose of medical research is to generate new knowledge, this goal can never take
precedence over the rights and interests of individual research subjects.
9. It is the duty of physicians who are involved in medical research to protect the life, health, dignity, integrity,
right to self-determination, privacy, and confidentiality of personal information of research subjects. The
responsibility for the protection of research subjects must always rest with the physician or other health care
professionals and never with the research subjects, even though they have given consent.
10. Physicians must consider the ethical, legal and regulatory norms and standards for research involving human
subjects in their own countries as well as applicable international norms and standards. No national or
international ethical, legal or regulatory requirement should reduce or eliminate any of the protections for
research subjects set forth in this Declaration.
11. Medical research should be conducted in a manner that minimises possible harm to the environment.
12. Medical research involving human subjects must be conducted only by individuals with the appropriate
ethics and scientific education, training and qualifications. Research on patients or healthy volunteers requires
the supervision of a competent and appropriately qualified physician or other health care professional.
13. Groups that are underrepresented in medical research should be provided appropriate access to
participation in research.
14. Physicians who combine medical research with medical care should involve their patients in research only to
the extent that this is justified by its potential preventive, diagnostic or therapeutic value and if the physician has
good reason to believe that participation in the research study will not adversely affect the health of the patients
who serve as research subjects.
15. Appropriate compensation and treatment for subjects who are harmed as a result of participating in
research must be ensured.
(Q.36) Plastic finger prints are available on :
(a)

Finger print taken on paper

(b)

Finger print taken on plastic

(c)

Finger print taken on soap

(d)

Finger print made visible by iodine vapour

Your Response :

Correct Answer :

Exp:

Finger print taken on soap


Plastic fingerprints are finger tip impressions left on soft materials like dust, soap or wax. Iodine vapour is used to
visualize the latent finger prints.

(Q.37) A 30-year-old male from West Bengal presents to you with hyperkeratosis, and transverse nail lines. Most likely cause is
(a)

Chronic arsenic poisoning

(b)

Chronic lead poisoning

(c)

Chronic mercury poisoning

(d)

Acute arsenic poisoning

Your Response :

Correct Answer :

Exp:

Chronic arsenic poisoning


Through the water supply in W. Bengal the arsenic comes into the system causing rain drop pigmentation on
the back, Mees lines, Nail Leukonychia, and palmoplantar hyperkeratosis.

http://dbmci.examonair.com/Result/ShowAllQuestionInHtml.aspx?testid=5615

13/111

12/25/2014

Dr Bhatia Medical Coaching Institute:Online Test Platform


Chronic arsenic poisoning causes some skin signs and symptoms such as- Hyperkeratosis (mostly at distal parts),
Desquamation (of palms and soles), Raindrop pigmentation, Mees lines (transverse white striae of the fingernails
technically called as striate leukonychia).
Chronic Arsenic poisoning also causesGIT symptomsloss of weight, malaise, loss of appetite, salivation, colicky pain abdomen, constipation.
Hematological symptomsanemia-normocytic, normochromic and leucopoenia, thrombocytopenia.
Peripheral neuropathy sensory and motor polyneuritis (sensory symptoms predominate), numbness and
tingling.
Arsenic exposure can increase the risk of internal malignancy such as lung, bladder, kidney and prostate.
Signs of previous arsenic exposure include hyperkeratotic lesions in palms and soles, diffuse truncal
hyperpigmentation and multiple squamous cell carcinomas.
(Q.38) All of the following include grievous injuries, except:
(a)

Loss of testes

(b)

Loss of an eye

(c)

Loss of a kidney

(d)

Abrasion of the face

Your Response :

Correct Answer :

Exp:

Abrasion of the face


An abrasion on the face is not a grievous injury, rather it is a simple injury. As per Sec. 320 of Grievous Hurt,
Grievous injury includes1. Emasculation.
2. Permanent privation of sight of either eye.
3. Permanent privation of hearing or either ear.
4. Privation of any member or joint.
5. Destruction or permanent impairing of the power of any member or joint.
6. Permanent disfiguration of the head or face.
7. Fracture or dislocation of a bone or tooth.
8. Any hurt (1) which endangers life or (2) which causes the victim to be in severe bodily pain for 20 days or (3)
unable to follow his ordinary pursuits for a period of 20 days.
A Simple injury or Slight injury is an injury which is neither extensive nor serious and which heals rapidly without
leaving any permanent deformity or disfiguration.
An abrasion heals completely without leaving any deformity or disfiguration on the face and is therefore a simple
injury.

(Q.39) Floating Water- Lily sign is seen in:


(a)

Hydatid cyst

(b)

Aspergillosis

(c)

Tuberculosis cavity

(d)

Amoebic liver abscess

Your Response :

Correct Answer :

Exp:

Hydatid cyst
Radiographic and related imaging studies are important in detecting and evaluating echinococcal cysts. Plain films
will define pulmonary cysts of E. granulosus usually as rounded masses of uniform density but may miss cysts
in other organs unless there is cyst wall calcification (as occurs in the liver). MRI, CT, and ultrasound reveal well
defined cysts with thick or thin walls. When older cysts contain a layer of hydatid sand that is rich in accumulated
scolices, these imaging methods may detect this fluid layer of different density Floating Water- Lily sign.
However the most path gnomonic finding, if demonstrable, is that of daughter cysts within the larger cyst. This
finding, like eggshell or mural calcification on CT, is indicative of E. granulosus infections and helps to distinguish
the cyst form carcinomas, bacterial or amebic liver abscesses, or hemangiomas.

(Q.40) A dead body is having cadaveric lividity of bluish green colour. The most likely cause of death is by poisoning due to:
(a)

Hydrocyanic acid

(b)

Hydrogen sulphide

(c)

Oleander

(d)

Sodium nitrite

Your Response :

Correct Answer :

Exp:

Hydrogen sulphide
Postmortem Lividity
The incidence, extent and degree of fixation of postmortem lividity is important.
In an advanced case of sudden death, it presents as a series of mottled patches over the dependant parts in about
1-3 hours. These patches coalesce in about 3-6 hours. The lividity is fully developed and fixed in about 6-8 hours.
Lividity will not be seen (1) if the body is constantly altering its position, e.g. drowning (2) if the skin is dark, and
(3) if too much blood is lost. e.g. in massive hemorrhage.
The hypostatic areas have distinct co lour in certain cases of poisoning :
Hydrogen sulphide
Carbon monoxide

http://dbmci.examonair.com/Result/ShowAllQuestionInHtml.aspx?testid=5615

Bluish green
Bright cherry red

14/111

12/25/2014

Dr Bhatia Medical Coaching Institute:Online Test Platform


Nitrites; potassium

Chocolate or copper

chlorate, nitrobenzene,

brown

acetanilide, bromates
Opium

Almost black

Phosphorus

Dark brown

Potassium cyanide

Pink

(Q.41) Fracture-ala signature is :


(a)

Gutter fracture

(b)

Depressed fracture

(c)

Ring fracture

(d)

Sutural separation

Your Response :

Correct Answer :

Exp:

Depressed fracture
This is caused by a heavy weapon with a small striking surface, e.g. hammer.
The fractured bone is driven inward and its shape may indicate the type of weapon with which it is produced. It is
therefore also known as a signature fracture.
A late effect of depressed skull fracture may be post traumatic epilepsy.
Comminuted fracture:
This is caused by vehicular accidents, falls from height, and blows from weapons a with a large striking surface,
e.g. a heavy iron bar.
It is often a complication of a depressed fracture and has a stellate appearance when there is no displacement of
fragment.
Pond or indented fracture: Pond fracture occurs in children due to elasticity of their skull bones.
Fissured fractures may be seen round the periphery of the dent.
Gutter fracture: When a part of the thickness of skull bone is removed, e.g. in glancing bullet wounds, the fracture
is known as a gutter fracture.
Penetrating fracture: This is a clean cut opening due to a penetrating weapon, such as a, dagger or a bullet.
Elevated fracture: This is the result of a blow from a moderately heavy sharp edged weapon, eg, axe, machete,
which elevate one end of bone above the surface of the skull while the other end may dip down in the cranial
cavity injuring dura or the brain.

(Q.42) In methyl alcohol poisoning there is CNS depression, cardiac depression and optic nerve atrophy. These effects are produced due to:
(a)

Formaldehyde and formic acid.

(b)

Acetaldehyde

(c)

Contact shot entry

(d)

Acetic acid

Your Response :

Correct Answer :

Exp:

Formaldehyde and formic acid.


Toxin: Methanol
Features: Methanol metabolized to formate, causing profound metabolic acidosis and ocular toxicity.
CNS depression, cardiac depression and optic nerve atrophy
Management
Methanol assays not widely available as an urgent assay.
Diagnosis usually made on presence of a high anion gap, osmolal gap (> 10m Osm).
Antidotal treatment inhibits alcohol dehydrogenase; includes oral ethanol, i.v. ethanol or i.v. 4-methylpyrazole
(nonsedating).
Hemodialysis should be considered in severe cases.

(Q.43) The best prognostic indicator of hepatitis in

Neonates is :(a)

SGPT

(b)

Prothrombin time

(c)

Albumin

(d)

Bilirubin

Your Response :

Correct Answer :

Exp:

Prothrombin time
Hepatitis in neonate due to specific causes usually is distinguished from the term neonatal hepatitis, which has
been used to designate hepatitis inflammation of unknown cause. The levels of serum aminotransferases and
Bilirubin are poor predictors of outcome. Because of the short half - life of the coagulation proteins, the
prothrombin time (PT) is the best prognostic indicator.

(Q.44) At 28 weeks of age, an infant can perform all of the following tasks except:
(a)

Sits with support

(b)

Transfers object from one hand to other

http://dbmci.examonair.com/Result/ShowAllQuestionInHtml.aspx?testid=5615

15/111

12/25/2014

Dr Bhatia Medical Coaching Institute:Online Test Platform


(c)

Uncovers hidden toy

(d)

Enjoys mirror

Your Response :

Correct Answer :

Exp:

Uncovers hidden toy


Pediatric milestones is a very important factual scoring opportunity in the entrance examination.however it is
complicated by its varied timecourse described in different standard texts.
Here I have made an attempt of drawing a consensus from all standard texts and providing a comprehensive list
all together.you will have to mug it up.
GROSS MOTOR MILESTONES
Neck holding

3m
4m

Bidextrous reach

5m

Sits with support

6m

Sits in tripod position

8m

Sits without support

9m

Stands with support

10m

Walks with support

11m

Crawling and Pivoting

12m

Stands without support

15m

Walks without support

18m

Explores drawers, running, walks backwards

24m

Jumps, walking upstairs 2 feet/step

36m

Rides tricycle, walks upstairs 1 feet/step and downstairs 2 feet/step

4 yrs

Walks downstairs 1 feet/step

6 yrs

Hops on both legs

FINE MOTOR MILESTONES


Grasp reflex disappears

3- 4 m
4m

Mouthing starts, Hand regard (11-20 weeks), bidextrous reach

5m

Bidextrous grasp,

6m

Transfers object from one hand to another, Unidextrous reach, palmar grasp

7m

Unidextrous/Ulnar grasp

8m

Radial grasp

9m

immature pincer grasp

10m

Feeds with much spilling

11m

Mouthing stops

12m

Mature pincer grasp

15m

Feeds without much spilling, imitates, scribbles

18m

Turns 2-3 pages at a time

2 yrs

Turns 1 page at a time, puts on socks

2.5 yrs

Threads a bead

3 yrs

Dresses/undresses, Handedness develops, uses knife

4-5 yrs

Ties shoelaces

LANGUAGE MILESTONES
1m

Turns head to sound

2m

Vocalizes

3m

Cooing and babbling

4m

Laughs aloud

6m

Monosyllables

9m

Bisyllables

12m

Speaks 1st true word, repetition, knows 2 words with meaning

15-18m

Jargon speech

18m

Knows 10 words with meaning

2yrs

Uses pronouns, points 1 part of body, speaks simple sentence with 3 words.

3yrs

Knows name and sex, shares toys,asks questions

4yrs

Tells a story, plays co-operatively in a group,goes to toilet alone

5yrs

Names 5 colors, helps in hosehold tasks

SOCIAL MILESTONES
1m

Face regard-looks intently at mother

2m

Social smile

3m

Recognizes mother, anticipates feeds, shows interest in surroundings

6m

Smiles at mirror image, likes and dislikes develop

7m

Stranger anxiety, resists when toy is pulled

9m

Wavws bye-bye, pulls mother to attract her

11m

Holds arm out for shirt and feet for sleeves

http://dbmci.examonair.com/Result/ShowAllQuestionInHtml.aspx?testid=5615

16/111

12/25/2014

Dr Bhatia Medical Coaching Institute:Online Test Platform


12m

Comes when name is called, understands phrases, plays simple ball game,

2yrs

Asks for food, pulls people to show toys.

BOWEL AND BLADDER MELISTONES


1.5-2yrs

Start toilet training

1.5 yrs

Bowel control is achieved

1.5-2 yrs

Dry by day

2 yrs

50 % dry by night

3 yrs

75 % dry by night

5 yrs

90 % dry by night

PAPER-WORK MILESTONES
18m

Spontaneous scribbling

2yrs

Copies horizontal line

3 yrs

Draws a circle

4yrs

Draws a rectangle or a plus sign

5yrs

Draws a triangle or a tilted cross

6 yrs

Draws a hexagon

7 yrs

Draws a kite

8yrs

Draws a double cross

9yrs

Draws a cylinder

11 yrs

Draws a cube.

PLAY MILESTONES
10m

Plays peek-a-boo

1yr

Egocentric pretend play

2yr

Parallel play

4yr

Plays with others

5yr

Domestic play.

MISCELLANEOUS MILESTONES
3m

Ocular fixation occurs

4m

Binocular vision is established

10m

Uncovers hidden toys

18m

Rapproachment develops.

(Q.45) True regarding enuresis is :


(a)

Twin studies show that there is a marked familial pattern

(b)

Prevalence of enuresis at age 5 years is 3 % for males and 2% for females

(c)

Management of a child with enuresis begins with pharmacologic treatment

(d)

Consistent dry bed training is usually not helpful.

Your Response :

Correct Answer :

Exp:

Twin studies show that there is a marked familial pattern


The diagnosis of enuresis is made when urine is voided twice a week for at least 3 consecutive months or clinically
significant distress occurs in areas of child's life as a result of wetting. Twin studies show that there is a marked
familial pattern. There is 68% concordance in monozygotic twins and 36 % concordance rate in dizygotic twins.
The prevalence of enuresis age 5 year is 7 % for males and 3 % for females. At age 10 year, it is 3 % for males and
2% for females, and at 18 years it is 1 % in males and extremely rare in females.
Management of the child with enuresis should begin with behavioral therapy such as rewarding for dry nights,
voiding before going to bed and avoiding punishment or humiliating the child.
Consistent dry bed training should be the cornerstone in treatment of enuresis and is very useful. Consistent dry
bed training with positive reinforcement has a success rate of 85% or more.

(Q.46) Important side effect of felbamate is?


(a)

Renal failure

(b)

Aplastic anemia

(c)

Severe allergy

(d)

Severe vomiting

Your Response :

Correct Answer :

Exp:

Aplastic anemia
Felbamateis ananti-epileptic drugused in the treatment ofepilepsy. It is used to treatpartial seizures(with
and without generalization) in adults and partial and generalized seizures associated withLennox-Gastaut
syndromein children. However, an increased risk of potentially fatalaplastic anemiaand/orliver failure
limit thedrugsusage to severe refractoryepilepsy.

(Q.47) The success rate for establishing good bile flow following Kasai operation for extrahepatic biliary atresia is about 90% if done before

the age of:

http://dbmci.examonair.com/Result/ShowAllQuestionInHtml.aspx?testid=5615

17/111

12/25/2014

Dr Bhatia Medical Coaching Institute:Online Test Platform


(a)

2 weeks

(b)

8 weeks

(c)

16 weeks
24 weeks

(d)
Your Response :

Correct Answer :

Exp:

8 weeks
Hepatoportoenterostomy procedure of Kasai which is performed for correction of extrahepatic V biliary atresia
carries a success rate of 90 % ~ if performed before the age of 8 weeks.
Success rate for establishing good bile flow following Kasai operation done after 8 weeks of age goes on
decreasing. Therefore, the importance of early referral and prompt evaluation of infants with suspected biliary
atresia is emphasized.
Kasai operation before the 7th wk of life increases the success rate of this technique significantly. Children with
cirrhosis at the time of diagnosis should be evaluated for primary liver transplantation. gammaGT and bilirubin 5
weeks after Kasai operation may be useful markers for the success of this procedure. Patients with a gammaGT >
100 U/l and a bilirubin level >5mg/dl should be followed closely and should be evaluated for liver transplantation
early.

(Q.48) In Downs syndrome, false is:


(a)

Increased PAPP-A

(b)

Increased Beta-HCG

(c)

Absent nasal bone

(d)

Reversal of a wave in ductus venosus

Your Response :

Correct Answer :

Exp:

Increased PAPP-A
PAPP is decreased in Downs syndrome
First trimester markers (Combined Test)
Pregnancy Associated Plasma Protein-A (PAPP-A)
free -human chorionic gonadotrophin (free -hCG)
Nuchal translucency (NT)
Second trimester markers (Quadruple Test)
Alpha-fetoprotein (AFP) unconjugated oestriol (uE3)
free -human chorionic gonadotrophin (free -hCG) Inhibin-A (inhibin)
Integrated test -when Ist and 2nd trimester tests both done.
In the first trimester of pregnancy the PAPP-A level is, on average, low in Down's syndrome pregnancies (about
half that of unaffected pregnancies), and the nuchal translucency measurement and free -hCG levels are, on
average, high (about double that of unaffected pregnancies). In the second trimester AFP and uE3 levels are, on
average, low (about three-quarters that of unaffected pregnancies) and inhibin and free -hCG levels are, on
average, high (about double that of unaffected pregnancies).
Antenatal USG and Doppler studies
Aneuploidy score, definitions:
Nuchal translucency [NT] according to Nicola ides normal NT < 95thcentileabnormal NT > 95th centile (mild
abnormal NT was below 3,9 mm) in trisomies
Nasal bone normal, visible abnormal, not visible in trisomies
Ductus venosus [DV] Doppler normal: a wave, positive abnormal: a wave absent or reverse in trisomies

(Q.49) Ophthalmic exam to screen for retinopathy of prematurity is mandatory for newborns with birth weight less than:
(a)

1500 grams

(b)

2000 grams

(c)

1000 grams

(d)

2500 grams

Your Response :

Correct Answer :

Exp:

1500 grams
All infants with a birth weight under 1500 grams (gestational age less than 32 weeks) and those between 1500
and 2000 grams (32-34 weeks) who received oxygen therapy should have an eye exam to screen for retinopathy
of prematurity (ROP). Thus screening for Rap is mandatory for newborns with birth weight less than 1500 grams.

(Q.50) Single best growth chart indicator of acute undernutrition is :


(a)

Weight for age

(b)

Height for age

(c)

Weight for height

(d)

Mid upper arm circumference

Your Response :

Correct Answer :

Exp:

Weight for height


Weight for height below the 5th percentile is the single best growth chart indicator for acute undernutrition.

http://dbmci.examonair.com/Result/ShowAllQuestionInHtml.aspx?testid=5615

18/111

12/25/2014

Dr Bhatia Medical Coaching Institute:Online Test Platform


Children with chronic malnutrition may be short as well as thin, so that their weight for height curves may appear
relatively normal.
Weight for height
Actual weight of child
= --------------------------------------------------------

x 100

50th percentile of expected weight for the


actual height of child
(Q.51) On a fetal ultrasonography at 15 - 20 weeks, nuchal pad thickening is considered significant if it is more than:
(a)

10 mm

(b)

6 mm

(c)

2 mm
4 mm

(d)
Your Response :

Correct Answer :

Exp:

6 mm
Thenuchal fold thicknessis a parameter that is measured in asecond trimester scan(18-22 weeks) and it not to be
confused withnuchal translucency(which is measured in thefirst trimester).
Pathology
The proposed aetiology of increased nuchal thickness is as result ofcongenital heart diseaseand lymphatic
obstruction. Although as the pregnancy progresses excess nuchal thickness or evencystic hygromascan resolve,
the risk of karyotypic abnormalities is not reduced.
Associations
aneuploidy:
thetrisomies
Turner syndrome
congenital heart disease
Natural course
Most thickened nuchal folds tend to resolve towards the third trimester, but that does not decrease the increased
risk of aneuploidy anomalies.
Radiographic assessment
Antenatal ultrasound
The nuchal thickness is measured on an axial section through the head and the level of thethalami,cavum septi
pellucidumand cerebellar hemispheres (i.e in the same plane that is used to assess posterior fossa structures).
One caliper should be placed at the skin, and the other against the outer edge of the bone of the occiput.
It is ideally obtained with the long axis of the head at (or as close as possible to) 45% to the transducer)
An abnormal value is one that is more than 6 mm in thickness.
A thick nuchal fold is often considered the most sensitive1and most specific3(best) 2ndtrimester marker forDown
syndrome1with false positive rates as low as 1%.

(Q.52) Following organ is most commonly injured during birth:


(a)

Liver

(b)

Spleen

(c)

Adrenals
Ureter

(d)
Your Response :

Correct Answer :

Exp:

Liver
Though intra - Abdominal birth trauma is uncommon; liver is the most commonly injured solid organ during
birth, spleen & Adrenal injuries can occur.
Facial nerve injury is the most common peripheral nerve injury in neonates. clavicle fracture is the most
commonly injured bone during delivery.

(Q.53) The following is not the clinical feature of Guillain-Barre syndrome :


(a)

Pain in the muscle fibres

(b)

Symmetrical weakness

(c)

Diminished tendon reflex

(d)

Headache

Your
Response :
Correct
Answer :
Exp:

D
Headache
Clinical features
Symmetric weakness of the muscles
Diminished reflexes

http://dbmci.examonair.com/Result/ShowAllQuestionInHtml.aspx?testid=5615

19/111

12/25/2014

Dr Bhatia Medical Coaching Institute:Online Test Platform


Paraesthesia
Pain in the muscle
Facial cranial nerve involvement
Diminished tendon reflexes
Autonomous nervous system is involved
Hypertension
Postural hypotension
(Q.54) Treatment of choice for the prophylaxis of Bronchial Asthma in a one year old child is?
(a)

Inhaled beta -2 agonists

(b)

Theophylline

(c)

Oral ketotifen

(d)

Oral Leukotriene receptor antagonists

Your Response :

Correct Answer :

Exp:

Inhaled beta -2 agonists


Inhaled beta-2 agonists are the drug of choice both for treatment as well as for prevention of an acute attack.
beta-2 agonists are of 2 types:
Short acting - Salbutamol
Long acting - Salmeterol, formoterol
Salbutamol is mainly used for aborting an acute attack whereas long acting 2-agonists are used for the
prophylaxis of asthma. Inhaled drugs are always preferred over oral drugs as they have:
Fewer systemic side effects
Drug directly reaches the site of action
Theophylline has low level of safety and is rarely used in pediatric asthma.
LT-modifiers are used in pediatric patient more than 6 years of age. Use in children < 6years of age is not
recommended.

(Q.55) Jitteriness can be distinguished from seizure by all except?


(a)

Autonomic involvement

(b)

Precipitated by a stimulus

(c)

Gaze

(d)

Frequency of movements

Your Response :

Correct Answer :

Exp:

Precipitated by a stimulus
Jitteriness is defined as rhythmic tremors of equal amplitude around a fixed axis. It is the MC diagnosis in case of
involuntary movements in a healthy full term infant.
Most apparent when infant is crying or being examined.
Clinical features that differentiate seizures from non epileptic events are:
Autonomic events such as tachycardia and increase in (B)P are common with seizures
Non epileptic movements are suppressed by gentle restraints.
Nystagmus, gaze palsy may be seen with epileptic seizures.
There is a significant increase in Prolactin after a true epileptic seizure.

(Q.56) False statement regarding following is ?


(a)

Metaplasia is replacement of mature cells of one type with cells of another type

(b)

Pleomorphism seen in both dysplasia and anaplasia

(c)

Dysplasia is irreversible

(d)

Polarity is lost in anaplasia

Your Response :

Correct Answer :

Exp:

Dysplasia is irreversible
Metaplasia

Dysplasia

Anaplasia

Replacement of mature cells


of one type with cells of
another type (Abnormal
differentiation)

Loss in the uniformity of


individual cells as well as
loss in architectural
orientation (Abnormal
differentiation and
maturation)

Lack of differentiation in
neoplastic cells

Reversibility

Seen

Partially reversible

Not seen

Organization and
orientation

Partially reversible

Not maintained

Not maintained

Polarity

Not lost

Lost

Lost

Other feature

MC epithelial metaplasia

Mitotic figures are more Atypical, bizarre


but in normal pattern
abundant mitotic
figures

Definition

(Abnormal
differentiation &
Maturation)

Remember: Pleomorphism, hyperchromatism seen in both dysplasia and anaplasia.

http://dbmci.examonair.com/Result/ShowAllQuestionInHtml.aspx?testid=5615

20/111

12/25/2014

Dr Bhatia Medical Coaching Institute:Online Test Platform


(Q.57) X-ray appearance of hyaline membrane disease includes ?
(a)

Homogeneous ground glass appearance

(b)

Air bronchgrams may be seen

(c)

Bilateral symmetric involvement

(d)

All of the above

Your Response :

Correct Answer :

Exp:

All of above
Hyaline membrane diseasealso known as neonatalrespiratory distress syndrome,lung disease of prematurity,
orsurfactant deficiencyrefers to lung pathology which results from insufficient production of surfactant.
Epidemiology
The incidence is estimated at 6 per 1000 births2.
Clinical presentation
Respiratory distress presents in the first few hours of life in a premature baby. Symptoms include tachypnoea,
expiratory grunting, nasal flaring. The infant may or may not be cyanosed. Substernal and intercostal retractions
may be evident.
Risk factors includematernal diabetes, greater prematurity, prenatal asphyxia and multiple gestation.
Associated abnormalities are those that can occur in prematurity: intracranial haemorrhage,necrotising
enterocolitis,patent ductus arteriosus, delayed developmental milestones, hypothermia and hypoglycaemia.
Pathophysiology
Immature type II pneumocytes cannot producesurfactant. The lack of surfactant lowers the surface tension in
alveoli causing collapse. Patients have a decreased lecithin:sphingomyelin ratio. Damaged cells, necrotic cells, and
mucus line the alveoli.
Radiographic features
Plain film - chest radiograph
Typically gives diffuse ground glass lungs with low volumes and a bell-shaped thorax
Often tends to be bilateral and symmetrical
Amay be evident
Hyperinflation (in a non ventilated patient) excludes the diagnosis
Radiographs may show hyperinflationifthe patient is intubated
RDS can be safely excluded if the neonate has a normal chest X-ray at 6 hours after birth.
If treated with surfactant thereapy there may be asymmetric improvement.
Treatment and prognosis
Exogenous surfactant administration. Supportive oxygen therapy.
Complications
Acute
persistent patent ductus arteriosus (PDA)due to reduced oxygen stimulus
barotrauma(from treatment)
oxygen toxicity (from treatment)
pulmonary haemorrhage(can also included in the differential diagnosis)
Chronic
bronchopulmonary dysplasia
pulmonary interstital emphysema
recurrent pulmonary infection
subglottic stenosis(from intubation)
Differential diagnosis
congenital heart disease
group B Strep pneumonia
pulmonary haemorrhage
pulmonary oedema/pulmonary venous congenstion
neonatal pneumonia

(Q.58) False statement regarding synovial fluid is ?


(a)

Produced by Type A synovial cells

(b)

Has thixotropic characteristics

(c)

Exhibits non-Newtonian flow characteristics

(d)

None of the above

Your Response :

Correct Answer :

Exp:

Produced by Type A synovial cells


Synovial tissue is sterile and composed of vascularized connective tissue that lacks a basement membrane.
Two cells type (type A and type B) are present: Type B produce synovial fluid.
Synovial fluid is made of hyaluronic acid and lubricin, proteinases, and collagenases.
Synovial fluid exhibitsnon-Newtonian flow characteristics; the viscosity coefficient is not a constant and the fluid
is not linearly viscous.
Synovial fluid hasthixotropiccharacteristics; viscosity decreases and the fluid thins over a period of continued
stress.
Normal synovial fluid contains 3-4 mg/mlhyaluronan(hyaluronic acid), a polymer ofdisaccharides composed of D-

http://dbmci.examonair.com/Result/ShowAllQuestionInHtml.aspx?testid=5615

21/111

12/25/2014

Dr Bhatia Medical Coaching Institute:Online Test Platform


glucuronic acid and D-N-acetyl glucosamine joined by alternating beta-1,4 and beta-1,3 glycosidic bonds
Hyaluronan is synthesized by the synovial membrane and secreted into the joint cavity to increase the viscosity
and elasticity of articular cartilages and lubricate the surfaces betweensynoviumand cartilage.
Synovial fluid containslubricin secreted by synovial cells. It is chiefly responsible for so-called boundary-layer
lubrication, which reduces friction between opposing surfaces of cartilage.
There is also some evidence that it helps regulate synovial cell growth.
Its functions are :reducing friction by lubricating the joint, absorbing shocks, and supplying oxygen and nutrients
to and removing carbon dioxide and metabolic wastes from the chondrocytes within articular cartilage.
It also contains phagocytic cells that remove microbes and the debris that results from normal wear and tear in
the joint.
Synovial fluid can be collected by syringe in a procedure termedarthrocentesis, also known as joint aspiration.
Classification
Synovial fluid can be classified into normal, noninflammatory, inflammatory, septic, and hemorrhagic:
Classification of synovial fluid in an adultkneejoint
Normal

Noninflammatory Inflammatory

Septic

Hemorrhagic

Volume (ml)

<3.5

>3.5

>3.5

>3.5

>3.5

Viscosity

High

High

Low

Mixed

Low

Clarity

Clear

Clear

Cloudy

Opaque

Mixed

Color

Colorless/straw

Straw/yellow

Yellow

Mixed

Red

WBC/mm3

<200

200-2,000

2,000-75,000

>100,000

Same as blood

Polys(%)

<25

<25

>50

>75

Same as blood

Gram stain

Negative

Negative

Negative

Often positive

Negative

(Q.59) Identify the curve displayed in the adjoining diagram.

(a)

SiggaardAndersen curve nomogram

(b)

Oxygen-Hemoglobin dissociation curve

(c)

Spirometry curve

(d)

CO2 response curve

Your Response :
Correct Answer :
Exp:

A
SiggaardAndersen curve nomogram
REF : Ganong's Review of Medical Physiology, Twenty-Third Edition, Chap 40
The SiggaardAndersen Curve Nomogram
Use of the SiggaardAndersen curve nomogram to plot the acidbase characteristics of arterial blood is helpful in
clinical situations. This nomogram has PCO2 plotted on a log scale on the vertical axis and pH on the horizontal
axis. Thus, any point to the left of a vertical line through pH 7.40 indicates acidosis, and any point to the right
indicates alkalosis. The position of the point above or below the horizontal line through a PCO2 of 40 mm Hg
defines the effective degree of hypoventilation or hyperventilation. If a solution containing NaHCO3 and no
buffers were equilibrated with gas mixtures containing various amounts of CO2, the pH and PCO2 values at
equilibrium would fall along the dashed line on the left in Figure 407 or a line parallel to it. If buffers were
present, the slope of the line would be greater; and the greater the buffering capacity of the solution, the steeper
the line. For normal blood containing 15 g of hemoglobin/dL, the CO2 titration linepasses through the 15-g/dL
mark on the hemoglobin scale (on the underside of the upper curved scale) and the point where the PCO2 = 40
mm Hg and pH = 7.40 lines intersect, as shown in Figure 407. When the hemoglobin content of the blood is low,
there is significant loss of buffering capacity, and the slope of the CO2 titration line diminishes. However, blood of
course contains buffers in addition to hemoglobin, so that even the line drawn from the zero point on the
hemoglobin scale through the normal PCO2pH intercept is steeper than the curve for a solution containing no
buffers.
For clinical use, arterial blood or arterialized capillary blood is drawn anaerobically and its pH measured. The pHs
of the same blood after equilibration with each of two gas mixtures containing different known amounts of CO2
are also determined. The pH values at the known PCO 2 levels are plotted and connected to provide the CO2
titration line for the blood sample. The pH of the blood sample before equilibration is plotted on this line, and the
PCO 2 of the sample is read off the vertical scale. The standard bicarbonatecontent of the sample is indicated by
the point at which the CO2 titration line intersects the bicarbonate scale on the PCO2 = 40 mm Hg line. The
standard bicarbonate is not the actual bicarbonate concentration of the sample but, rather, what the bicarbonate
concentration would be after elimination of any respiratory component. It is a measure of the alkali reserve of the
blood, except that it is measured by determining the pH rather than the total CO2 content of the sample after

http://dbmci.examonair.com/Result/ShowAllQuestionInHtml.aspx?testid=5615

22/111

12/25/2014

Dr Bhatia Medical Coaching Institute:Online Test Platform


equilibration. Like the alkali reserve, it is an index of the degree of metabolic acidosis or alkalosis present.
Additional graduations on the upper curved scale of the nomogram are provided for measuring buffer base
content; the point where the CO2 calibration line of the arterial blood sample intersects this scale shows the
mEq/L of buffer base in the sample. The buffer base is equal to the total number of buffer anions (principally
Prot, HCO3, and Hb) that can accept hydrogen ions in the blood. The normal value in an individual with 15 g of
hemoglobin per deciliter of blood is 48 mEq/L.
The point at which the CO2 calibration line intersects the lower curved scale on the nomogram indicates the base
excess.This value, which is positive in alkalosis and negative in acidosis, is the amount of acid or base that would
restore 1 L of blood to normal acidbase composition at a PCO2 of 40 mm Hg. It should be noted that a base
deficiency cannot be completely corrected simply by calculating the difference between the normal standard
bicarbonate (24 mEq/L) and the actual standard bicarbonate and administering this amount of NaHCO3 per liter
of blood; some of the added HCO3 is converted to CO2 and H2O, and the CO2 is lost in the lungs. The actual
amount that must be added is roughly 1.2 times the standard bicarbonate deficit, but the lower curved scale on
the nomogram, which has been developed empirically by analyzing many blood samples, is more accurate.

(Q.60) Optically inactive amino acid is


(a)

Aspartate

(b)

Glutamate

(c)

Glycine
Lysine

(d)
Your Response :

Correct Answer :

Exp:

Glycine
Tetrahedral carbon atom with 4 distinct constituents is said to be chiral. The one amino acid not exhibiting
chirality is glycine since it's "R-group" is a hydrogen atom.
H-CH-COOH
I
NH2
Glycine
Chirality describes the handedness of a molecule that is observable by the ability of a molecule to rotate the
plane of polarized light either to the right (dextrorotatory) or to the left (levorotatory).
All of the amino acids in proteins exhibit the same absolute steric configuration as L-glyceraldehyde. Therefore,
they are all L-a-amino acids. D-amino acids are never found in proteins, although they exist in nature. D-amino
acids are often found in polypeptide antibiotics.

(Q.61) Technique which is able to detect posttranslational modification of amino acids such as addition of phosphoryl, hydroxyl group etc.
(a)

Capillary Electrophoresis

(b)

Mass spectrometry

(c)

Density gradient centrifugation

(d)

Chromatography

Your Response :

Correct Answer :

Exp:

Mass spectrometry
Mass spectrometry, which discriminates molecules based solely on their mass, is ideal for detecting the
phosphate, hydroxyl and other groups on post-translationally modified amino acids. Each adds a specific and
readily identified increment of mass to the modified amino acid.
Uses of Mass spectrometry
Accurate molecular weight measurements:
Sample confirmation, to determine the purity of a sample, to verify amino acid substitutions, to detect posttranslational modifications, to 'calculate the number of disulphide bridges
Reaction monitoring:
To monitor enzyme reactions, chemical modification, protein digestion
Amino acid sequencing:
Sequence confirmation, de novo characterization of peptides, identification of proteins by database searching
with a sequence "tag" from a proteolytic fragment
Oligonucleotide sequencing:
The characterization or quality control of oligonucleotides
Protein structure:
Protein folding monitored by HID exchange, protein-ligand complex formation under physiological conditions,

http://dbmci.examonair.com/Result/ShowAllQuestionInHtml.aspx?testid=5615

23/111

12/25/2014

Dr Bhatia Medical Coaching Institute:Online Test Platform


macromolecular structure determination.
(Q.62) The following is a comparative depiction of dissociation curves of Hemoglobin and Myoglobin. Wrong statement regarding this curve

is ?

(a)

Curve A is for Hemoglobin.

(b)

Curve B is for Myoglobin

(c)

Myoglobin releases O2 only high at PO2 values

(d)

Myoglobin dissociation curve is a rectangular hyperbola.

Your Response :
Correct Answer :

C
Myoglobin releases O2 only high at PO2 values

Exp:

REF : Ganong's Review of Medical Physiology, Twenty-Third Edition, Chap 36


Myoglobin
Myoglobin is an iron-containing pigment found in skeletal muscle. It resembles hemoglobin but binds 1 rather
than 4 mol of O2 per mole. Its dissociation curve is a rectangular hyperbola rather than a sigmoid curve. Because
its curve is to the left of the hemoglobin curve , it takes up O2 from hemoglobin in the blood. It releases O2 only
at low PO2 values, but the PO2 in exercising muscle is close to zero. The myoglobin content is greatest in muscles
specialized for sustained contraction. The muscle blood supply is compressed during such contractions, and
myoglobin may provide O2 when blood flow is cut off. The myoglobin binding curve (B) lacks the sigmoidal shape
of the hemoglobin binding curve (A) because of the single O2 binding site in each molecule. Myoglobin also has
greater affinity for O2 than hemoglobin (curve shifted left) and thus can store O2 in muscle.
(Q.63) The adjoining diagram denotes the double helix structure of DN(A) Which of the following is true ?

(a)

Distance A is 34

(b)

Distance B is 20

(c)

The rise in distance per base pair is 3.4

(d)

The 2 DNA strands are parallel to each other

Your Response :

Correct Answer :

Exp:

The rise in distance per base pair is 3.4


REF : Harper's Illustrated Biochemistry, Twenty-Seventh Edition, Chap 34
The diagrammatic representation of the Watson and Crick model of the double-helical structure of the B form of
DNA is shown. The horizontal arrow indicates the width of the double helix (20 ), and the vertical arrow
indicates the distance spanned by one complete turn of the double helix (34 ). One turn of B-DNA includes 10
base pairs (bp), so the rise is 3.4 per bp. The central axis of the double helix is indicated by the vertical rod. The
short arrows designate the polarity of the antiparallel strands. The major and minor grooves are depicted. (A,
adenine; C, cytosine; G, guanine; T, thymine; P, phosphate; S, sugar [deoxyribose].) Hydrogen bonds between A/T
and G/C bases indicated by short, colored, horizontal lines.
The chemical nature of the monomeric deoxynucleotide units of DNAdeoxyadenylate, deoxyguanylate,
deoxycytidylate, and thymidylateis described in Chapter 32. These monomeric units of DNA are held in
polymeric form by 3',5'-phosphodiester bridges constituting a single strand, as depicted in Figure 341. The
informational content of DNA (the genetic code) resides in the sequence in which these monomerspurine and
pyrimidine deoxyribonucleotidesare ordered. The polymer as depicted possesses a polarity; one end has a 5'hydroxyl or phosphate terminal while the other has a 3'-phosphate or hydroxyl terminal. The importance of this
polarity will become evident. Since the genetic information resides in the order of the monomeric units within the
polymers, there must exist a mechanism of reproducing or replicating this specific information with a high degree
of fidelity. That requirement, together with x-ray diffraction data from the DNA molecule and the observation of
Chargaff that in DNA molecules the concentration of deoxyadenosine (A) nucleotides equals that of thymidine (T)
nucleotides (A = T), while the concentration of deoxyguanosine (G) nucleotides equals that of deoxycytidine (C)
nucleotides (G = C), led Watson, Crick, and Wilkins to propose in the early 1950s a model of a double-stranded

http://dbmci.examonair.com/Result/ShowAllQuestionInHtml.aspx?testid=5615

24/111

12/25/2014

Dr Bhatia Medical Coaching Institute:Online Test Platform


DNA molecule. The model they proposed is depicted in Figure 342. The two strands of this double-stranded helix
are held in register by both hydrogen bonds between the purine and pyrimidine bases of the respective linear
molecules and by van der Waals and hydrophobic interactions between the stacked adjacent base pairs. The
pairings between the purine and pyrimidine nucleotides on the opposite strands are very specific and are
dependent upon hydrogen bonding of A with T and G with (C) This common form of DNA is said to be righthanded because as one looks down the double helix, the base residues form a spiral in a clockwise direction. In
the double-stranded molecule, restrictions imposed by the rotation about the phosphodiester bond, the favored
anticonfiguration of the glycosidic bond and the predominant tautomers (see Figure 322) of the four bases (A, G,
T, and C) allow A to pair only with T and G only with C, as depicted in Figure 343. This base-pairing restriction
explains the earlier observation that in a double-stranded DNA molecule the content of A equals that of T and the
content of G equals that of (C) The two strands of the double-helical molecule, each of which possesses a polarity,
are antiparallel; ie, one strand runs in the 5' to 3' direction and the other in the 3' to 5' direction. This is
analogous to two parallel streets, each running one way but carrying traffic in opposite directions. In the doublestranded DNA molecules, the genetic information resides in the sequence of nucleotides on one strand, the
template strand. This is the strand of DNA that is copied during ribonucleic acid (RNA) synthesis. It is sometimes
referred to as the noncoding strand. The opposite strand is considered the coding strand because it matches the
sequence of the RNA transcript that encodes the protein

(Q.64) Carbon Dioxide in blood is mainly transported as:


(a)

Carbamates

(b)

Bound to albumin

(c)

Bicarbonate

(d)

Free form

Your Response :

Correct Answer :

Exp:

Bicarbonate
Metabolizing cells produce CO2 which diffuses into the blood and enters the circulating red blood cells (RBCs).
Within RBCs the CO2 is rapidly converted to carbonic acid through the action of carbonic anhydrase as shown in
the equation below:
CO2 + H2O ----> H2CO3

--- H+ + HCO3-

The bicarbonate ion produced in this dissociation reaction diffuses out of the RBC and is carried in the blood to
the lungs. This effective CO2 transport process is referred to as isohydric transport.
Approximately 80% of the CO2 produced in metabolizing cells is transported to the lungs in this way. A small
percentage of CO2 is transported in the blood as a dissolved gas.
(Q.65) High energy phosphates are produced in the following EXCEPT
(a)

HMP shunt

(b)

Oxidative pathway

(c)

TCA

(d)

Glycolysis

Your Response :

Correct Answer :

Exp:

HMP shunt
Hexose Monophosphate Shunt Pathway (HMP)produces NADPH that is required for several synthetic pathways,
e.g., for fatty acids and steroid synthesis. However, the NADPH produced is not converted to ATP or used for
energy production (unlike NADH produced in glycolysis and TCA cycle ) Devlin and Rama Rao
The HMP shunt or pentose phosphate pathway serves several purposes, including synthesis and degradation of
sugars other than hexoses, particularly pentoses is necessary for synthesis of nucleotides and other glycolytic
intermediates.
Most important is the ability to synthesize NADPH, which has a unique role in biosynthetic reactions.
The direction of flow and path taken by G6P after entry into the pathway is determined largely by the needs of
the cell for NADPH or sugar intermediates.
When more NADPH than ribose 5-phosphate is required, the pathway leads to complete oxidation of G6P to CO 2
and re-synthesis of G6P from ribulose 5-phosphate.
Alternatively, if more ribose 5-phosphate than NADPH is required, G6P is converted to fructose 6-phosphate and
glyceraldehyde 3- phosphate by the glycolytic pathway.
The distribution of the pentose phosphate pathway in tissues is consistent with its functions. It is present in
erythrocytes for production NADPH, required to generate reduced glutathione, which is essential for
maintenance of normal red cell structure.
It is also active in liver, mammary gland, testis, and adrenal cortex, sites of fatty acid or steroid synthesis that also
require the reducing power of NADPH.

(Q.66) TRUE regarding the main function of Apoprotein C


(a)

Cholesterol synthesis

(b)

TG transport

(c)

Increases activity of lipoprotein lipase

(d)

Fatty acid synthesis

Your Response :

Correct Answer :

Exp:

Increases activity of lipoprotein lipase

http://dbmci.examonair.com/Result/ShowAllQuestionInHtml.aspx?testid=5615

25/111

12/25/2014

Dr Bhatia Medical Coaching Institute:Online Test Platform


Lipases hydrolyze blood triacylglycerol for their fatty acids to become available to tissues.
Lipoprotein lipase hydrolyzes triacylglycerol in plasma lipoproteins.
Lipoprotein lipase is located on the surface of endothelial cells of capillaries and possible of adjoining tissue cells.
It hydrolyzes fatty acids from the 1 and/or 3 positions of tri-and diacylglycerols present in VLDL or chylomicrons.
One of the lipoprotein apoproteins (Apo C-II)must be present to activate the process. Fatty acids released are
either bound to serum albumin or taken up by the tissue.
Apoprotein C along with phospholipids helps in activating lipoprotein lipase for catalyzing the conversion of
Chylomicrons and VLDL into Chylomicron and VLDL remnants.

(Q.67) Okazaki fragments are formed in


(a)

Transcription

(b)

dsDNA replication

(c)

Single stranded DNA replication

(d)

Translation

Your Response :

Correct Answer :

Exp:

dsDNA replication
Okazaki fragments are short fragments of newly synthesized DNA strands produced during double stranded DNA
replication
All the known DNA polymerases can only synthesis DNA in one direction, the 5 to 3 direction.
However as the strands separate, replication forks will be moving along one parental strand in the 3 to 5
direction and 5 to 3 on the other parental strand.
On the former, the leading strand DNA can be synthesized continuously in the 5 to 3 direction.
On the later, the lagging strand DNA synthesis can only occur when a stretch of single stranded DNA has been
exposed and proceeds in the direction opposite to the movement of the replication fork (still 5 to 3).It is thus
discontinuous and the series of fragments are then covalently linked by ligases to give a continuous strand. Such
fragments were first observed by Okazaki using pulse labeling with radioactive thymidine.
In eukaryotes, Okazaki fragments are typically a few hundred nucleotides long, whereas in prokaryotes they may
contain several thousands of nucleotides.

(Q.68) The structure of a RNA molecule has been shown here. It represents ?

(a)

mRNA

(b)

tRNA

(c)

rRNA

(d)

None of the above

Your Response :

Correct Answer :

Exp:

tRNA
Ref : Harper's Illustrated Biochemistry, Twenty-Seventh Edition, Chap 34
Transfer RNA (tRNA)
Typical aminoacyl tRNA in which the amino acid (aa) is attached to the 3' CCA terminal. The anticodon, TC, and
dihydrouracil (D) arms are indicated, as are the positions of the intramolecular hydrogen bonding between these
base pairs. tRNA molecules vary in length from 74 to 95 nucleotides. They also are generated by nuclear
processing of a precursor molecule (Chapter 36). The tRNA molecules serve as adapters for the translation of the
information in the sequence of nucleotides of the mRNA into specific amino acids. There are at least 20 species of
tRNA molecules in every cell, at least one (and often several) corresponding to each of the 20 amino acids
required for protein synthesis. Although each specific tRNA differs from the others in its sequence of nucleotides,
the tRNA molecules as a class have many features in common. The primary structureie, the nucleotide
sequenceof all tRNA molecules allows extensive folding and intrastrand complementarity to generate a
secondary structure that appears in two dimensions like a cloverleaf. All tRNA molecules contain four main arms.
The acceptor arm terminates in the nucleotides CpCpAOH. These three nucleotides are added
posttranscriptionally by a specific nucleotidyl transferase enzyme. The tRNA-appropriate amino acid is attached,
or "charged" onto, the 3'-OH group of the A moiety of the acceptor arm. The D, TC, and extra arms help define a
specific tRN(A) Although tRNAs are quite stable in prokaryotes, they are somewhat less stable in eukaryotes. The
opposite is true for mRNAs, which are quite unstable in prokaryotes but generally more stable in eukaryotic
organisms.

http://dbmci.examonair.com/Result/ShowAllQuestionInHtml.aspx?testid=5615

26/111

12/25/2014

Dr Bhatia Medical Coaching Institute:Online Test Platform

(Q.69) All are substrates required for PCR except


(a)

T AQ Polymerase

(b)

d NTP

(c)

Primer

(d)

Radiolabelled DNA probe

Your Response :

Correct Answer :

Exp:

Radiolabelled DNA probe


Polymerase chain 'reaction is an example of enzyme based cloning.
Radiolabelled DNA probe is not required for it, is required for only hybridization techniques.
What is cloning?
Production of identical copies. Here, we are aimed at synthesizing identical copies of DNA with a single copy of
DN(A)
The enzyme involved is DNA Polymerase.
Three steps are involved:
Denaturation;
Since polymerase enzyme requires a single strand of DNA as the template, the first step is splitting the double
strand of DNA into two single strands which occurs at a high temperature of 94 degrees.
Annealing
The second requirement of polymerase enzyme is that it requires an RNA primer for initiating the synthesis,
hence we select RNA primers which are complementary to the flanking regions of the DNA to be amplified and
anneal them to the parent strand.
Elongation
Since the first step occurs at a higher temperature, we need a polymerase enzyme, which will be active at a
higher temperature also, hence polymerase enzyme ot an organism of hot spring, thermus aquaticus (Taq
polymerase) is selected to serve the purpose.
Further, the polymerase requires deoxy forms of all the nucleotides to join them in the sequence shown by the
template sequence.
Now the polymerase will start synthesizing the new strand according the parent or template strand.

(Q.70) All are actions of Endothelin 1 except


(a)
(b)
(c)
(d)

Bronchodilatation
Vasoconstriction
Decreased GFR
Vasodilatation

Your Response :

Correct Answer :

Exp:

Bronchodilatation
There are three isoforms of endothelin - Endothelin 1 ,2,3. Endothelin 1 is found in plasma
Endothelin 2 is found in kidney, G.I.T. (functional significance unknown)
Endothelin 3 is found in CNS
These endothelin act through two types of receptors:
Endothelin A
Endothelin B
Both are G protein coupled (Gq type) which act by increasing intracellular calcium ions.
But functionally they exert contrast properties:
Endothelin A causes vasoconstriction ~ decreased GFR Endothelin B causes vasodilatation by increasing NO
release.
Clinical importance: Thus, when we use endothelin antagonist for pulmonary hypertension, it is better to use
endothelin A selective antagonist (sitaxentan) instead of a nonselective antagonist (Bosentan)
Both are pro-mitogenic.
No specific action on bronchial smooth muscle.

(Q.71) Carnitine Palmitoyltransferase-I (CPT-I) is inhibited by


(a)

Malonyl-CoA

(b)

Pyruvate

(c)

Acetyl-CoA
Citrate

(d)
Your Response :

Correct Answer :

Exp:

Malonyl-CoA
Carnitine Palmitoyltransferase-I (CPT-I) activity is low in the fed state, leading to depression of fatty acid oxidation
and high in starvation, allowing fatty acid oxidation to increase.
Malonyl-CoA, the initial intermediate in fatty acid synthesis is a potent inhibitor of CPT-I.

http://dbmci.examonair.com/Result/ShowAllQuestionInHtml.aspx?testid=5615

27/111

12/25/2014

Dr Bhatia Medical Coaching Institute:Online Test Platform


(Q.72) The diagram represents a important structural arrangement of DNA chain and the histone octamer. False statement regarding it is ?

(a)

It represents structure of Nucleosome

(b)

The DNA is supercoiled in a right-handed helix over the surface of the disk-shaped histone octamer

(c)

The position of histone H1, when it is present, is indicated by the dashed outline at the bottom of the figure.

(d)

1.75 superhelical turns of DNA are wrapped around the surface of the histone octamer

Your Response :

Correct Answer :

Exp:

The DNA is supercoiled in a right-handed helix over the surface of the disk-shaped histone octamer
(Ref. Harper biochemistry 28th, Chap 35)
The diagram depics the structure of the nucleosome, in which DNA is wrapped around the surface of a flat
protein cylinder consisting of two each of histones H2A, H2B, H3, and H4 that form the histone octamer. The 146
base pairs of DNA, consisting of 1.75 superhelical turns, are in contact with the histone octamer. This protects the
DNA from digestion by a nuclease. The position of histone H1, when it is present, is indicated by the dashed
outline at the bottom of the figure.
When the histone octamer is mixed with purified, doublestranded DNA under appropriate ionic conditions, the
same x-ray diffraction pattern is formed as that observed in freshly isolated chromatin. Electron microscopic
studies confirm the existence of reconstituted nucleosomes. Furthermore, the reconstitution of nucleosomes
from DNA and histones H2A, H2B, H3, and H4 is independent of the organismal or cellular origin of the various
components. Neither the histone H1 nor the nonhistone proteins are necessary for the reconstitution of the
nucleosome core. In the nucleosome, the DNA is supercoiled in a left-handed helix over the surface of the diskshaped histone octamer. The majority of core histone proteins interact with the DNA on the inside of the
supercoil without protruding, although the amino terminal tails of all the histones probably extend outside of this
structure and are available for regulatory covalent modifications
The (H3-H4)2 tetramer itself can confer nucleosome-like properties on DNA and thus has a central role in the
formation of the nucleosome. The addition of two H2A-H2B dimers stabilizes the primary particle and firmly binds
two additional half-turns of DNA previously bound only loosely to the (H3-H4)2. Thus, 1.75 superhelical turns of
DNA are wrapped around the surface of the histone octamer, protecting 146 base pairs of DNA and forming the
nucleosome core particle. In chromatin, core particles are separated by an about 30-bp region of DNA termed
"linker." Most of the DNA is in a repeating series of these structures, giving the so-called "beads-on-a-string"
appearance when examined by electron microscopy. The assembly of nucleosomes is mediated by one of several
nuclear chromatin assembly factors facilitated by histone chaperones, a group of proteins that exhibit highaffinity histone binding. As the nucleosome is assembled, histones are released from the histone chaperones.
Nucleosomes appear to exhibit preference for certain regions on specific DNA molecules, but the basis for this
nonrandom distribution, termed phasing, is not yet completely understood. Phasing is probably related to the
relative physical flexibility of certain nucleotide sequences that are able to accommodate the regions of kinking
within the supercoil as well as the presence of other DNA-bound factors that limit the sites of nucleosome
deposition.

(Q.73) Arachidonic acid contains :


(a)

One double bond

(b)

Two double bonds

(c)

Three double bonds

(d)

Four double bonds

Your
Response :
Correct
Answer :
Exp:

D
Four double bonds
Unsaturated fatty acids:
Monoenoic acids (one double bond):
Palmitoleic acid, Oleic acid, Elaidic acid
Dienoic acids (two double bonds): linoleic acid
Trienoic acids (three double bonds): -Linolenic acid, Linolenic acid
Tetraenoic acids (Four double bonds): Arachidonic acid
Pentaenoic acids (Five double bonds): Timnodonic acid
Hexanoic acids (Six double bonds): Cervonic acids

(Q.74) Millennium development goals were adopted at:


(a)

New York

(b)

Geneva

(c)

Rome

(d)

Alma - Ata

http://dbmci.examonair.com/Result/ShowAllQuestionInHtml.aspx?testid=5615

28/111

12/25/2014

Dr Bhatia Medical Coaching Institute:Online Test Platform


Your Response :

Correct Answer :

Exp:

New York
Millennium development goals were adopted in September 2000 in millennium summit at New York.
On 17 December 1998, the General Assembly adopted resolution 53/202 by which it decided to convene the
Millennium Summit of the United Nations as an integral part of the Millennium Assembly of the United Nations in
order to address the above-mentioned problem. The Summit opened at the United Nations Headquarters in New
York on 6 September 2000. The MDGs consolidated by the Millennium Declaration adopted at the conclusion of
the Millennium Summit as the ever comprehensive international framework which represents the ambitious
quest for bridging the gap among and within world communities. They also represent collective will and resolve of
governments, responding to the world's main development challenges and to the calls of civil society, to address
the issues ranging from eradication of poverty, better health conditions through combating the HIV/AIDS and
other diseases, better education, maternal health, child mortality, gender The MDGs are an agreed set of goals
that were intended to and can certainly be achieved by 2015 deadline, if all actors, public, private and even
individuals work together and do their part. Poor countries have pledged, on their part, to ensure good
governance, and invest in health care and education for their people. Rich countries have pledged to support
them in their efforts, through aid, debt relief, and fairer trade.
The eight MDGs listed below guide the efforts of virtually all organizations working in development and have
been commonly accepted as a framework for measuring development progress:
Eradicate extreme poverty and hunger
Achieve universal primary education
Promote gender equality and empower women
Reduce child mortality
Improve maternal health
Combat HIV/AIDS, malaria, and other diseases
Ensure environmental sustainability
Develop a Global Partnership for Development

(Q.75) In a hospital set-up, the following symbol at the entrance of a room represents te presence of ?

(a)

Biohazardous material.

(b)

Radioactivity

(c)

Magnetic field

(d)

Medical biowaste

Your Response :

Correct Answer :

Exp:

Radioactivity
Ref : Stevens, Christine Dorresteyn. Clinical immunology and serology : a laboratory perspective / Christine
Dorresteyn Stevens. 3rd ed.
Radioactivity is encountered in the clinical laboratory when procedures using radioisotopes, such as
radioimmunoassay, are performed. The amount of radioactivity present in most medical situations is very small
and represents little danger However, the effects of radiation are related to the length of exposure and are
cumulative. Exposure to radiation is dependent on the combination of time, distance, and shielding. Persons
working in a radioactive environment are required to wear measuring devices to determine the amount of
radiation they are accumulating. Laboratorians should be familiar with the radioactive symbol shown. This symbol
must be displayed on the doors of all areas where radioactive material is present. Exposure to radiation during
pregnancy presents a danger to the fetus, and personnel who are or who think they may be pregnant should
avoid areas with this symbol.

(Q.76) Which of following is not a mode of specific protection?


(a)

Immunization

(b)

Chemoprophylaxis

(c)

Voidance of allergens

(d)

Food fortification

Your
Response c
:
Correct
Answer :

Exp:
(Q.77) World cancer day is on ?

http://dbmci.examonair.com/Result/ShowAllQuestionInHtml.aspx?testid=5615

29/111

12/25/2014

Dr Bhatia Medical Coaching Institute:Online Test Platform


(a)

Feb 4

(b)

Feb 6

(c)

April 7
May 1

(d)
Your Response :

Correct Answer :

Exp:

Feb 4
Global public health campaigns offer great potential to raise awareness and understanding about health issues
and mobilize support for action, from the local community to the international stage.There are many world days
observed throughout the year related to specific health issues or conditions from Alzheimer's to
zoonoses.However, WHO focuses particular attention on the seven days and one week that WHO Member States
have mandated as "official" global public health campaigns. These are:
World TB Day

24 March

World Health Day

7 April

World Immunization Week

last week of April

World Malaria Day

25 April

World No Tobacco Day

31 May

World Blood Donor Day

14 June

World Hepatitis Day

28 July

World AIDS Day

1 December

Other Commemorative days are


World Cancer Day International Day of Zero Tolerance to Female Genital
Mutilation National Science Day(India)
Zero discrimination day
International Women's Day
International Day for the Elimination of Racial
Discrimination
World Down Syndrome Day
World Water Day
World Tuberculosis Day
World autism awareness day
World Hemophilia Day
Earth Day
World Malaria Day
World Thallasemia Day
World Hypertension Day
World Milk Day
World Environment Day
World day against Child Labour
World Blood Donor Day
World Population Day
Nelson Mandela International Day
World Hepatitis Day
World Breastfeeding Week
International Youth Day
World Humanitarian Day
World Contraception Day
World Breast Cancer Awareness Month
World Vegetarian Day
World Mental Health Day
World Food Day
United Nations Day
World Diabetes Day
Human Rights Day

http://dbmci.examonair.com/Result/ShowAllQuestionInHtml.aspx?testid=5615

February 4
February 6
February 28
1 March
March 8
March 21
March 21 (Remember as Downs is 21 Trisomy (3rd
month) i;e 21/03)
March 22
March 24
April 2
April 17
April 22
April 25
May 8
May 17
June 1
June 5
June 12
June 14
July 11
July 18
July 28
August 01 to 07
August 12
August 19
September 26
October
October 1
October 10
October 16
October 24
November 14
December 10

30/111

12/25/2014

Dr Bhatia Medical Coaching Institute:Online Test Platform


(Q.78) A 34-year old woman is treated for UTI with amoxicillin. 5-days after beginning treatment, she develops recurrent fever, abdominal

bloating, and diarrhea with six to eight loose stools per day. You suspect antibiotic-associated colitis. False statement regarding
diagnosis of this condition is ?
(a)

Stool culture for (C) difficileis most sensitive test.

(b)

Diagnosis is based on a combination of clinical criteria

(c)

Empirical treatment is appropriate if strongly suspected on clinical grounds

(d)

None of the above

Your Response :

Correct Answer :

Exp:

None of the above


C. difficile is an important cause of diarrhea in patients who receive antibiotic therapy. (C) difficile proliferates in
the gastrointestinal tract when the normal enteric flora are altered by antibiotics. Commonly implicated
antibiotics include ampicillin, penicillin, clindamycin, cephalosporins, and trimethoprim-sulfamethoxazole. The
diarrhea is usually mild to moderate, but can occasionally be profound. Other clinical findings include pyrexia,
abdominal pain, abdominal tenderness, leukocytosis, and serum electrolyte abnormalities. The diagnosis is made
by demonstration at sigmoidoscopy of yellowish plaques (pseudomembranes) that cover the colonic mucosa or
by detection of (C) difficile toxin in the stool. The pseudomembranes consist of a tenacious fibrinopurulent
mucosal exudate that contains extruded leukocytes, mucin, and sloughed mucosa. Isolation of (C) difficile from
stool cultures is not very specific because of asymptomatic carriage, particularly in infants. Serological tests are
not clinically useful for diagnosing this infection. Pseudomembranous colitis demands discontinuation of the
offending antibiotic. Antibiotic therapy for moderate or severe disease includes oral vancomycin or
metronidazole. Cholestyramine and colestipol are also used therapeutically to bind the diarrheogenic toxin.
Diagnosis
The diagnosis of CDI is based on a combination of clinical criteria: (1) diarrhea (>3 unformed stools per 24 h for > 2
days) with no other recognized cause plus (2) toxin A or B detected in the stool, toxin-producing (C) difficile
detected in the stool by polymerase chain reaction (PCR) or culture, or pseudomembranes seen in the colon. PMC
is a more advanced form of CDI and is visualized at endoscopy in only 50% of patients with diarrhea who have a
positive stool culture and toxin assay for (C) difficile. Endoscopy is a rapid diagnostic tool in seriously ill patients
with suspected PMC and an acute abdomen, but a negative result in this examination does not rule out CDI.
Relative Sensitivity and Specificity of Diagnostic Tests for Clostridium Difficile Infection (CDI)
Relative

Relative

Comment

Stool culture for (C)


difficile

++++

+++

Most sensitive test; specificity of + + + + if the (C)


difficile isolate tests positive for toxin; with clinical
data, is diagnostic of CDI; turnaround time too slow for
practical use

Cell culture cytotoxin


test on stool

+++

++++

With clinical data, is diagnostic of CDI; highly specific


but not as sensitive as stool culture; slow turnaround
time

Enzyme immunoassay
for toxin A or toxins A
and B in stool

+ + to + + +

+++

With clinical data, is diagnostic of CDI; rapid results,


but not as sensitive as stool culture or cell culture
cytotoxin test

Type of Test

Sensitivitya

Specificitya

Enzyme immunoassay + + + to + + + + + + +
for (C) difficile common
antigen in stool

Detects glutamate dehydrogenase found in toxigenic


and nontoxigenic strains of (C) difficile and other stool
organisms; more sensitive and less specific than
enzyme immunoassay for toxins; rapid results

PCR for (C) difficile toxin + + + +


B gene in stool

++++

Detects toxigenic (C) difficile in stool; newly approved


for clinical testing, but appears to be more sensitive
than enzyme immunoassay toxin testing and at least as
specific

Colonoscopy or
sigmoidoscopy

++++

Highly specific if pseudomembranes are seen;


insensitive compared with other tests

Despite the array of tests available for (C) difficile and its toxins, no single test has high sensitivity, high specificity,
and rapid turnaround. Most laboratory tests for toxins, including enzyme immunoassays (EIAs), lack sensitivity.
However, testing of multiple additional stool specimens is not recommended. PCR assays have now been
approved for diagnostic testing and appear to be both rapid and sensitive while retaining high specificity.
Empirical treatment is appropriate if CDI is strongly suspected on clinical grounds. Testing of asymptomatic
patients is not recommended except for epidemiologic study purposes. In particular, so-called tests of cure
following treatment are not recommended because many patients continue to harbor the organism and toxin
after diarrhea has ceased and test results do not always predict recurrence of CDI. Thus these results should not
be used to restrict placement of patients in long-term-care or nursing home facilities.

(Q.79) One of the following is not a ratio:


(a) Prevalence
(b) Case fatality rate
(c) Proportional mortality rate
(d) Age - specific death rate
Your
Response b
:
Correct D
Answer :
Exp:

Age - specific death rates

http://dbmci.examonair.com/Result/ShowAllQuestionInHtml.aspx?testid=5615

31/111

12/25/2014

Dr Bhatia Medical Coaching Institute:Online Test Platform


Rates which are actually ratio
are:
- Case fatality rate
- Proportional mortality rate
- Prevalence
(Q.80) Which of the following disease have bimodal age distribution?
(a)

Hodgkin's disease

(b)

Leukemia

(c)

Female breast cancer

(d)

All of the above

Your Response :

Correct Answer :

Exp:

All of the above


Sometimes there may be two separate peaks instead of one in the age incidence curves of a disease as in the case
of Hodgkin's disease, leukaemia, and female breast cancer.

(Q.81) For evaluation of new therapies, the best method is :


(a)

Prospective cohort study

(b)

Retrospective cohort study

(c)

Randomized controlled trial

(d)

Non randomized trial

Your
Response c
:
Correct
C
Answer :
Exp:
(Q.82) Cushings disease is characterized by
(a)

Increased urinary catecholamines

(b)

Increased serum ACTH and serum cortisol

(c)

Increased serum ADH

(d)

Decreased serum ACTH and increased serum cortisol

Your Response :

Correct Answer :

Exp:

Increased serum ACTH and serum cortisol


Regardless of etiology all cases of endogenous Cushings syndrome are due to an increased production of cortisol
by the adrenal gland.
In most cases, cause is bilateral adrenal hyperplasia due to hypersecretion of pituitary ACTH or ectopic production
of ACTH by a nonpituitary source.
The incidence of pituitary-dependent adrenal hyperplasia is three times greater in women than in men.
The primary defect is a pituitary adenoma, as tumor is found in >90% of patients with pituitary-dependent
adrenal hyperplasia.
Alternatively, the defect may occasionally reside in the hypothalamus or in higher neural centers, leading to a
release of CRH inappropriate to the level of circulating cortisol.
This primary defect leads to hyperstimulation of the pituitary, resulting in hyperplasia and tumor formation.
Most individuals with hypersecrete tion of pituitary ACTH are found to have a microadenoma (<10mm) in
diameter and macroadenoma (>10 mm).
Traditionally, only an individual who has an ACTH producing pituitary tumor is defined as having Cushings
disease, where Cushings syndrome refers to all causes of excess cortisol: exogenous ACTH tumor, adrenal tumor,
pituitary ACTH-secreting tumor, or excessive glucocorticoid treatment.

(Q.83) Pseudo carriers are:


(a)

Who have contracted infection from a carrier.

(b)

Who have developed a carrier state without suffering fro in overt disease

(c)

Who shed the infectious agent for short period

(d)

Who are carriers of avirulent organisms

Your Response :

Correct Answer :

Exp:

Who are carriers of avirulent organisms.


Carriers of avirulent organisms are called pseudo carriers. They are not important epidemiologically.
Carriers may be classified as follows:
1. True carriers
A. Incubatary carriers temporary
B. Convalescent carriers temporary, chronic and
relapsing
C. Contact carriers temporary and chronic, primary and secondary.

http://dbmci.examonair.com/Result/ShowAllQuestionInHtml.aspx?testid=5615

32/111

12/25/2014

Dr Bhatia Medical Coaching Institute:Online Test Platform


2. Pseudo-carriers
3. Possible carriers
1. True carriers. The parasites are pathogenic and virulent.
Questions of relative virulence are of course involved and many of them are unsettled at present. The weight of
evidence, however, is against sudden changes of virulence and in true carriers, according to actual tests or
reasonable assumption, the organisms are of sufficient virulence to infect.
A. Incubationary or precocious carriers are infected and infective individuals in the incubation period of various
diseases. These carriers are recognised as especially dangerous in the acute exanthemata. They have also been
recognized in cerebrospinal meningitis, typhoid, cholera, and other diseases. In some carriers the germ
apparently has a "resting" period much longer than the clinical incubation period. Even with the ordinary period
of days and weeks, an apparently healthy person may in this way spread trouble in many directions.
B. Convalescent carriers are of three kinds, the temporary, chronic and relapsing.
1. Every case must pass through the temporary convalescent carrier stage and fortunately most cases terminate
in this way. There is a clean-up fight between the body cells and the parasite with complete victory for the
tissues. Along with convalescence or soon afterwards, the patient is disinfected by immune bodies and
phagocytes and does not infect his environment or immediate contacts.
2. A small percentage of convalescents, however, become chronic carriers. These form the bulk of the carrier
menace and problem. The microorganisms perpetuate themselves from carrier to case and from case to carrier.
This is the vicious circle which must be broken to win the fight against infectious diseases.
3. Relapsing carriers. A group of this kind is useful as it covers convalescent carriers who later relapse into cases.
Examples of such carriers are found especially in protozoal infections in which the immunity is not as sharp as in
bacterial infections. The point of difference between a chronic or latent case and a relapsing carrier is the slowly
progressive nature of the infection in cases and a complete balance, even if temporary in carriers. If the carrier
conception is broadened to include the infection of one organ within the individual by a carrier lesion in another
organ, cases of focal infection can be considered relapsing carriers.
C. Contact carriers or "healthy" carriers are those who acquire the parasite from association with cases or carriers
without
developing the disease themselves. Such carriers are already immune. Differential diagnosis between a contact
and incuba- tionary carrier can be made in most cases only by the outcome Contact carriers are usually
temporary, but may become chronic if there already exists some predisposing chronic focus. Thus, an immune
contact carrier with a negative Schick test and an obstructed nasal passage may carry diphtheria bacilli until the
deformity is corrected. Contact carriers may be primary, that is, infected by a case, or secondary, infected by
another carrier. The importance of pure contact carriers has been exaggerated. Many so-called contact carriers
are really convalescent carriers after mild infections, as has recently been emphasized by Craig.
1. Contact carriers might include also "mechanical" carriers such as surgeons with contaminated hands, but such
carriers as well as inanimate objects which carry infection by indirect contact are best considered under personal
hygiene and general sanitation. There is nothing distinctive about their carrier relationships.
2. Pseudo-carriers. A group should be made for carriers of non-pathogenic and non-virulent organisms. These
may be called pseudo-carriers. In any extensive carrier work suspicious organisms are found which, however, on
further examination, turn out to be of no importance. In the meantime, the patient may have been handled as a
true carrier. The diagnosis should be changed to that of pseudo-carrier. Such carriers are especially well
recognized in diphtheria.
3. Possible carriers. A group of possible carriers may well be made to account for the considerable number of
persons who carry organisms of uncertain significance. These organisms at present can neither be proved nor
disproved to be of importance, on account of the inadequate state of our knowledge. Under this heading would
come many carriers of streptococci, influenza bacilli, and various intestinal organisms. With increasing knowledge
this group will naturally disappear as the members of it are distributed among the true and psuedo-carriers.
Convalescent carriers have been called active and contact carriers, passive. These terms are valuable in
emphasizing the difference between the active focus of multiplication in convalescent carriers and the usual
absence of a home in the contact carrier, but they are somewhat misleading as all true carriers are active from
the point of view of the parasite.
(Q.84) Period of isolation recommended for a person having mumps is:
(a)

3 days after onset

(b)

Until fever subsides

(c)

Until earache subsides

(d)

Until 5 days after onset of parotitis

Your Response :
Correct Answer :
Exp:

D
Until 5 days after onset of parotitis
The scientific evidence from the CDC and AAP review indicates that, although mumps virus can be isolated from
saliva or respiratory secretions 5 or more days after parotitis onset, virus most often is isolated before or around
the time of onset, and viral load decreases rapidly during the 4 days after onset of parotitis. Therefore, the risk for
transmission after 5 days is considered low; most transmission likely occurs before onset of parotitis and within
the subsequent 5 days. Transmission also occurs from persons with subclinical infections who are not isolated. A
longer isolation period of 9 days likely would result in less compliance and more cost and not produce any
substantial decrease in mumps transmission.
Based on this review, CDC, AAP, and HICPAC now recommend a 5-day period after onset of parotitis for 1)
isolation of persons with mumps in either community or health-care settings and 2) use of standard precautions
and droplet precautions. Postexposure recommendations remain unchanged. HCP with no evidence of mumps
immunity who are exposed to patients with mumps should be excluded from duty from the 12th day after first
exposure through the 26th day after last exposure.

(Q.85) Height of the body can be deduced from the length of humerus by following equation:
(a)

Humerus is 10 percent of height

(b)

Humerus length is 50 percent of height

(c)

Humerus length is 20 percent of height

http://dbmci.examonair.com/Result/ShowAllQuestionInHtml.aspx?testid=5615

33/111

12/25/2014

Dr Bhatia Medical Coaching Institute:Online Test Platform


(d)

Humerus length is 2 percent of height

Your Response :

Correct Answer :

Exp:

Humerus length is 20 percent of height


(Height or Stature is determined in a dismembered body by:
(1) Length from the tip of middle finger to the tip of opposite middle finger when arms are fully extended.
(2) Twice the length of one arm + 30 cm for two clavicles + 4 cm for the sternum.
(3) Humerus length = 1/5th of height.
(4) The length from the vertex to the Symphysis pubis is half of the total length.
(5) The length from the sternal notch to Symphysis pubis x 3.3.
(6) The length of forearm measured from tip of middle finger is = 5/19 of total length.
(7) The height of head measured by the vertical distance from the top of the head (vertex) to the tip chin = 1/8 of
the total length.
(8) The length of Vertebral Column = 34/100 of total length. To the length of entire skeleton add 2.5 4 cm for
thickness of the soft parts.)

(Q.86) Diagnostic power of test is reflected by:


(a)

Sensitivity

(b)

Specificity

(c)

Predictive value

(d)

Yield

Your
Response d
:
Correct C
Answer :
Exp:
(Q.87) Strain used in Mumps vaccine?
(a)

Edmonsten Zagreb

(b)

Oka

(c)

Jeryl Lynn

(d)

Danish 1331

Your Response :

Correct Answer :

Exp:

Jeryl Lynn.
(Ref. Parks Textbook of PSM 20th /pg. 141)
Here is comprehensive list of all the specific strains used for vaccination :
VACCINE

STRAIN USED

BCG

Danish 1331

Measles

Edmonston-Zagreb strain
Schwatz strain
Moraten strain

Mumps

Jeryll Lynn strain


RIT 4385
Leningrad 3 strain
L-Zagren strain
Urabe strain
Rubini strain

Rubella

RA 27/3 strain
HPV 77 strain
Cendehill strain

Yellow fever

17 D strain
Asibi strain
Dakar strain

Varicella

OKA strain

Japanese encephalitis

Nakayama strain MC
Beijing P3 strain
SA 14-14-2 strain

Malaria

SPf 66 strain {Lytic cocktail}


Pf 25 strain

Brucella

Brucella abortus 19 BA strain

Typhoral

TY21a

HIV

mVA {modified vaccinai Ankara } strain


rAAV {recombinant adeno associated viral} strain
CTL {cytotoxic Tlymphocyte strain}
AIDSVAX strain
Subunit vaccine strain

http://dbmci.examonair.com/Result/ShowAllQuestionInHtml.aspx?testid=5615

34/111

12/25/2014

Dr Bhatia Medical Coaching Institute:Online Test Platform


Pnemococcal

PPV 23{Unconjugated vaccine}


PCV 7{conjugated vaccine}

Plague

Recombinant vaccine using F1 and V antigens

Anthrax

Mazzuchi/Sterne vaccine

Rickettsia

Weigls vaccine{R.prowazeki}
Castaneda formalinized mouse lung vaccine
Live attenuated strain E vaccine
Cox inactivated yolk sac vaccine

HPV

HPV-VLT vaccines
Gardasil quadrivalent {6,11,16,18}
Cervarix bivalent {16,18}

Cholera

Whole Cell killed with or without nontoxic beta


subunit
Live attenuated- Strain CVD 103 HgR.

(Q.88) Aging can be explained on the basis of ?


(a)

Genetic hypothesis

(b)

Mitochondral dysfunction

(c)

Hormonal changes

(d)

All of the above

Your Response :
Correct Answer :
Exp:

D
All of above
Some Theories of Aging

Hypothesis

How It May Work

Genetic

Aging is a genetic program activated in post-reproductive life when an


individual's evolutionary mission is accomplished

Oxidative stress

Accumulation of oxidative damage to DNA, proteins, and lipids interferes


with normal function and produces a decrease in stress responses

Mitochondrial
dysfunction

A common deletion in mitochondrial DNA with age compromises


function and alters cell metabolic processes and adaptability to
environmental change

Hormonal changes

The decline and loss of circadian rhythm in secretion of some hormones


produces a functional hormone deficiency state

Telomere shortening Aging is related to a decline in the ability of cells to replicate


Defective host
defenses

The failure of the immune system to respond to infectious agents and


the overactivity of natural immunity create vulnerability to
environmental stresses

Accumulation of
senescent cells

Renewing tissues become dysfunctional through loss of ability to renew

(Q.89) Which of the following person work at a subcentre level?


(a)

Multi-purpose health worker

(b)

Medical officer

(c)

Laboratory technician

(d)

Health educator

Your Response :

Correct Answer :

Exp:

Multi-purpose health worker


The sub-center is a peripheral outpost of existing health care delivery system in rural area, which are established
for every 5000 population in general and every 3000 population in hilly, tribal and backward areas.
Currently a subcentre is staffed by 1 Female health worker known as Auxillary Nurse Midwife (ANM) and 1 male
health worker known as Multipurpose worker Male
The Ministry of Health and Family welfare, Govt of Indial provides assistance to all subcentres since April 2002. In
the form of salary of ANMs and Lady health visitors rent and contingency.
The salary of Male health worker is borne by the state governments.
A voluntary worker is provided at the subcentre level as a helper to ANM. The ANM pays to this voluntary worker
from her contingency fund.
The voluntary worker at the subcentre level should be preferably a Trained Birth Attendant (TBA) and be paid Rs
200 / - at least (With equal contribution by the government of india and the concerned state.)
The services provided by Subcentre are known as ASSURED SERVICES under the Indian Public Health Standards.
Manpower requirement :
In order to provide proper services, each subcentre should have the following personnel :
MANPOWER
Health worker Female / ANM

http://dbmci.examonair.com/Result/ShowAllQuestionInHtml.aspx?testid=5615

EXISTING

PROPOSED

35/111

12/25/2014

Dr Bhatia Medical Coaching Institute:Online Test Platform


Health worker Male / MPW

1 (Funded and appointed by the state


govt.)

Voluntary worker to keep the


1 (optional)
subcentre clean and assist the ANM.
She is paid by the ANM from her
contingency fund @ Rs 100 / pm.

1(optional)

Total

3/4

2/3

(Q.90) Maximum saturated fatty acids are present in?


(a)

Palm oil

(b)

Ground nut oil

(c)

Corn oil
Soyabean oil

(d)
Your Response :

Correct Answer :

Exp:

Palm oil
(Ref. Park PSM 20th ed. 528; Table 2)
MAXIMUM IN

MINIMUM IN

Saturated fatty acids

Coconut oil

Sunflower / Corn oil

Mono-unsaturated FAs

Groundnut oil

Coconut oil

Poly-unsaturated FAs

Safflower oil

Coconut oil

TYPE OF FATTY ACIDS

---------------------------------------------------------------------------------------------------------------------------------------------------------Type of oil

% of Polyunsaturated fatty acid (PUFA)

---------------------------------------------------------------------------------------------------------------------------------------------------------Coconut oil

2%

Butter

3%

Margarine

50%

Palm oil

10%

Cottonseed oil

50%

Groundnut oil

31%

Sunflower seed oil


Corn oil

65%
65%

Safflower oil

75%

Soyabean oil

62%

(Q.91) The staphylococcal reference centre of India is located at ?


(a)

AIIMS, New Delhi

(b)

J.J Hospital Mumbai

(c)

MAMC, New Delhi

(d)

JALMA, Agra.

Your Response :

Correct Answer :

Exp:

MAMC, New Delhi


INSTITUTES

LOCATED AT

National environmental engineering research


institute (NEERI)

Nagpur

National institute of nutrition

Hyderabad

Indian agriculture research institute

New delhi

Central food technological research institute

Mysore

Central food laberotaries(whose report is


considered final under the PFA act 1954)

Kolkata,Mysore,Ghaziabad and Pune.

FAO/WHO Brucella reference centre,

Izatnagar

Indian veterinary research institute,


National and international centre for Brucella
Salmonella serotyping and reference centre

Kasauli

Shigella serotyping and reference centre

KGMC Lucknow

Staphylococcal reference centre

MAMC, New delhi

Salmonella phage typing

Lady Hardinge Med College New delhi

Vibrio phage typing

NICED Kolkata

National leptospiral reference centre

Port Blait

National institute of virology

Pune

HIV reference centre

JJ Hospital Mumbai and NACO New Delhi

Influenza reference centre

AFMC Pune

National TB institute

Bangluru

TB chemotherapy centre

Chetput,Chennai

http://dbmci.examonair.com/Result/ShowAllQuestionInHtml.aspx?testid=5615

36/111

12/25/2014

Dr Bhatia Medical Coaching Institute:Online Test Platform


BCG lab of India

Guindy,Tamil nadu

International reference centre for BCG quality Copenhagen


control
National institute of health and family welfare New delhi
The leprosy mission

New delhi

Central JALMA institute of leprosy

Agra

Central leprosy teaching and training institute Chingelput,Chennai


All india institute of hygiene and public health Kolkata
Central labour institute

Mumbai

Central mining research station

Dhanbad

Industrial toxicology research centre

Lucknow

Occupational health research institute

Ahmedabad

National institute for mentally handicapped

Secunderabad

Ali Yawar Jung national institute for hearing


handicapped

Mumbai

National institute for visually handicapped

Dehradun

National institute for orthopedically


handicapped

Kolkata.

(Q.92) All of the following are true about RDA except ?


(a) The RDA is the nutrient-intake goal for planning diets of individuals.
(b)

(c)

(d)

The nutrient content in a food is stated by weight or as a percentage of the daily value (DV), a variant of the RDA
that, for an adult, represents the highest RDA for an adult consuming 2000 kcal/d.
The RDA is the average daily dietary intake level that meets the nutrient requirements of nearly all healthy
persons of a specific sex, age, life stage, or physiologic condition (such as pregnancy or lactation).
The RDA is defined statistically as 1 standard deviation (SD) above the EAR to ensure that the needs of most
individuals are met.

Your Response :

Correct Answer :

Exp:

The RDA is defined statistically as 1 standard deviation (SD) above the EAR to ensure that the needs of most
individuals are met. (Ref HARRISON 17th Ed Chapter 70)
Recommended Dietary Allowances
The RDA is the nutrient-intake goal for planning diets of individuals; it is used in the MyPyramid food guide of the
U.S. Department of Agriculture (USDA), therapeutic diets, and descriptions of the nutritional content of processed
foods and dietary supplements. The nutrient content in a food is stated by weight or as a percentage of the daily
value (DV), a variant of the RDA that, for an adult, represents the highest RDA for an adult consuming 2000 kcal/d.
The RDA is the average daily dietary intake level that meets the nutrient requirements of nearly all healthy
persons of a specific sex, age, life stage, or physiologic condition (such as pregnancy or lactation).
The RDA is defined statistically as 2 standard deviations (SD) above the EAR to ensure that the needs of most
individuals are met.
The risk of dietary inadequacy increases as intake falls further below the RD(A) However, the RDA is an overly
generous criterion for evaluating nutrient adequacy. For example, by definition the RDA exceeds the actual
requirements

(Q.93) All are true about Kwashiorker except ?


(a)

Child has a well nourished appearance

(b)

Develops in few months to years

(c)

Edema is a feature

(d)

Has a high mortality.

Your Response :

Correct Answer :

Exp:

Develops in few months to years.


REF : HARRISON 17th Ed Chapter 72.
Comparison of Marasmus and Kwashiorkor
Marasmus

Kwashiorkor

Clinical setting

Energy intake

Protein intake during stress state

Time course to
develop

Months or years

Weeks

Clinical features

Starved appearance

Well-nourished appearance

Weight <80% standard for height

Easy hair pluckabilityb

Triceps skinfold <3 mm

Edema

Mid-arm muscle circumference <15


cm
Laboratory

Creatinine-height index <60%

http://dbmci.examonair.com/Result/ShowAllQuestionInHtml.aspx?testid=5615

Serum albumin <2.8 g/dL

37/111

12/25/2014

Dr Bhatia Medical Coaching Institute:Online Test Platform


findings

standard
Total iron-binding capacity <200 g/dL
Lymphocytes <1500/L
Anergy

Clinical course

Reasonably preserved responsiveness Infections


to short-term stress
Poor wound healing, decubitus
ulcers, skin breakdown

Mortality

Low unless related to underlying


disease

Diagnostic criteria Triceps skinfold <3 mm

High
Serum albumin <2.8 g/dL

Mid-arm muscle circumference <15


cm

At least one of the following:


Poor wound healing, decubitus
ulcers, or skin breakdown
Easy hair pluckabilityb
Edema

(Q.94) Obesity is defined as BMI more than:


(a)

10

(b)

20

(c)

30

(d)

40

Your Response :

Correct Answer :

Exp:

30
"Although not a direct measure of adiposity, the most widely used method to gauge obesity is Body Mass Index
(BMI), which is equal to weight/height (in kg/ m2). Other approaches for quantifying obesity include
anthropometry (skin fold thickness), densitometry (underwater weighing), CT or MRI and electrical impedance
using data from metropolitan life tables, BMIs for the midpoint of all heights and frames among both men and
women range from 19 to 26 kg/m, at a similar BMI, women have more body fat than men. Based on unequivocal
data of substantial morbidity, a BMI of 30 is most commonly used as a threshold for obesity in both men and
women".
Classification of Weight Status and Risk of Disease
BMI (kg/m2)

Obesity Class

Risk of Disease

Underweight

<18.5

Healthy weight

18.524.9

Overweight

25.029.9

Obesity

30.034.9

High

Obesity

35.039.9

II

Very high

Extreme Obesity

>40

III

Extremely high

Increased

Source: Adapted from National Institutes of Health, National Heart, Lung, and Blood Institute: Clinical Guidelines
on the Identification, Evaluation, and Treatment of Overweight and Obesity in Adults. U.S. Department of Health
and Human Services, Public Health Service, 1998.
Body Mass Index (BMI) Classification of Children and Adolescents
BMI PERCENTILE FOR AGE

WEIGHT STATUS

<5th percentile

Underweight

5th84th percentile

Normal weight

85th94th percentile

At risk for overweight

95th percentile

Overweight

(Q.95) Weil Felix Test is positive in following except?


(a)

Epidemic typhus

(b)

Endemic typhus

(c)

Scrub typhus

(d)

Rickettsial pox

Your Response :

Correct Answer :

Exp:

Rickettsial pox
Rickettsial diseases is a very frequently asked topic in exams.Here is a tell-all table to be remembered about
these diseases.
DISEASE

CAUSATIVE
ORGANISM

INSECT VECTOR

http://dbmci.examonair.com/Result/ShowAllQuestionInHtml.aspx?testid=5615

RESERVIOR

WEIL- NEIL-MOOSER
TransESCHAR MISCELLANEOUS
FELIX REACTION/TUNICA ovarian
TEST REACTION
transmission

38/111

12/25/2014

Dr Bhatia Medical Coaching Institute:Online Test Platform


Epidemic
R.prowazeki
typhus/Red
louse
disease/Gaol
fever/Brill
zinsers
disease

Human louse

Endemic
typhus /
Murine
typhus

Rat flea Xenophelus


cheopsis

R.typhi
(mooseri)

++ OX -19

--

--

Incubation
period 5 15 d.
Only rickettsial
disease showing
recrudescence.

Rodent

++ OX ++
19

--

--

Moosers bodies

Rodent

++ OX -K

++

++

Zoonotic tetrad

++

++

Tache noire
eschar.

++ OX -19
and
OX 2

++

--

Most severe
form.

Cat flea Ctenocephalus


felis

Scrub typhus R.
Trombiculid mite
/ Chigger
Tsutsugamushi Leptotrombidium
borne
deliensis India.
typhus
L.akamushi Japan
Indian tick
typhus/

Humans

Pediculus humanis
corporis/capitis.

R.conorii

Tick Rhipicephalus
Rodents,Dogs -sanguinis,Haemophysalis
leachi,Amblyomma and
Hyalloma ticks

RMSF

R.rickettsiae

Tick Dermatocentor
andersoni

Rickettsial
pox

R.akari

Mite Gamasid mite Mice


Liponyssoides sanguines

--

++

++

++

Mildest form

Q fever

Coxiella
burnetti

NIL (airborne
transmission)

--

--

--

--

No rash,
Nonarthropod
airborne
disease,reduced
CD4 : CD8 ratio.

Fever
boutonessi

Rodents

cattle

++

Only rickettsia
causing chronic
disease.
Trench
fever/ Five
day fever

Rochiamlae
Quintana

Human louse

Humans

--

--

--

--

Pediculus humanis
corporis/capitis

(Q.96) When variables are not measurable, which of the following scale is used to be represented:
(a)

Nominal

(b)

Ordinal

(c)

Interval
Ratio

(d)
Your Response :

Correct Answer :

Exp:

Nominal
The calibration used for any measurement qualitative or quantitative - is called the scale. This can be nominal,
ordinal ratio or interval.
1. Nominal scale :- Assessment of a characteristic in terms of names only. Blood group has a nominal scale since
O, A, B and AB are just names with no order or no grading among them. Remember that such names or codes do
not make them quantitative.
2. Ordinal scale :- A polytomous variable where the categories have defined order, such as severity of disease into
mild, moderate, serious and critical. The distance between mild and moderate is underfed.
3. Interval scale :- A polytomous variable where the categories have defined order, along with meaningful equal
intervals.
e.g. Body temperature in colicus / Fahrenheit scale.
4. Ratio scale :- Have the same characteristic as interval scale, but it has an absolute zero.

(Q.97) Cinacalcet is best used for which condition?


(a)

Primary hyperparathyroid

(b)

Secondary hyperparathyroid

(c)

Tertiary hyperparathyroid

(d)

Hypercalcemia in malignancy

Your
Response d
:
Correct D
Answer :
Exp:
(Q.98) Which of the following is virtually diagnostic of aortitis on chest x-ray?
(a)

Calcification in descending aorta

(b)

Calcification of ascending aorta

(c)

Calcification of pulmonary artery

(d)

Focal oligemia

Your Response :

Correct Answer :

http://dbmci.examonair.com/Result/ShowAllQuestionInHtml.aspx?testid=5615

39/111

12/25/2014

Dr Bhatia Medical Coaching Institute:Online Test Platform


Calcification of ascending aorta

Exp:

Aortitis
Aortitis cannot usually be recognized at the stage of uniform aortic dilatation, but once ascending aortic wall
calcification, the diagnosis is obvious.
The fine linear calcification of aortitis may be distinguished from the thicker irregular calcification of atheroma,
which, however, may occur secondarily following aortitis.
Atheromatous calcification usually also involves the aortic arch.
Curvilinear calcification confined to the ascending aorta and sparing the arch is usually due to aortitis.
Aortitis of the ascending aorta may cause dilatation of the valve ring, leading to aortic regurgitation.
Aortic root aneurysm
It may be confined by the pericardium or may extend to the arch.
Aortic root aneurysms may vary from being entirely invisible in the frontal and lateral view, to bulging the
superior mediastinum to the right, either forming a shadow continuous with the heart, or separate from it.
Rarely, they extend to the left where the only radiological abnormality may be a prominent pulmonary conus,
displaced to the left by the aneurysm.
(Q.99) If the probability of getting Rh +ve is 0.9 and probability of newborn being male is 0.5, then the probability of new born being a male

with Rh +ve is :
(a)

0.9

(b)

0.5

(c)

0.45
1.3

(d)
Your Response :

Correct Answer :

Exp:

0.45
PROBABILITY AND IT'S RULES Probability is the extent of belief in a phenomenon or a statement.
Addition rule :
P (A or B) = p (A) + P(B) if A and Bare mutually exclusive.
Multiplication rule :
P (A and B) = p (A) x P(B) if A and Bare independent.
e.g. Getting Rh +ve & being male both are independent factors so we apply rule of multiplication to get the
probability.

(Q.100) Which of the following statement about postmortem staining of a dead body which is lying undisturbed, is true?
(a)

Disappears in 1-2 days

(b)

Disappears in 3-4 days

(c)

Disappears in 7 days

(d)

Persists till it merges with discoloration of putrefaction

Your Response :

Correct Answer :

Exp:

Persists till it merges with discoloration of


Post Mortem Staining = Post Mortem Hypostasis= Livor mortis= Suggilations= Cadaveric lividity= Vibices=
Darkening of death- is bluish-red / purplish-red discolouration under the skin (superficial layer) on dependant
parts of body (due to capillo-venous distension), except over pressure points. It (1) Begins shortly after death as
patchy mottling not visible - 1 hr in normal persons, 1-4 hr in anemic. (2) Well developed in 4 hr and reaches
maximum 6-12 hrs. (3) Fixation of hypostasis occurs in 6 hrs. (4) It persists till it merges with changes of
decomposition.)

(Q.101) Hyperamylasemia is not seen in :


(a)

Diabetic ketoacidosis

(b)

Carcinoma oesophagus

(c)

Ruptured ectopic pregnancy

(d)

Hypertriglyceridemia induced pancreatitis

Your Response :

Correct Answer :

Exp:

Hypertriglyceridemia induced pancreatitis


Patients with hypertriglyceridemia and proven pancreatitis have spuriously low levels of S. amylase and
perhaps lipase activity.

(Q.102) Model for end stage liver disease (MELD) scoring system calculated by using the following criterias except :
(a)

S. bilirubin

(b)

S. Albumin

(c)

Prothrombin time expressed as INR

(d)

S. Creatinine

Your Response :

Correct Answer :

Exp:

S. Albumin

http://dbmci.examonair.com/Result/ShowAllQuestionInHtml.aspx?testid=5615

40/111

12/25/2014

Dr Bhatia Medical Coaching Institute:Online Test Platform


MELD includes PT expressed as INR, S. bilirubin and S. Creatinine. Recently MELD has replaced child Pugh
classification for assessment for need of liver transplantation.
(Q.103) Reissners membrane is seen in:
(a)

External ear

(b)

Middle ear

(c)

Semicircular canals

(d)

Cochlea

Your Response :

Correct Answer :

Exp:

CONFIRM
There are bony and membranous labyrinth in internal ear.
Bony Labyrinth has three parts:
Cochlea
Vestibule
Semicircular canals. CVS
Reissner's membrane(vestibular membrane,vestibular wall) is amembraneinside thecochleaof theinner ear. It
separatesscala mediafromscala vestibuli. Together with thebasilar membraneit creates a compartment in the
cochlea filled withendolymph, which is important for the function of theorgan of Corti. It primarily functions as
adiffusionbarrier, allowing nutrients to travel from theperilymphto theendolymphof themembranous labyrinth.
Histologically, the membrane is composed of two layers of flattenedepithelium, separated by abasal lamina. Its
structure suggests that its function is transport of fluid andelectrolytes.
Reissner's membrane is named after GermananatomistErnst Reissner

(Q.104) Hiatal hernias are best demonstrated by X rays in:


(a)

Prone position

(b)

Supine position

(c)

Upright position

(d)

Left lateral position

Your Response :

Correct Answer :

Exp:

Prone
Hiatal hernias are best demonstrated with the patient prone because the increased abdominal pressure
produced in this position promote displacement of the esophago-gastric junction above the diaphragm.

(Q.105) The most damaging element of duodeno-gastric reflux in GERD is :


(a)

Gastric acid

(b)

Gastric acid with pepsin

(c)

Soluble unionized bile acid

(d)

Unconjugated bile acid

Your Response :

Correct Answer :

C
Soluble unionized bile acid

Exp:

The component of duodenal juice thought to be more damaging is bile acids, bile acids should be both non polar
(un-ionized) and soluble (conjugated).
(Q.106) A middle aged female was being treated with high dose steroids for SLE. She had fever for 4 days, followed by otorrhea, deep

seated earache. The ear lobule showed blackish discoloration and bloody discharge for the last 1 day. Looking at the photograph
pick the wrong statement regarding this diagnosis.

(a)

The causative agent is Pseudomonas aeruginosa

(b)

Underlying immunosuppression is essential event

(c)

Hyperbaric oxygen therapy is useful

(d)

Antibiotic treatment for 3 weeks is sufficient

Your Response :
Correct Answer :

http://dbmci.examonair.com/Result/ShowAllQuestionInHtml.aspx?testid=5615

41/111

12/25/2014

Dr Bhatia Medical Coaching Institute:Online Test Platform


Antibiotic treatment for 3 weeks is sufficient

Exp:

Malignant external otitis (MEO) is an infection that affects the external auditory canal and temporal bone. The
causative organism is usuallyPseudomonas aeruginosa,and the disease commonly manifests in elderly patients
with diabetes. The infection begins as an external otitis that progresses into an osteomyelitis of the temporal
bone. Spread of the disease outside the external auditory canal occurs through the fissures of Santorini and the
osseocartilaginous junction
Predisposing conditions :
Diabetes (90%) or immunosuppression (illness or treatment related)
Severe, unrelenting, deep-seated otalgia
Temporal headaches
Purulent otorrhea
Possibly dysphagia, hoarseness, and/or facial nerve dysfunction
Treatment includes meticulous glucose control, aural toilet, systemic and ototopic antimicrobial therapy,
andhyperbaric oxygen therapy.
Systemic antibiotic choice: Until the development of third-generation antipseudomonal cephalosporins, longterm intravenous antibiotics using an antipseudomonal penicillin and aminoglycoside were the mainstay of
medical treatment.
Several authors have demonstrated the effectiveness of intravenous ceftazidime monotherapy in the treatment
of malignant external otitis (MEO).
Fluoroquinolones that attain high soft tissue and bone levels with oral doses were then developed.
Subsequently, several authors have demonstrated the efficacy of oral ciprofloxacin monotherapy.
Although no established treatment guidelines are available, case series and anecdotal experience suggest that
initial outpatient therapy with oral ciprofloxacin is efficacious for patients without a fluoroquinolone allergy,
cranial neuropathy, or intracranial complication and who do not require hospital admission for diabetes or pain
management.
The widespread use of fluoroquinolones for upper respiratory infections and simpler ear infections is beginning
to confound the typical clinical spectrum of malignant external otitis (MEO). Ciprofloxacin-resistantP
aeruginosahas been increasingly isolated in patients with malignant external otitis (MEO), accounting for as
many as 33% of isolates in patients who failed outpatient management in a study by Berenholz et al.[18]Most

notably in this patient population, 63% of isolates from 1998-2001 were resistant to ciprofloxacin, whereas only
15% of isolates were found to be resistant in the 10 years before this 3-year period. No increased morbidity or
mortality was found in patients with ciprofloxacin-resistantPseudomonas. Patients with resistantP
aeruginosarequire parenteral antibiotics with antipseudomonal beta-lactam antibiotics with or without an
aminoglycoside.
Duration of therapy
Symptoms and examination findings improve with appropriate therapy, but these changes do not correlate with
the length of needed therapy. Despite symptom relief, prolonged antimicrobial treatment as indicated for
osteomyelitis is still indicated.
Imaging studies are helpful in determining the adequate length of treatment for each patient.
Treatment response should be evaluated with a gallium citrate Ga 67 scan every 4-6 weeks during treatment.
Benecke recommended ending treatment 1 week after the gallium citrate Ga 67 scan findings return to normal
and confirming this with a repeat scan 1 month after the treatment is stopped. Using this protocol for 13
patients, the average duration of treatment was 8.8 weeks with a range of 4-17 weeks.
Hyperbaric oxygen therapy
This should be used only as an adjunct to antimicrobial therapy; it should not be used alone.
Hyperbaric oxygen therapy may be helpful for patients with complications, experiencing a poor response to
therapy, or with recurrent cases.

(Q.107) Traditional indications for (CBD) exploration are all except :


(a)

Palpable duct stones

(b)

Jaundice or history of jaundice or cholangitis

(c)

Abnormal L.F.T. esp. increased alkaline/ phosphatase

(d)

Single stone in Hartmann pouch

Your Response :

Correct Answer :

Exp:

Single stone in Hartmann pouch


The purpose of CBD exploration for choledocholithiasis is to detect and remove all stones within the bile duct
system. The absolute indications for CBD exploration are (1) palpable stones in the CBD, (2) jaundice with
cholangitis, and (3) a stone seen at intraoperative cholangiography . Even where local endoscopic, radiologic, or
laparoscopic expertise coexists, there are still indications for open choledochotomy:
Patients with multiple CBD stones who are not candidates for endoscopic sphincterotomy or in whom
endoscopic approaches are compromised (e.g., the presence of duodenal diverticula or after previous
gastrectomy)
Patients with gallstones and concomitant jaundice or acute suppurative cholangitis who cannot be managed by
endoscopic sphincterotomy
Patients in whom an open cholecystectomy is performed for different reasons, such as suspicion of cancer or the
presence of a biliary-enteric fistula or Mirizzi syndrome, who should have palpation of the CBD and operative
cholangiography performed; if CBD stones are seen, choledochotomy may be necessary.
Patients in whom, during laparoscopic cholecystectomy, concomitant CBD stones are found by cholangiography,
and the surgeon decides that they cannot be removed intraoperatively at laparoscopy or postoperatively by
endoscopic sphincterotomy
A dilated CBD and
Abnormal LFT, in particular, a raised alkaline phosphatase.

(Q.108)

A neutropenic patient presented with severe septicemic shock. However on 2nd day of illness the patient presented with large
painful, reddish, maculopapular lesions that became black and necrotic in couple of days. A biopsy revealed vascular invasion at
that site. Apart from Aspergillosis and Mucormycosis, which of the following bacterium may lead to such a condition ?

http://dbmci.examonair.com/Result/ShowAllQuestionInHtml.aspx?testid=5615

42/111

12/25/2014

Dr Bhatia Medical Coaching Institute:Online Test Platform

(a)

Pseudomonas

(b)

Mycoplasma

(c)

Streptococcus

(d)

Treponema

Your Response :

Correct Answer :

Exp:

Pseudomonas
Ref : HARRISON 18th Ed Chap 152
The clinical history and photograph provided are suggestive of diagnosis of Ecthyema gangrenosum.
Crude mortality rates exceeding 50% have been reported among patients with P. aeruginosa bacteremia.
Consequently, this clinical entity has been much feared, and its management has been attempted with the use of
multiple antibiotics. Recent publications report attributable mortality rates of 2844%, with the precise figure
depending on the adequacy of treatment and the seriousness of the underlying disease. In the past, the patient
with P. aeruginosa bacteremia classically was neutropenic or had a burn injury. Today, however, a minority of
such patients have bacteremic P. aeruginosa infections. Rather, P. aeruginosa bacteremia is seen most often in
patients on ICUs.
The clinical presentation of P. aeruginosa bacteremia rarely differs from that of sepsis in general. Patients are
usually febrile, but those who are most severely ill may be in shock or even hypothermic. The only point
differentiating this entity from gram-negative sepsis of other causes may be the distinctive skin lesions (ecthyma
gangrenosum) of Pseudomonas infection, which occur almost exclusively in markedly neutropenic patients and
patients with AIDS. These small or large, painful, reddish, maculopapular lesions have a geographic margin; they
are initially pink, then darken to purple, and finally become black and necrotic. Histopathologic studies indicate
that the lesions are due to vascular invasion and are teeming with bacteria. Although similar lesions may occur in
aspergillosis and mucormycosis, their presence suggests P. aeruginosa bacteremia as the most likely diagnosis.

(Q.109) Conus syndrome is characterized by all of the following EXCEPT


(a)

Sacral and coccygeal involvement

(b)

Absent Knee reflexes

(c)

Flexor plantar response

(d)

Saddle anesthesia

Your Response :

Correct Answer :

Exp:

Absent knee jerks


Lower motor neuron lesions make all the muscles go into flaccid paralysis.
All the reflexes are lost (deep tendon like knee reflex, as well as superficial reflexes like plantar reflex will be lost)
Note: Root value of Knee jerk is L-2, 3 and 4 (femoral nerve) and ankle jerk is S-1 (tibial nerve).
Babinski reflex is a superficial reflex with afferents from the outer margin of foot and the sole, including the ball
of great toe.
Sensory nerves involved are: sural L5, S1, 2; lateral plantar nerve S2, 3 and medial plantar nerve S1, 2.
Efferent component is deep peroneal (L-5) which contracts extensor hallucis longus resulting in upgoing toe.
Babinski reflex is present in UMN type of lesions.
Upper motor neuron lesions produce spastic type of paralysis.
Conus medullaris syndrome (CMS) and Cauda equina syndrome (CES) both are LMN lesions.
Conus medullary syndrome is going to damage all the spinal cord segments from Sacral-3, till coccygeal level.
It is usually secondary to an intramedullary tumor (like ependymoma) and the onset is sudden and bilateral.
Sensory loss is in saddle shaped area around the anal aperture (dermatome S-3 onwards are affected).
Bladder and bowel disturbances will be crippling (controlled by root value - S-2, 3 and 4) and so will be the
damaged sexual performance.
Knee jerks are present in the conus medullaris syndrome.
As is evident their spinal segment level is higher than the lesion and is thus unaffected.
Plantar reflex will be flexor and is normal.
Its because the plantar reflex arc is higher than the level of the lesion in CMS and remains unaffected.
Cauda equina syndrome (CES):
Spinal segments affected are L-3 to Co.
Usually result from a nerve root tumor (unilateral presentation).
Unilateral saddle shaped anesthesia is observed.
Unilateral knee reflexes are lost.
Bladder/bowel and sexual functions are not damaged severely (since the lesion is unilateral).
Plantar reflex will be lost on the affected side (superficial reflexes are lost in LMN lesions!).
. Symptoms and Signs of Conus Medullaris and Cauda Equina Syndromes
Conus Medullaris Syndrome

Cauda Equina Syndrome

Presentation

Sudden and bilateral

Gradual and unilateral

Reflexes

Knee jerks preserved but ankle jerks Both ankle and knee jerks affected

http://dbmci.examonair.com/Result/ShowAllQuestionInHtml.aspx?testid=5615

43/111

12/25/2014

Dr Bhatia Medical Coaching Institute:Online Test Platform


affected
Radicular pain

Less severe

More severe

Low back pain

More

Less

Sensory
symptoms and
signs

Numbness tends to be more


localized to perianal area;
symmetrical and bilateral; sensory
dissociation occurs

Numbness tends to be more localized to saddle area;


asymmetrical, may be unilateral; no sensory dissociation;
loss of sensation in specific dermatomes in lower
extremities with numbness and paresthesia; possible
numbness in pubic area, including glans penis or clitoris

Motor strength Typically symmetric, hyperreflexic


distal paresis of lower limbs that is
less marked; fasciculations may be
present

Asymmetric areflexic paraplegia that is more marked;


fasciculations rare; atrophy more common

Impotence

Frequent

Less frequent; erectile dysfunction that includes inability


to have erection, inability to maintain erection, lack of
sensation in pubic area (including glans penis or clitoris),
and inability to ejaculate

Sphincter
dysfunction

Urinary retention and atonic anal


Urinary retention; tends to present late in course of
sphincter cause overflow urinary
disease
incontinence and fecal incontinence;
tend to present early in course of
disease

(Q.110) In osteomalacia, all are true except:


(a)

Increased serum alkaline phosphatase

(b)

Increased serum calcium

(c)

Looser zones

(d)

Proximal myopathy

Your Response :

Correct Answer :
Exp:

Increased serum calcium


The hypocalcemia and hypophosphatemia that accompany vitamin D deficiency result in impaired mineralization
of bone matrix, a condition known as osteomalacia. Osteomalacia is also a feature of longstanding
hypophosphatemia, which may be a consequence of renal phosphate wasting or chronic use of etidronate or
phosphate-binding antacids. This hypomineralized matrix is biomechanically inferior to normal bone and, as a
result, patients with vitamin D deficiency are prone to bowing of weight-bearing extremities because of
abnormal remodeling and to skeletal fractures. Vitamin D and calcium supplementation have been shown to
decrease the incidence of hip fracture among ambulatory nursing home residents in France, suggesting that
undermineralization of bone contributes significantly to morbidity in the elderly. Proximal myopathy is a striking
feature of severe vitamin D deficiency, both in children and in adults. Rapid resolution of the myopathy is
observed after vitamin D repletion. )

(Q.111) Comparison of the value obtained and a predetermined objective is done by:
(a)

Evaluation

(b)

Monitoring

(c)

Input - output analysis

(d)

Network analysis

Your Response :

Correct Answer :

Exp:

Evaluation
According to WHO expert committee on National Health planning in Developed countries evaluation measures
the degree to which objectives and targets are fulfilled and the quality of the results obtained.
The purpose of evaluation is to assess the achievements of the stated objectives.
Monitoring is confined to day to day activity mostly concerned with the final outcome.
Network analysis is a graphic plan of all events and activities to be completed in order to reach an end objective.
Input-output analysis is an economic technique
Input is resources in terms of manpower, money, material and time and output is like cases treated, lives saved.
An input-output table shows how much of each input is needed to produce an unit amount of output.

(Q.112) For prevention of Neonatal tetanus, all of the following are effective except:
(a)

Give 2 doses of TT to pregnant women

(b)

Immunize all woman of reproductive age group with 2 doses of TT

(c)

Practice 5 cleans

(d)

Give penicillin to all newborns

Your Response :

Correct Answer :

Exp:

Give penicillin to all newborns


"In Area where incidence of neonatal tetanus is high, the primary 2 doses course can be extended to all women
of reproductive age, particularly if the percent coverage of antenatal case is low"
Strategy for neonatal tetanus elimination.
Increase and sustain high coverage levels with two doses or in booster dose of TT in pregnant woman.
Increase proportion of deliveries by trained personnel; intensify dais training.

http://dbmci.examonair.com/Result/ShowAllQuestionInHtml.aspx?testid=5615

44/111

12/25/2014

Dr Bhatia Medical Coaching Institute:Online Test Platform


Supply disposable delivery kits to ensure clean practices for domiciliary deliveries.
Implement essential new born care, including cord care, to reduce risks of neonatal tetanus.
Strengthen surveillance system and undertake follow up action in areas from where cases are reported.
Continue IEC activities to promote clean deliveries.
Five Clean Practices
Clean surface for delivery
Clean hands of the attendant
Clean blade for cutting the cord
Clean cord tie
No applicants on the cut stump of the cord (clean cord)
(Q.113) Randomization in a clinical trial is done manly to:
(a)

Help ensure the studies subjective are representative of general population

(b)

Reduce selection bias in allocation of treatment

(c)

Ensure group are comparable on base line characteristic

(d)

Facilitate double blinding

Your Response :

Correct Answer :

Exp:

Reduce selection bias in allocation of treatment


Randomization is a statistical procedure by which the participants are allocated into groups.
It is an attempts to eliminate 'bias' and allow for comparability.
Randomization is the "least" of a control trial. It will give the greatest confidence that the groups are comparable
so that like can be compared with like. It ensures that the investigator has no control over allocation of
participants to either study or control group, thus eliminating what is known as selection bias.

(Q.114) Main action of Ivabradine is ?


(a)

ACEI

(b)

Bradycardia

(c)

Diuretic

(d)

Calcium channel blocker

Your Response :

Correct Answer :

Exp:

Bradycardia
Ivabradine,
It is called as bradycardic drugs, relatively selective If sodium channel blockers, reduce cardiac rate by
inhibiting the hyperpolarization-activated sodium channel in the sinoatrial node.
No other significant hemodynamic effects occurs.
Ivabradine appears to reduce anginal attacks with an efficacy similar to that of calcium channel blockers
and beta blockers.
The lack of effect on gastrointestinal and bronchial smooth muscle is an advantage of ivabradine.
Ivabradine acts by reducing the heart ratein a mechanism different frombeta blockersandcalcium channel
blockers, two commonly prescribedantianginal drugs.
It is classified as a cardiotonic agent

Mechanism of action
Ivabradine acts on the Ifion current, which is highly expressed in thesinoatrial node.
It is a mixed Na+K+inward current activated by hyperpolarization and modulated by the autonomic
nervous system.
It is one of the most important ionic currents for regulating pacemaker activity in the sinoatrial (SA) node.
Ivabradine selectively inhibits the pacemaker current in a dose-dependent manner.
Blocking this channel reducescardiac pacemakeractivity, slowing theheart rateand allowing more time for
blood to flow to the myocardium.

Adverse effects
Luminous Phenomena(by patients described as sensations of enhanced brightness in a fully maintained
visual field). This is due to blockage of Ihion channels in theretinawhich are very similar to cardiac If.
These symptoms are mild, transient, fully reversible and non-severe.
Headaches.
First-degreeAV block
Ventricular Extrasystoles
Dizziness and/or blurred vision.
(Q.115) The deepest and the commonest site of subphrenic abscess is :
(a)

Right subphrenic space

(b)

Right subhepatic space

(c)

Left subphrenic space

(d)

Left subhepatic space

http://dbmci.examonair.com/Result/ShowAllQuestionInHtml.aspx?testid=5615

45/111

12/25/2014

Dr Bhatia Medical Coaching Institute:Online Test Platform


Your Response :

Correct Answer :

Exp:

Right subhepatic space


The complicated arrangement of the peritoneum results in the formation of 4 intraperitoneal and 3
extraperitoneal spaces in which pus may collect. 4 intraperitoneal spaces are - right subphrenic, right subhepatic,
left subphrenic and left subhepatic spaces.
3 extraperitoneal spaces are - right and left paracolic and pelvic spaces.
Right sub hepatic space
This is the deepest space of the four spaces
Commonest site of subphrenic abscess
It lies transuersely beneath right lobe of liver in rutherford morisons pouch
Common cause of an abscess of right sub hepatic space
Appendicitis
Cholecystitis
Perforated duodenal ulcer
Following upper abdominal surgery

(Q.116) Schwartz sign is found in:


(a)

Menires disease

(b)

Ototoxicity

(c)

Otospongiosis

(d)

Acoustic neuroma

Your Response :

Correct Answer :

Exp:

Otospongiosis
(Otospongiosis is a primary disease of the bony labyrinth)
Painless deafness (conductive)
Paracusis willisii (better hearing in noise)
Tinnitus, vertigo
Schwartze sign ( Flemingo's flush sign) (Reddish hue on promontory through tympanic membrane)
Negative Rinne test (Bc > AC).
In about 10% of cases of otosclerosis, there is a redness of the promontory of the cochlea seen through the
tympanic membrane due to prominent vascularity associated with an otospongiotic focus

(Q.117) A 45 year old patient presented with bloody nasal discharge and recurrent upper respiratory symptoms. He suddenly developed

palpable purpuric lesions and hemoptysis following which a CT Thorax was performed.
It revealed multiple cavitatory lesions as displayed here. The most likely diagnosis is ?

(a)

Wegeners granulomatosis

(b)

Kawasaki disease

(c)

Tuberculosis

(d)

Cavitatory metastasis

Your Response :

Correct Answer :

Exp:

Wegeners granulomatosis
REF : HARRISON 18th Ed Chap 326
Granulomatosis with polyangiitis (Wegener's) is an uncommon disease with an estimated prevalence of 3 per
100,000. It is extremely rare in blacks compared with whites; the male-to-female ratio is 1:1. The disease can be
seen at any age; 15% of patients are <19 years of age, but only rarely does the disease occur before adolescence;
the mean age of onset is 40 years.
Involvement of the upper airways occurs in 95% of patients with granulomatosis with polyangiitis (Wegener's).
Patients often present with severe upper respiratory tract findings such as paranasal sinus pain and drainage and
purulent or bloody nasal discharge, with or without nasal mucosal ulceration . Nasal septal perforation may
follow, leading to saddle nose deformity. Serous otitis media may occur as a result of eustachian tube blockage.
Subglottic tracheal stenosis resulting from active disease or scarring occurs in 16% of patients and may result in
severe airway obstruction. Pulmonary involvement may be manifested as asymptomatic infiltrates or may be
clinically expressed as cough, hemoptysis, dyspnea, and chest discomfort. It is present in 8590% of patients.
Endobronchial disease, either in its active form or as a result of fibrous scarring, may lead to obstruction with
atelectasis. Eye involvement (52% of patients) may range from a mild conjunctivitis to dacryocystitis, episcleritis,

http://dbmci.examonair.com/Result/ShowAllQuestionInHtml.aspx?testid=5615

46/111

12/25/2014

Dr Bhatia Medical Coaching Institute:Online Test Platform


scleritis, granulomatous sclerouveitis, ciliary vessel vasculitis, and retroorbital mass lesions leading to proptosis.
Skin lesions (46% of patients) appear as papules, vesicles, palpable purpura, ulcers, or subcutaneous nodules;
biopsy reveals vasculitis, granuloma, or both. Cardiac involvement (8% of patients) manifests as pericarditis,
coronary vasculitis, or, rarely, cardiomyopathy. Nervous system manifestations (23% of patients) include cranial
neuritis, mononeuritis multiplex, or, rarely, cerebral vasculitis and/or granuloma. Renal disease (77% of patients)
generally dominates the clinical picture and, if left untreated, accounts directly or indirectly for most of the
mortality rate in this disease. Although it may smolder in some cases as a mild glomerulitis with proteinuria,
hematuria, and red blood cell casts, it is clear that once clinically detectable renal functional impairment occurs,
rapidly progressive renal failure usually ensues unless appropriate treatment is instituted. While the disease is
active, most patients have nonspecific symptoms and signs such as malaise, weakness, arthralgias, anorexia, and
weight loss. Fever may indicate activity of the underlying disease but more often reflects secondary infection,
usually of the upper airway. Characteristic laboratory findings include a markedly elevated erythrocyte
sedimentation rate (ESR), mild anemia and leukocytosis, mild hypergammaglobulinemia (particularly of the IgA
class), and mildly elevated rheumatoid factor. Thrombocytosis may be seen as an acute-phase reactant.
Approximately 90% of patients with active granulomatosis with polyangiitis (Wegener's) have a positive
antiproteinase-3 ANC(A) However, in the absence of active disease, the sensitivity drops to 6070%. A small
percentage of patients with granulomatosis with polyangiitis (Wegener's) may have antimyeloperoxidase rather
than antiproteinase-3 antibodies, and up to 20% may lack ANC(A) Patients with granulomatosis with polyangiitis
(Wegener's) have been found to have an increased incidence of venous thrombotic events. Although routine
anticoagulation for all patients is not recommended, a heightened awareness for any clinical features suggestive
of deep venous thrombosis or pulmonary emboli is warranted.
(Q.118) Ogilvie's Syndrome is :
(a)

Acquired mega colon

(b)

Colonic pseudo obstruction

(c)

Colonic diverticulosis

(d)

Sigmoid volvulus

Your Response :

Correct Answer :

Exp:

Colonic pseudo obstruction


Acute colonic pseudo-obstruction (ACPO), also known as Ogilvie's syndrome, is a condition characterized by
massive colonic distension in the absence of mechanical obstruction. Patients presenting with Ogilvie's
syndrome have underlying medical and surgical conditions predisposing them to the syndrome. Ogilvie's
syndrome can often be managed by conservative therapy. However, unrecognized and untreated, the continued
distension associated with Ogilvie's syndrome can lead to perforation that is associated with a high mortality
rate

(Q.119) An elderly patient presented with Cerebellar signs, ipsilateral medial rectus palsy with a fixed dilated pupil and contralateral limb

ataxia (typically affecting only the arm). The area of infarction is shown in the adjoining diagram. Based on location of the infarct
the diagnosis is ?

(a)

Wallenberg syndrome

(b)

Benedickt syndrome

(c)

Parinaud syndrome

(d)

Two and a half syndrome

Your Response :

Correct Answer :

Exp:

Benedikt syndrome
REF : Clinical Neurology, 7th Edition, SIMON, ROGER P.. Chap 3
Paramedian Midbrain Infarction
Paramedian midbrain infarction caused by occlusion of the paramedian penetrating branches of the basilar
artery affects the third nerve root fibers and red nucleus. The resulting clinical picture (Benedikt syndrome)
consists of ipsilateral medial rectus palsy with a fixed dilated pupil and contralateral limb ataxia (typically
affecting only the arm). Cerebellar signs result from involvement of the red nucleus, which receives a crossed
projection from the cerebellum in the ascending limb of the superior cerebellar peduncle.

(Q.120) A patient of pneumonia develops septic shock with hypotension. The drug of choice is
(a)

Adrenaline

(b)

Noradrenaline

(c)

Phenylephrine

(d)

Ephedrine

Your Response :

Correct Answer :

http://dbmci.examonair.com/Result/ShowAllQuestionInHtml.aspx?testid=5615

47/111

12/25/2014

Dr Bhatia Medical Coaching Institute:Online Test Platform


Exp:

Noradrenaline
"Highest priority is given to aggressive volume expansion and restoration of arterial oxygenation. Former is done
with crystalloid solution to PCWP mm Hg and later is done with inspired oxygen and mechanical ventilation.
If augmentation of cardiac output is required it is done with inotrope like dopamine, nor epinephrine or
vasopressin if there is hypotension or with dobutamine if arterial pressure is normal. Surgical debridement and
antibiotics are given as required.
Since the patient in this case has hypotension and dopamine and vasopressin are not given in the choice, hence
the best answer is nor adrenaline.

(Q.121) The most effective intravesical therapy for superficial bladder cancer is :
(a)

BCG

(b)

Thiopeta

(c)

Mitomycinc
Doxorubicin

(d)
Your Response :

Correct Answer :

Exp:

BCG
Immunotherapy by intravesicular delivery of Bacillus CalmetteGurin (BCG) is also used to treat and prevent the
recurrence of superficial tumors. BCG is a vaccine against tuberculosis that is prepared from attenuated
(weakened) live bovine tuberculosis bacillus,Mycobacterium bovis, that has lost its virulence in humAns: BCG
immunotherapy is effective in up to 2/3 of the cases at this stage, and in randomized trials has been shown to be
superior to standard chemotherapy. The mechanism by which BCG prevents recurrence is unknown, but the
presence of bacteria in the bladder may trigger a localized immune reaction which clears residual cancer cells.
Instillations of chemotherapy, such as valrubicin (Valstar) into the bladder, can also be used to treat BCGrefractory CIS disease when cystectomy is not an option. The drug Urocidin is currently in Phase III of trials for
this procedure.

(Q.122) PAIR stands for:


(a)

Percutaneous angiography and imaging radiology

(b)

Per abdominal intense radiotherapy

(c)

Percutaneous aspiration, infusion and re-aspiration

(d)

Percutaneous air infused radiography

Your Response :

Correct Answer :

Exp:

Percutaneous aspiration, infusion and reaspiration


PAIR - Percutaneous aspiration of hydatid cyst, infusion of scolicidal drugs and then reaspiration is used as a
minimally invasive procedure for treatment of hydatid cyst.

(Q.123) Splenunculi can be found in all of the following areas except :


(a)

Splenic hilum

(b)

Posterior abdominal wall

(c)

Tail of pancreas

(d)

Left spermatic cord

Your Response :

Correct Answer :

Exp:

Posterior abdominal wall


Splenunculi (accessory spleen) may be found in splenic hilum and to vascular pedicle (80% cases), gastrocolic
ligament, tail of pancreas, greater omentum, greater curvature of stomach, splenocolic ligament small and large
bowel mesentery, left broad ligament in women and left spermatic cord in men.

(Q.124) BRCA-1 associated breast cancers have following distinguishing features; except :
(a)

Early age of onset

(b)

A higher prevalence of bilateral breast

(c)

Mostly hormone receptor-positive

(d)

Presence of associated cancers in some affected individuals, specially ovarian

Your Response :

Correct Answer :

Exp:

Mostly hormone receptor-positive


In general, BRCA-1 associated breast cancers are invasive ductal carcinomas poorly differentiated and are
hormone receptor negative.
BRCA-l associated breast cancers have a number of distinguishing clinical features, such as an early age of onset
when compared with sporadic cases; a higher prevalence of bilateral breast cancer; and the presence of
associated cancer in some affected individuals, specially ovarian cancer.

(Q.125) Lumbar sympathectomy is of principal value in the management of :


(a)

Gangrene of the toes or foot

(b)

Delay the requirement for amputation

http://dbmci.examonair.com/Result/ShowAllQuestionInHtml.aspx?testid=5615

48/111

12/25/2014

Dr Bhatia Medical Coaching Institute:Online Test Platform


(c)

Causalgia

(d)

Intermittent claudication

Your Response :

Correct Answer :

Exp:

Causalgia
Lumbar - sympathectomy is seldom indicated as the only treatment for patients with occlusion of major arteries
in the lower extremities.
Sympathectomy is ineffective in the management of gangrene of the toes or foot and d~ not lower the required
level of amputation or delay the requirement for amputation.
Sympathectomy does not improve claudication
May improve ischemic rest pain
Does not improve long--term patency of peripheral vascular term patency bypass grafts
Subjective and objective preoperative assessment of response to sympathetic blockade greatly enhances
response to sympathetic blockade
Three main indications: Three main indications:
Causalgia
Inoperative arterial occlusive disease with limb threatening ischemia causing rest pain, limited ulceration, or
superficial ischemia causing rest pain
digital gangrene
Symptomatic vasospastic disorders.

(Q.126) This neonate had a soft sac like structure protruding from his lower back. The antenatal ultrasound had mentioned a small sized

posterior fossa and dilated ventricles in the brain. In view of the above findings which of the following options best describes the
complete diagnosis in this case ?

(a)

Meningo-myelocele

(b)

Dandy-Walker syndrome

(c)

Arnold Chairi Type I

(d)

Arnold Chiari Type I

Your Response :

Correct Answer :

Exp:

Arnold Chiari Type II


Ref : Nelson 18th Ed Chap 592
Though the image is that of a meningo-myelocele, the complete diagnosis in this case would be a Chiari II
malformation.
Chiari II malformationis a relatively common congenital malformation of the spine and posterior fossa
characterised by lumbosacral spina bifida aperta/myelomeningocoele and a small posterior fossa with descent of
the brain stem. Numerous associated abnormalities are also frequently encountered.
Classical signs described on ultrasound include:
lemon sign
banana cerebellum sign
There may also be evidence offetal ventriculomegalydue to obstructive effects as a result of downward
cerebellar herniation. Additionally many of the associated malformations (e.g.corpus callosal dysgenesis) may be
identified.
MRI
MRI is the modality of choice for detecting and characterising the full constellation of findings associated with
Chiari II malformations. The key features are discussed below, whereas the wide range of
associated abnormalities (see above) are discussed separately.
Posterior fossa
small posterior fossa with a low attachment of the tentorium and low torcula
the brainstem appears 'pulled' down with an elongated and low lying fourth ventricle
the tectal plate appears beaked:inferior colliculusis elongated and points posteriorly, with resulting angulation of
the aqueduct which results inaqueductal stenosisandhydrocephalus
cerebellar tonsils and vermis are displaced inferiorly through foramen magnum which appears crowded
Spine
spina bifida aperta/myelomeningocoele
tethered cord

http://dbmci.examonair.com/Result/ShowAllQuestionInHtml.aspx?testid=5615

49/111

12/25/2014

Dr Bhatia Medical Coaching Institute:Online Test Platform

(Q.127) A child presented characteristic multiple pigmented macules at various sites over his body, along with nodules on his iris. There was

history of impaired vision in his right eye and some freckling in the axillae. What do think these syndromic findings suggest ?

(a)

Tuberous sclerosis

(b)

Neurofibromatosis

(c)

Von Hippel Landau syndrome

(d)

Sturge Weber syndrome

Your Response :

Correct Answer :

Exp:

Neurofibromatosis
Ref : Nelson 18th Ed Chap 596
The photograph demonstrates Lisch nodules and Caf-au-lait macules. The axillary freckling and vision
impairment that could be due to a Optic nerve glioma all point to the diagnosis of Neurofibromatosis Type I.
Neurofibromatosis (NF), von Recklinghausen disease, is a common autosomal dominant disorder. The condition
is protean, because virtually every system and organ may be affected, and progressive because distinctive
features may be present at birth, but the development of complications is delayed for decades. NF is the
consequence of an abnormality of neural crest differentiation and migration during the early stages of
embryogenesis (see also Chapter 651 ).
Clinical manifestations and diagnosis.
There are two distinct forms of NF. NF-1 is the most prevalent type, with an incidence of 1/4,000, and is
diagnosed when any two of the following seven signs are present: (1) six or more cafau-lait macules over 5 mm
in greatest diameter in prepubertal individuals and over 15 mm in greatest diameter in postpubertal individuals.
Caf au-lait spots are the hallmark of neurofibromatosis and are present in almost 100% of patients. They are
present at birth but increase in size, number, and pigmentation, especially during the 1st few years of life. The
spots are scattered over the body surface, with predilection for the trunk and extremities, and with sparing of
the face. (2) Axillary or inguinal freckling consisting of multiple hyperpigmented areas 23 mm in diameter. (3)
Two or more iris Lisch nodules. Lisch nodules are hamartomas located within the iris and are best identified by a
slit-lamp examination. They are present in >74% of patients with NF-1 but are not a component of NF-2. The
prevalence of Lisch nodules increases with age, from only 5% of children <3 yr of age, to 42% among children 34
yr of age, and virtually 100% of adults 21 yr of age. (4) Two or more neurofibromas or one plexiform
neurofibroma. Neurofibromas typically involve the skin, but they may be situated along peripheral nerves and
blood vessels and within viscera including the gastrointestinal tract. These lesions appear characteristically
during adolescence or pregnancy, suggesting a hormonal influence. They are usually small, rubbery lesions with a
slight purplish discoloration of the overlying skin. Plexiform neurofibromas are usually evident at birth and result
from diffuse thickening of nerve trunks that are frequently located in the orbital or temporal region of the face.
The skin overlying a plexiform neurofibroma may be hyperpigmented to a greater degree than a cafau-lait spot.
Plexiform neurofibromas may produce overgrowth of an extremity and a deformity of the corresponding bone.
(5) A distinctive osseous lesion such as sphenoid dysplasia (which may cause pulsating exophthalmos) or cortical
thinning of long bones with or without pseudoarthrosis. Scoliosis is the most common orthopedic manifestation
of NF-1, although it is not specific enough to be included as a diagnostic criterion. (6) Optic gliomas are present
in 15% of patients with NF-1. These relatively benign tumors consist of glial cells and a mucinous material. Most
patients with optic gliomas are asymptomatic and have normal or near-normal vision, but 20% have visual
disturbances or evidence of precocious sexual development secondary to tumor invasion of the hypothalamus.
Symptomatic optic nerve tumors typically produce symptoms before 6 yr of age. Children are rarely aware of
unilateral visual loss; thus, diagnosis may be delayed. Patients with a unilateral optic glioma typically display an
afferent pupillary defect. To test for this, each eye is alternately stimulated by a bright light source (swinging
flashlight test). The affected pupil dilates rather than constricts, whereas light in the unaffected eye causes both
pupils to constrict equally. NF-1 and a plexiform neuroma of the eyelid have a high association with an ipsilateral
optic glioma. The MRI findings of an optic glioma include diffuse thickening, localized enlargement, or a distinct
focal mass originating from the optic nerve or chiasm. (7) A first-degree relative with NF-1 whose diagnosis was
based on the aforementioned criteria. The majority of mutations in NF-1 occur in the paternal germ line. The NF1 gene on chromosome region 17q11.2 encodes all mRNA of 1113 kb containing at least 59 exons that produce
a protein neurofibromin. More than 300 independent mutations have been reported in the NF-1 gene.

(Q.128) The following statements about thyroglossal cyst are true, except :
(a)

Frequent cause of anterior midline neck masses in the first decade of life -

(b)

The cyst is located within 2 cm of the midline

(c)

Incision and drainage is the treatment of choice

(d)

The swelling moves upwards on protrusion of tongue.

Your Response :

Correct Answer :

Exp:

Incision and drainage is the treatment of choice


THYROGLOSSAL DUCT

http://dbmci.examonair.com/Result/ShowAllQuestionInHtml.aspx?testid=5615

50/111

12/25/2014

Dr Bhatia Medical Coaching Institute:Online Test Platform


PATHOLOGY:
The thyroid gland buds off the foregut diverticulum at the base of the tongue in the region of the future foramen
cecum at 3 weeks of embryonic life. Residual thyroid tissue left behind in the migration may persist and
subsequently present in the midline of the neck as a thyroglossal duct cyst.
The mass is most commonly appreciated in the 2 to 4 years old child when the baby fat disappear and
irregularities in the neck become more readily apparent.
Usually the cyst is encountered in the midline at or below the level of the hyoid bone, and moves up and down
with swallowing or with protrusion of the tongue.
CLINICAL MANIFESTATIONS :
Most thyroglossal duct cysts are asymptomatic.
If the duct retains its connection with the pharynx, infection may occur, and the resulting abscess will necessitate
incision and drainage, occasionally resulting in a salivary fistula. Submental lymphadenopathy and midline
dermoid cyst can be confused with a thyroglossal duct cyst.
The presence of malignancy in a thyroglossal cyst should be suspected when the cyst grows rapidly, or when the
ultrasound demonstrates a complex anechoic pattern or the presence of calcification.
TREATMENT:
If the cyst presents with an abscess, treatment should consist of drainage and antibiotics. Following resolution of
the inflammation resection of the cyst in continuity with the central portion of the hyoid bone and the tract
connecting to the pharynx, in addition to ligation at the foramen cecum (the Sistrunk operation) is curative.
(Q.129) Marker of acute HBV infection is:
(a)

HBV DNA polymerase

(b)

IgG anti HBc antibody

(c)

HBc antigen

(d)

HBs antigen

Your Response :

Correct Answer :

Exp:

HBV DNA polymerase


Recent and remote HBV infections can be distinguished by determination of the immunoglobulin class of antiHBc. Ant-HBc of the IgM class (IgM anti-HBc) predominates during the first 6 months after acute infection,
whereas IgG anti-HBc is the predominant class of anti-HBc beyond 6 months. Therefore, patients with current or
recent acute hepatitis B, including those in the anti-HBc window, have IgM anti-HBc in the serum. In patients
who have recovered from hepatitis B in the remote past as well as those with chronic HBV infection, anti-HBc is
predominantly of the IgG class. The other readily detectable serologic marker of HBV infection HBeAg, appears
concurrently with or shortly after HBsAg. Its appearance coincides temporally with high levels of virus replication
and reflects the presence of circulating intact virions and detectable HBV DN(A)

(Q.130) Proper treatment for frostbite consists of


(a)

Debridement of the affected part followed by silver sulfadiazine dressings

(b)

Administration of corticosteroids & vasodilators

(c)

Immersion of the affected part in water at 4044C (104111.2F)

(d)

Rewarming of the affected part at room temperature

Your Response :

Correct Answer :

Exp:

Immersion of the affected part in water at 4044C

(Greenfield, 2/e, pp 412414.)

Many methods of treating frostbite have been tried throughout the years. These include massage, warm-water
immersion, or covering the affected area. Rapid warming by immersion in water slightly above normal body
temperature (4044C) is the most effective method; however, because the frostbitten region is numb and
especially vulnerable, it should be protected from trauma or excessive heat during treatment. Further treatment
may include elevation to minimize edema, administration of antibiotics and tetanus toxoid, and debridement of
necrotic skin as needed.
REF HARRISON 17th Ed Chapter 20.
Frostbite: Treatment
Frozen tissue should be rapidly and completely thawed by immersion in circulating water at 3740(C) Rapid
rewarming often produces an initial hyperemia. The early formation of large clear distal blebs is more favorable
than smaller proximal dark hemorrhagic blebs. A common error is the premature termination of thawing, since
the reestablishment of perfusion is intensely painful. Parenteral narcotics will be necessary with deep frostbite. If
cyanosis persists after rewarming, the tissue compartment pressures should be monitored carefully.
Numerous experimental antithrombotic and vasodilatory treatment regimens have been evaluated. There is no
conclusive evidence that dextran, heparin, steroids, calcium channel blockers, hyperbaric oxygen, or
prostaglandin inhibitors salvage tissue. A treatment protocol for frostbiteis summarized in Table 20-4.
Treatment for Frostbite
During Thawing

After Thawing

Remove from
environment

Consider parenteral analgesia and


ketorolac

Gently dry and protect part; elevate;


pledgets between toes, if macerated

Prevent partial
thawing and
refreezing

Administer ibuprofen, 400 mg PO

If clear vesicles are intact, aspirate sterilely;


if broken, debride and dress with antibiotic
or sterile aloe vera ointment

Before Thawing

Stabilize core
Immerse part in 3740C (thermometer- Leave hemorrhagic vesicles intact to
temperature and treat monitored) circulating water containing an prevent dessication and infection
hypothermia
antiseptic soap until distal flush (1045
min)

http://dbmci.examonair.com/Result/ShowAllQuestionInHtml.aspx?testid=5615

51/111

12/25/2014

Dr Bhatia Medical Coaching Institute:Online Test Platform


Protect frozen part Encourage patient to gently move part
no friction or massage

Continue ibuprofen 400 mg PO (12 mg/kg


per day) q812h

Address medical or
surgical conditions

Consider tetanus and streptococcal


prophylaxis; elevate part

If pain is refractory, reduce water


temperature to 3537C and administer
parenteral narcotics

Hydrotherapy at 37C
Consider phenoxybenzamine in severe
cases

Unless infection develops, any decision regarding debridement or amputation should be deferred until there is
clear evidence of demarcation, mummification, and sloughing. Magnetic resonance angiography may
demonstrate the line of demarcation earlier than clinical demarcation. The most common symptomatic sequelae
reflect neuronal injury and the persistently abnormal sympathetic tone, including paresthesias, thermal
misperception, and hyperhidrosis. Delayed findings include nail deformities, cutaneous carcinomas, and
epiphyseal damage in children.
Management of the chilblain syndrome is usually supportive. With refractory perniosis, alternatives include
nifedipine, steroids, or limaprost, a prostaglandin E1 analogue.
(Q.131) Due to effect of a chemotherapeutic drug, the DNA of the cells were damaged and due to that p53 gene was activated. Now, the

cells will stop at which phase of the cycle?


(a)

Go Phase

(b)

G1 Phase

(c)

S Phase

(d)

G2 Phase

Your Response :

Correct Answer :

Exp:

G1 Phase
Exposure of cells to radiation or chemotherapeutic agents induces apoptosis by a mechanism that is initiated by
DNA damage and that involves the tumor suppressor gene p53, p53 accumulates when DNA is damaged and
arrests the cell cycle (at the G1 phase) to allow time for repair.

(Q.132) Mallory body is composed of:


(a)

Keratin intermediate filaments

(b)

Triglycerides

(c)

Cholesterol esters

(d)

Mucopolysaccharide

Your Response :

Correct Answer :

Exp:

Keratin intermediate filaments


Mallory body or "alcoholic hyaline" is an eosinophilic intracytoplasmic inclusion cells that is characteristic of
alcoholic liver disease.
They are most common in alcoholic hepatitis and alcoholic cirrhosis.
They are a recognized feature of Wilson's disease , primary biliary cirrhosis , non-alcoholic cirrhosis
,hepatocellular carcinoma and morbid obesity , among other conditions
Such inclusions are composed predominantly of keratin intermediate filaments.

(Q.133) The terms oncosis means:


(a)

Prelethal changes preceding necrotic cell death

(b)

Initiation of carcinogensis

(c)

Promotion of carciogenesis

(d)

Ability of chemical to induce mutations

Your Response :

Correct Answer :

Exp:

Prelethal changes preceding necrotic cell death


The term oncosis (derived from nkos, meaning swelling) was proposed in 1910 by von Reckling-hausen
precisely to mean cell death with swelling. Oncosis leads to necrosis with karyolysis and stands in contrast to
apoptosis, which leads to necrosis with karyorhexis and cell shrinkage..

(Q.134) Which of the following enzyme does not help in scavenging free radicals?
(a)

Catalase

(b)

Superoxide dismutase

(c)

Glutathione peroxidase

(d)

NADPH oxidase

Your Response :

Correct Answer :

Exp:

NADPH Oxidase
A series of enzymes acts as free radical scavenging systems and break down hydrogen peroxide and

http://dbmci.examonair.com/Result/ShowAllQuestionInHtml.aspx?testid=5615

52/111

12/25/2014

Dr Bhatia Medical Coaching Institute:Online Test Platform


superoxide anion. These are:
Catalase
Superoxide dismutase
Glutathione peroxide
While, NADPH oxidase is used in rapid bursts of superoxide production in activated polymorphonuclear
leucocytes.
(Q.135) Wiskott - Aldrich syndrome is an inherited disease of cytoskeleton. It does not consists of:
(a)

Platelet abnormalities

(b)

Immune deficiency

(c)

Eczema

(d)

Red blood cell abnormalities

Your Response :

Correct Answer :

Exp:

Red blood cell abnormalities


The Wiskott - Aldrich syndrome is an inherited disease characterized by eczema, thrombocytopenia and immune
deficiency. The protein that is mutated in this disease is involved in linking lymphocyte antigen receptors to the
cytoskeleton, and defects in the protein interfere with diverse cellular responses.

(Q.136) . All of the following cause late onset opportunistic infection in renal transplant recipients except?
(a)

Legionella

(b)

Hep B

(c)

Nocardia
Aspergillus

(d)
Your Response :

Correct Answer :

Exp:

Legionella
The Most Common Opportunistic Infections in the Renal Transplant
Recipient

Late (>6 months)

Peritransplant (<1 month)

Aspergillus

Wound infections

Nocardia

Herpesvirus

BK virus (polyoma)

Oral candidiasis

Herpes zoster

Urinary tract infection

Hepatitis B

Early (16 months)

Hepatitis C

Pneumocystis carinii
Cytomegalovirus
Legionella
Listeria
Hepatitis B
Hepatitis C

(Q.137) Following is associated with max risk of invasive cervical cancer


(a)

HPV 16 with dysplasia

(b)

Mild dysplasia

(c)

High grade squmous intraepithelial lesion(HGSIL)

(d)

HIV associated

Your Response :

Correct Answer :

Exp:

High grade squmous intraepithelial lesion(HGSIL)


High grade squamous intra epithelial lesion is the answer here. The mild dysplasia choice is out of the old
classification of cervical cytology and now the Bethesda system (i.e LGSIL, HGSIL) is used. If the choice was severe
dysplasia, it would have made same sense as HGSIL
HIV predisposes more to pre invasive cervical cancer than invasive disease.
Severity of dysplasia is directly proportional to malignancy.
HPV 18-is this most definite association with ca cervix

(Q.138) Thrifty gene are involved in the pathogenesis of which type of diabetes?

Type I

http://dbmci.examonair.com/Result/ShowAllQuestionInHtml.aspx?testid=5615

53/111

12/25/2014

Dr Bhatia Medical Coaching Institute:Online Test Platform


(a)
(b)

Type II

(c)

MODY
LADA

(d)
Your Response :

Correct Answer :
Exp:

Type II
Thrifty Genes
Thrifty genes are genes which enable individuals to efficiently collect and process food to deposit fat during
periods of food abundance.
According to the hypothesis, the 'thrifty' genotype would have been advantageous for hunter-gatherer
populations, especially child-bearing women, because it would allow them to fatten more quickly during
times of abundance.
Fatter individuals carrying the thrifty genes would thus better survive times of food scarcity. However, in
modern societies with a constant abundance of food, this genotype efficiently prepares individuals for a
famine that never comes. The result is widespread chronic obesity and related health problems like type II
diabetes.

(Q.139) Which of the following is an X -linked dominant disorder?


(a)

Tuberous Sclerosis

(b)

Diabetes insipidus

(c)

Fragile - X syndrome

(d)

Vitamin D - resistant rickets,

Your Response
d
:
Correct Answer D
:
Exp:

Vitamin D - resistant rickets.


Vitamin D - resistant rickets is an X linked dominant disorder.
Tuberous sclerosis is autosomal dominant disorder.
Diabetes insipidus and fragile - X syndrome are X - linked
recessive disorders.

(Q.140) Which of the following disorders are not due to mutations affecting collagen synthesis?
(a)

Marfan syndrome

(b)

Ehlers - Danlos syndrome.

(c)

Alport syndrome

(d)

Epidermolysis bullosa

Your Response : c
Correct Answer : A
Exp:

Marfan syndrome
Collagen:-most abundant protein in the human body
Types:
a. Type I: Bone, Skin, Tendon
b. Type II: Cartilage
c. Type III: reticulin
d.Type IV: Basement membrane
Collagen synthesis
Inside Fibroblast:
1)Synthesis (RER)
-translation of collagen -chains
2)Hydroxylation(ER)
-requires VIT C
3)Glycosylation(ER)
-form procollagen which is a triple helix of collagen chains
4)Exocytosis of procollagen into extracellular space
Outside Fibroblast:
5)Proteolytic cleavage --> becomes tropocollogen
6)Crosslinking --> becomes collagen fibrils
Disorders of Collagen synthesis
1)Ehler-Danlos
2)Osteogenesis imperfecta
3)Alports syndrome
4)Elastin
Ehler-Danlos
Abnormal type III
Faulty collagen synthesis results in

http://dbmci.examonair.com/Result/ShowAllQuestionInHtml.aspx?testid=5615

54/111

12/25/2014

Dr Bhatia Medical Coaching Institute:Online Test Platform


1)hyperextensible skin
2)easy bruising
3)hypermobile joints
Increased incidence of Berry aneurysm
Osteogenesis imperfecta
Abnormal Type I
Alports Syndrome
Abnormal Type IV
X-linked
Abnormal type IV(BM of kidney ears and eyes)
Elastin
Marfan: defect in fibrillin-1
Emphysema: 1-antitrypsin deficiency

(Q.141) In which kind of graph are points plotted over the mid-points of class intervals:
(a)

Bar diagram

(b)

O give

(c)

Frequency polygon

(d)

Histogram

Your Response :

Correct Answer :

Exp:

Frequency Polygon
A frequency distribution may also be represented diagrammatically by the frequency polygon. It is obtained by
joining the mid points of the histogram blocks.
Ogive :- It is a line diagram showing cumulative frequencies.

(Q.142) For a disease caused by several different mutations in a gene, what is the preferred method of diagnosis:
(a)

Polymerase chain reaction

(b)

Reverse transcriptase-polymerase chain reaction

(c)

Southern blot analysis

(d)

Linkage analysis

Your Response :

Correct Answer :

Exp:

Linkage analysis
For a disease caused by several different mutations in a gene, direct gene diagnosis is not feasible, and linkage
analysis remains the preferred method. Polymerase chain reaction, reverse transcriptase polymerase chain
reaction and southern blot analysis are methods of direct gene diagnosis.

(Q.143) Class I histocompatibility antigens are expressed on all except:


(a)

Macrophage

(b)

Lymphocyte

(c)

Platelets

(d)

Red Blood Cells

Your
d
Response :
Correct
D
Answer :
Exp:
(Q.144) Perforin molecules are found in:
(a)

Primary granules of neutrophils

(b)

Secondary granules of neutrophils

(c)

Cytotoxic T cells

(d)

Macrophages

Your
c
Response :
Correct
C
Answer :
Exp:

Cytotoxic T cells
Cytotoxic T cells kill their targets by two principle
mechanisms:
1. Perforin granzyme dependent killing
2. Fas - Fas ligand dependent killing

(Q.145) Most serious type of renal lesion in SLE

http://dbmci.examonair.com/Result/ShowAllQuestionInHtml.aspx?testid=5615

55/111

12/25/2014

Dr Bhatia Medical Coaching Institute:Online Test Platform


(a)

Mesangial lupus glomerulonephritis

(b)

Focal proliferative glomerulonephritis

(c)

Diffuse proliferative glomerulonephritis

(d)

Membranous proliferative glomerulo nephritis

Your Response :

Correct Answer :

Exp:

Diffuse proliferative glomerulonephritis


Diffuse proliferative glomerulonephritis is the most serious and most common of the renal lesions
in SLE.
Classification of Lupus Nephritis (International Society of Nephrology and Renal Pathology
Society)

Class I: Minimal Mesangial Lupus Nephritis


Normal glomeruli by light microscopy, but mesangial immune deposits by immunofluorescence.
Class II: Mesangial Proliferative Lupus Nephritis
Purely mesangial hypercellularity of any degree or mesangial matrix expansion by light
microscopy, with mesangial immune deposits. A few isolated subepithelial or subendothelial
deposits may be visible by immunofluorescence or electron microscopy, but not by light
microscopy.
Class III: Focal Lupus Nephritis
Active or inactive focal, segmental or global endo- or extracapillary glomerulonephritis involving
<50% of all glomeruli, typically with focal subendothelial immune deposits, with or without
mesangial alterations.
Class III (A): Active lesionsfocal proliferative lupus nephritis
Class III (A/C): Active and chronic lesionsfocal proliferative and sclerosing lupus nephritis
Class III (C): Chronic inactive lesions with glomerular scarsfocal sclerosing lupus nephritis
Class IV: Diffuse Lupus Nephritis
Active or inactive diffuse, segmental or global endo- or extracapillary glomerulonephritis
involving >50% of all glomeruli, typically with diffuse subendothelial immune deposits, with or
without mesangial alterations. This class is divided into diffuse segmental (IV-S) lupus nephritis
when >50% of the involved glomeruli have segmental lesions, and diffuse global (IV-G) lupus
nephritis when >50% of the involved glomeruli have global lesions. Segmental is defined as a
glomerular lesion that involves less than one-half of the glomerular tuft. This class includes cases
with diffuse wire loop deposits but with little or no glomerular proliferation.
Class IV-S (A): Active lesionsdiffuse segmental proliferative lupus nephritis
Class IV-G (A): Active lesionsdiffuse global proliferative lupus nephritis
Class IV-S (A/C): Active and chronic lesionsdiffuse segmental proliferative and sclerosing lupus
nephritis
Class IV-G (A/C): Active and chronic lesionsdiffuse global proliferative and sclerosing lupus
nephritis
Class IV-S (C): Chronic inactive lesions with scarsdiffuse segmental sclerosing lupus nephritis
Class IV-G (C): Chronic inactive lesions with scarsdiffuse global sclerosing lupus nephritis
Class V: Membranous Lupus Nephritis
Global or segmental subepithelial immune deposits or their morphologic sequelae by light
microscopy and by immunofluorescence or electron microscopy, with or without mesangial
alterations. Class V lupus nephritis may occur in combination with class III or IV, in which case
both will be diagnosed. Class V lupus nephritis may show advanced sclerosis.
Class VI: Advanced Sclerotic Lupus Nephritis
>90% of glomeruli globally sclerosed without residual activity.

(Q.146) Which of the following is not true about "Didactic method of communication":
(a)

Learning is authoritative

(b)

Knowledge is imposed

(c)

Human behaviour is influenced

(d)

No feedback from audience

Your Response :

Correct Answer :

Exp:

Human behaviour is influenced


Knowledge is imposed.
Learning is authoritative.
Little audience participation.
No feedback.
Does not influence human behaviour.
Adidactic method is ateaching methodthat follows a consistent scientific approach or educational style to
engage the students mind. The didactic method of instruction is often contrasted withdialecticsand theSocratic
method; the term can also be used to refer to a specific didactic method, as for instance constructivist didactics.
Didactics is a theory of teaching, and in a wider sense, a theory and practical application ofteachingandlearning.
In demarcation from "Mathetics" (the science of learning), didactics refers only to the science of teaching. This
theory might be contrasted withopen learning, also known asexperiential learning, in which people can learn by
themselves, in an unstructured manner, on topics of interest.

http://dbmci.examonair.com/Result/ShowAllQuestionInHtml.aspx?testid=5615

56/111

12/25/2014

Dr Bhatia Medical Coaching Institute:Online Test Platform


(Q.147) Major fibril protein in senile systemic amyloidosis is:
(a)

Transthyretin

(b)

Amyloid Transthyretin

(c)

-Amyloid protein

(d)

2 microglobulin

Your Response :

Correct Answer :

Exp:

Transthyretin
In senile systemic amyloidosis transthyretin is deposited in the heart of aged individuals.
Amyloid
type/Gene

Description

AL

amyloid light
chain

AL amyloidosis / multiple myeloma. Contains immunoglobulin lightchains (,) derived from plasma cells.

AA

SAA

Serum amyloid A protein (SAA) is an acute-phase reactant that is


deposited in the tissues in AA amyloidosis.

amyloid/APP

Found in Alzheimer disease brain lesions.

ATTR

transthyretin

A mutant form of a normal serum protein that is deposited in the


genetically determined familial amyloid polyneuropathies. TTR is also
deposited in the heart in senile systemic amyloidosis. Also found
in Leptomeningeal amyloidosis.

A2M

2microglobulin Not to be confused withA, 2m is a normal serum protein, part


of major histocompatibility complex (MHC) Class 1
molecules.Haemodialysis-associated amyloidosis

AIAPP

amylin

Found in the pancreas of patients with type 2 diabetes.

APrP

prion protein

In prion diseases, misfolded prion proteins deposit in tissues and


resemble amyloid proteins. Some examples are CreutzfeldtJakob
disease (humans), BSE or "mad cow disease (cattle),
and scrapie (sheep and goats). A recently described familial prion
disease presents with peripheral amyloidosis causing autonomic
neuropathy and diarrhea.

AGel

GSN

Finnish type amyloidosis

ACys

CST3

Cerebral amyloid angiopathy, Icelandic-type

AApoA1

APOA1

Familial visceral amyloidosis

AFib

FGA

Familial visceral amyloidosis

Officialabb.

(Q.148) True about TUR syndrome is ?


(a)

Immediate postoperative complication that happens in 2% of TURP patients

(b)

Excessive systemic absorption of glycine can lead to dilutional hyponatremia

(c)

Larger prostate glands can be resected with less perioperative risk.

(d)

All of above

Your Response :

Correct Answer :

Exp:

All of above
REF : Sabiston : 18th Ed. Chap 77
Transurethral Resection of Prostate
Transurethral resection of prostate (TURP) is a proven surgical technique that significantly improves lower
urinary tract symptoms associated with BPH. This intervention is most commonly recommended in the patient
with symptoms of bladder outlet obstruction and irritability that are moderate to severe, bothersome, and
interfere with the patient's quality of life. Although symptoms constitute the primary reason for recommending
intervention, in patients with an obstructing prostate, there are some absolute indications. These are acute
urinary retention, recurrent infection, recurrent hematuria, and azotemia.
Transurethral resection (TUR) syndrome is an immediate postoperative complication that happens in 2% of TURP
patients. Glycine, which is a hypotonic solution, is used during TURP. Excessive systemic absorption of glycine
can lead to dilutional hyponatremia. The symptoms associated with TUR syndrome include confusion, nausea,
vomiting, hypertension, bradycardia, and visual disturbance. Usually, the patient does not become symptomatic
until the serum sodium concentration reaches 125 mEq/dL. The risk is increased if the gland is larger than 45 g,
the resection time is longer than 90 minutes, or the irrigant fluid is greater than 70 cm H2O above the patient. All
these factors lead to greater fluid absorption and increase the risk for the syndrome. Other potential causes of
TUR syndrome include conversion of glycine to glycolic acid and ammonium. Glycolytic acid and ammonium,
which are byproducts of glycine, may be responsible for some of the side effects of confusion and metabolic
abnormalities observed in these patients.
Furosemide (Lasix) can be used to treat the hyponatremia associated with TUR syndrome. Combination of
diuretic and decreasing fluid overload gradually treats the hyponatremia over 8 to 12 hours. In severe cases,
slow infusion of 3% saline can be used to slowly correct the hyponatremia.
To avoid TUR syndrome associated with the standard TURP, newer technologies that use isotonic saline as
opposed to hypotonic solutions have emerged. Because these newer technologies use isotonic saline, systemic
fluid absorption is not associated with hyponatremia as is the case with standard TURP. Therefore, larger
prostate glands can be resected with less perioperative risk.

(Q.149) Which of the following statements regarding skull fractures is true?


(a)

Depressed fractures are those in which the patients level of consciousness is diminished or absent

http://dbmci.examonair.com/Result/ShowAllQuestionInHtml.aspx?testid=5615

57/111

12/25/2014

Dr Bhatia Medical Coaching Institute:Online Test Platform


(b)

Compound fractures are those in which the skull is fractured and the underlying brain is lacerated

(c)

Any bone fragment displaced more than 1 cm inwardly should be elevated surgically

(d)

Drainage of cerebrospinal fluid via the ear or nose requires prompt surgical treatment.

Your Response :

Correct Answer :

Exp:

Any bone fragment displaced more than 1 cm inwardly should be elevated surgically
(Schwartz, 7/e, pp 18791880.) Most skull fractures do not require surgical treatment unless they are depressed
or compound. A general rule is that all depressed skull fractures, defined as fractures in which the cranial vault is
displaced inward, should be surgically elevated, especially if they are depressed more than 1 cm, if a fragment is
over the motor strip, or if small, sharp fragments are seen on x-ray (as they may tear the underlying dura).
Compound fractures, defined as
fractures in which the bone and the overlying skin are broken, must be cleansed and debrided and the wound
must be closed. When a skull fracture occurs in an area of the paranasal sinuses, the mastoid air cells, or the
middle ear, a tear in the meninges may result in cerebrospinal fluid drainage from the ear or nose. The presence
of rhinorrhea or otorrhea requires observation and prophylactic antibiotics, because meningitis is a serious
sequel. Otorrhea usually heals within a few days. Persistent cerebrospinal fluid from the nose or ear for more
than 14 days requires surgical repair of the torn dura.

(Q.150) Which of the following is not an anti angiogenesis factor:


(a)

Thrombospondin - 1

(b)

Angiostatin

(c)

Tumstatin

(d)

HIF - 1

Your Response : c
Correct Answer : D
Exp:

HIF-l
HIF - 1, VEGF & bFGF are angiogenic factors.
Thrombospondin -1, angiostatin, endostatin, and tumstatin are anti
angiogenesis factors.

(Q.151) All of the following is true regarding wound healing in GIT except ?
(a)

pH remains constant throughout GIT

(b)

Intraluminal bulk transit and peristalsis exert distracting forces on the anastomosis

(c)

D-Penicillamine has no effect on collagen cross-linking

(d)

Dependent on intact vascular supply and neocapillary formation

Your Response :

Correct Answer :

Exp:

pH remains constant throughout GIT


REF : Schwartz 8th Ed Chapter 8
Comparison of Wound Healing in the Gastrointestinal Tract and Skin

Wound
pH
Environment

GI Tract

Skin

Varies throughout GI tract in


accordance with local exocrine
secretions

Usually constant except


during sepsis or local
infection

Microorganisms Aerobic and anaerobic, especially


in the colon and rectum;
problematic if they contaminate
the peritoneal cavity

Collagen
Synthesis

Skin commensals rarely


cause problems; infection
usually results from
exogenous contamination
or hematogenous spread

Shear stress

Intraluminal bulk transit and


Skeletal movements may
peristalsis exert distracting forces stress the suture line but
on the anastomosis
pain usually acts as a
protective mechanism
preventing excess
movement

Tissue
oxygenation

Dependent on intact vascular


Circulatory transport of
supply and neocapillary formation oxygen as well as diffusion

Cell type

Fibroblasts and smooth muscle


cells

Fibroblasts

Lathyrogens

D-Penicillamine has no effect on


collagen cross-linking

Significant inhibition of
cross-linking with
decreased wound strength

Steroids

Contradictory evidence exists


Significant decrease in
concerning their negative effect collagen accumulation
on GI healing; increased abscess
in the anastomotic line may play a
significant role

Collagenase
Activity

http://dbmci.examonair.com/Result/ShowAllQuestionInHtml.aspx?testid=5615

Increased presence throughout GI Not as significant a role in


tract after transection and
cutaneous wounds
reanastomosis; during sepsis

58/111

12/25/2014

Dr Bhatia Medical Coaching Institute:Online Test Platform


excess enzyme may promote
dehiscence by decreasing sutureholding capacity of tissue
Wound
Strength
Scar
Formation

Age

Rapid recovery to preoperative


level

Less rapid than GI tissue

Definite scarring seen in fetal


wound sites

Usually heals without scar


formation in the fetus

(Q.152) Meyers - Kouwenaar bodies are found in:


(a)

Filariasis

(b)

Malaria of

(c)

Schistosomiasis

(d)

Leprosy

Your Response :

Correct Answer :

Exp:

Filariasis
Meyers-Kouwenaar bodies, the carcasses of microfilariae (mff) covered by deposits of Splendore-Hoeppli (SH)
material, were found in small abscesses of eosinophils and in granulomas. The SH deposits varied from an
eosinophilic, hyaline fringe around intact mff to multilayered deposits surrounding an unrecognizable granular
remnant. In abscesses, peroxidase activity was intense in SH deposits and the surrounding eosinophils. The
presence and localization of IgG were variable in MK bodies, as detected by an enzyme-linked immunohistologic
assay; and antigens of mff were not detected in the SH deposits. Electron microscopy of the MK body
demonstrated a layered, radial deposition of amorphous and granular material on the mff and a structural
heterogeneity which apparently included leukocyte granules and other cell organelles. Leukocytes surrounding
MK bodies in abscesses were often degranulated and degenerate; incorporation of lysosomes of eosinophils and
cellular debris into the SH deposits at the periphery of the MK bodies was indicated.

(Q.153) Maximum permissible dose equivalent of radiations recommended by NCRP for pregnant women is:
(a)

0.1 rem

(b)

0.5 rem

(c)

1 rem
5reni

(d)
Your Response :

Correct Answer :

Exp:

0.5 rem
Maximum permissible doses
Subject exposed
1. Occupationally exposed persons
a. Whole body
b. Prospective annual limit
2. Long term accumulation to age n years
3. Pregnant woman (with respect to fetus)
4. General population
a. Occasional exposed person

Dose
5 rem/year
(n-18) x 5 rem
0.5 rem in gestation period
0.5 rem/year
0.1 rem/year
0.17 rem/year

b. Students
c. Population dose limit

(Q.154) Cobalt 60 is?


(a)

Natural radioactive source

(b)

Natural radioactive material

(c)

Artificial radioactive source

(d)

Artificial stable substance

Your Response :

Correct Answer :

Exp:

Artificial radioactive source


Cobalt-60 is artificial radioactive cobalt having a half-life of 5.3 years, used as a teletherapy source and in forms
of tubes & needles for interstitial & intracavitary therapy.
It decays, emits beta and gamma rays & has to be replaced at regular intervals of about 45 years.
It is produced from cobalt-59, which is a natural nonradioactive substance.
It decays to produce nickel, which is not radioactive substance

(Q.155) Paroxysmal Nocturnal Hemoglobinuria (PNH) is not associated with:


(a)

Deficiency of decay accelerating factor

(b)

Deficiency of membrane inhibitor of reactive lysis.

(c)

PNH is not responsible for stroke

(d)

PNH patients are at increased risk for developing acute myelogenous leukemia.

http://dbmci.examonair.com/Result/ShowAllQuestionInHtml.aspx?testid=5615

59/111

12/25/2014

Dr Bhatia Medical Coaching Institute:Online Test Platform


Your Response :

Correct Answer :

Exp:

PNH is not responsible for stroke.


Paroxysmal nocturnal hemoglobinuria(PNH), previouslyMarchiafavaMicheli syndrome, is a rare, genetically
acquired, life-threatening disease of the blood characterized by destruction of red blood cells by
the complement system, a part of the body's intrinsic immune system. This destructive process is a result of a
defect in the formation of surface proteins on the red blood cell, which normally function to inhibit such immune
reactions. Since the complement cascade attacks the red blood cells throughout the circulatory system, the
hemolysis is considered anintravascularhemolytic anemia. Other key features of the disease, notably the high
incidence of thrombosis, are not totally understood.
PNH is the only hemolytic anemia caused by anacquired(rather than inherited) intrinsic defect in the cell
membrane (deficiency ofglycophosphatidylinositol leading to absence of protective proteins on the
membrane). It may develop on its own ("primary PNH") or in the context of other bone marrow disorders such
as aplastic anemia ("secondary PNH"). Only a minority (26%) have the telltale red urine in the morning that
originally gave the condition its name.
Allogeneic bone marrow transplantation is the only curative therapy, but has significant rates of both mortality
and ongoing morbidity The monoclonal antibody eculizumab (Soliris) is effective at reducing the need for blood
transfusions, improving quality of life, and reducing the risk of thrombosis

(Q.156) CD79a is a marker for:


(a) B lymphocytes
(b) T lymphocytes
(c) Macrophage associated
(d) Activation marker.
Your
Response c
:
Correct A
Answer :
Exp:
(Q.157) Which of the following organisms is not stained by Indian ink preparation?
(a)

Treponema pallidum

(b)

Cryptococcus

(c)

Streptococcus pneumoniae

(d)

Vibrio cholera

Your Response :

Correct Answer :

Exp:

Vibrio cholera
Negative staining by Indian ink preparation is particularly useful in demonstration of bacterial capsule, very
slender organisms like spirochetes and capsulated fungi (Cryptococcus). It is not useful for vibrio cholera.

(Q.158) A patient Heeralal, 45 years old, having cough from previous 3 months, is suspected to have tuberculosis. His sputum is sent for

culture. Adding of which antiseptic will not affect growth of tubercle bacilli?
(a)

Hypochlorites

(b)

Malachite green

(c)

Chlorhexidine

(d)

Hexachlorophene

Your Response :

Correct Answer :

Exp:

Malachite green
Malachite green has no activity against tubercle bacilli but inhibits gram positive and negative organisms. Hence,
it is used as a selective agent in Lwenstein -Jensen medium.

(Q.159) Which of the following statements is incorrect regarding Stuart's medium?


(a)

It contains nutrient agar

(b)

It contains charcoal

(c)

It contains reducing agent

(d)

It is a transport medium for gonococci.

Your Response :

Correct Answer :

Exp:

It contains nutrient agar


Stuart's medium - a non - nutrient soft agar gel containing a reducing agent to prevent oxidation and charcoal to
neutralize certain, bacterial inhibitors - is a transport medium for gonococci.

(Q.160) Which culture method is used for the preparation of bacterial antigens and vaccines?
(a)

Lawn culture

(b)

Stroke culture

http://dbmci.examonair.com/Result/ShowAllQuestionInHtml.aspx?testid=5615

60/111

12/25/2014

Dr Bhatia Medical Coaching Institute:Online Test Platform


(c)

Pour plate culture

(d)

Liquid culture

Your Response :

Correct Answer :

Exp:

Lawn culture confirm


Lawn culture is employed when a large amount of growth is required on solid media as, for instance, in the
preparation of bacterial antigens and vaccines.

(Q.161) Clotting of limulus lysate is used as a test for detection of:


(a)

Histoplasma

(b)

Rickettsia prowazekii

(c)

Exotoxins

(d)

Endotoxins

Your Response :

Correct Answer :

Exp:

Endotoxins
Clotting of limulus lysate is used as a test for detection of endotoxins.
Limulus amebocyte lysate(LAL) is an aqueous extract of blood cells (amoebocytes) from thehorseshoe
crab,Limulus polyphemus. LAL reacts with bacterialendotoxinorlipopolysaccharide(LPS), which is a membrane
component ofGram negative bacteria. This reaction is the basis of theLAL test, which is the used for the
detection and quantification of bacterial endotoxins.

(Q.162) Which of the following test is not based on precipitation reaction?


(a)

Streptococci detection by Lancefield technique

(b)

VDRL test

(c)

Kahn test

(d)

Paul Bunnell test

Your Response :

Correct Answer :

Exp:

Paul Bunnell Test


Paul Bunnell test is based on agglutination reaction.
Heterophil antibody tests such as the Paul Bunnell or Monospot tests may be used in the diagnosis of glandular
fever.
Heterophil antibodies have the ability to agglutinate red blood cells of different animal species. The Paul-Bunnell
test uses sheep erythrocytes; the Monospot test, horse red cells.
In infectious mononucleosis, IgM heterophil antibodies are usually detectable for the first 3 months of infection.
Characteristically, they are able to agglutinate sheep erythrocytes; it is absorbed by ox red blood cell but not
guinea-pig kidney cells.

(Q.163) Deficiency of C5 to C8 components can lead to:


(a)

Hereditary angioneurotic edema

(b)

Systemic Lupus Erythematosus

(c)

Candidiasis

(d)

Meningococci

Your Response :

Correct Answer :

Exp:

Meningococci
Deficiency of C5 to C8 complement components can lead to bacteremia, mainly with Gram-negative
diplococci and toxoplasmosis.

COMPLEMENT SYSTEM
System of proteins that interact to play a role in humoral immunity and inflammation.
Complement defends against gram-negative bacteria.
Activated by IgG or IgM in the classic pathway.
Activated by molecules on the surface of microbes (especially endotoxin) in the alternate pathway.
C1, C2, C3, C4viral neutralization.
C3bopsonization.

http://dbmci.examonair.com/Result/ShowAllQuestionInHtml.aspx?testid=5615

61/111

12/25/2014

Dr Bhatia Medical Coaching Institute:Online Test Platform


C3a, C5aanaphylaxis.
C5aneutrophil chemotaxis.
C5b-9cytolysis by membrane attack complex (MAC).
Deficiency of C1 esterase inhibitor leads to hereditary angioedema (overactive complement).
Deficiency of C3 leads to severe, recurrent pyogenic sinus and respiratory tract infections.
Deficiency of C6C8 leads to Neisseria bacteremia.
Deficiency of decay accelerating factor (DAF) leads to paroxysmal nocturnal hemoglobinuria (PNH).
(Q.164) T cell undergo blast transformation on treatment with all except:
(a)

Anti - CD3

(b)

Anti - Ig

(c)

Phytohemagglutinin

(d)

Concanavalin A

Your Response :

Correct Answer :

Exp:

Anti - Ig
T cell undergo blast transformation on treatment with phytohemagglutinin, concanavalin A and anti - CD3 while
anti-Ig, endotoxins, Staphylococcus aureus and EB virus induces blast transformation of B Cell.

(Q.165) Most common cause of surgical wound infection is:


(a)

Staphylococcus aureus

(b)

Staphylococcus epidermidis

(c)

Pseudomonas

(d)

Bacteroides

Your Response :

Correct Answer :

Exp:

Staphylococcus aureus
Staphylococcus aureus is the most common cause of surgical wound infections and is second only to coagulase
negative staphylococci as a cause of primary bacteremia.

(Q.166) Which of the following bacteria is novobiocin resistant?


(a)

Staphylococcus aureus

(b)

Staphylococcus epidermidis

(c)

Staphylococcus hyicus

(d)

Staphylococcus saprophyticus

Your Response :

Correct Answer :

Exp:

Staphylococcus saprophyticus
Staphylococcus saprophyticus causes urinary tract infection particularly in sexually active young women. It
is novobiocin resistant.

(Q.167) Abiotrophia species (nutritionally variant strept-ococci) requires supplementation of:


(a)

Pyridoxamine

(b)

Ascorbic acid

(c)

Vitamin D
Retinoic acid

(d)
Your Response :

Correct Answer :

Exp:

Pyridoxamine
Nutritionally variant streptococci require supplemental thiol compounds or active fomr as of vitamin B6
(pyridoxal or pyridoxamine) for growth in the laboratory.

(Q.168) Which of the following is not a Gram negative cocci?


(a)

Neisseria

(b)

Moraxella

(c)

Veillonella

(d)

Eikenella

Your
Response a
:
Correct
D
Answer :
Exp:

http://dbmci.examonair.com/Result/ShowAllQuestionInHtml.aspx?testid=5615

62/111

12/25/2014

Dr Bhatia Medical Coaching Institute:Online Test Platform


(Q.169) Which of the following is not true about post diphtheritic paralysis?
(a)

Palatine paralysis

(b)

Ciliary paralysis

(c)

Pupillary paralysis

(d)

Spontaneous recovery

Your Response :

Correct Answer :

Exp:

Pupillary paralysis
Postdiphtheritic paralysis occurs in the third & fourth week of the disease. Palatine and ciliary but not pupillary
paralysis is characteristic, and spontaneous recovery is the rule.

(Q.170) Which of the following is not a screening test for significant bacteriuria?
(a)

Griess nitrite test.

(b)

Catalase test.

(c)

Glucose test paper.

(d)

Eiken's test.

Your Response :

Correct Answer :

Exp:

Eiken's test.
Eiken's test used to differentiate between heat stable toxin and heat labile toxin of Escherichia coil. Tests used
for screening of significant bacteriuria are:
Griess nitrite test.
Catalase test.
Triphenyltetrazolium chloride test.
Microscopic demonstration of bacteria.
Glucose test paper.
Dip slide culture.

(Q.171) A 52 years female presented with fever of 2 weeks. Microscopic examinations of her urine shows pus cells. Staphylococcal

coagglutination showed antibody coated bacteria in her urine. Which is the likely site of infection?
(a)

Kidney

(b)

Ureter

(c)

Bladder
Urethra

(d)
Your Response :

Correct Answer :

Exp:

Kidney
Fever with pus cells in urine suggests the diagnosis of UTI. Absence of dysuria and frequency is against the
diagnosis of cystitis and urethritis. Furthermore bacteria coated with specific antibodies are present in the urine
only when the kidneys are infected. Antibody coated bacteria are detected by immunofluorescence using
fluorescent tagged antihuman globulin or by staphylococcal coagglutination.

(Q.172) Pseudohemoptysis is caused by:


(a)

Mycobacterium bovis

(b)

Mycobacterium avium

(c)

Serratia

(d)

Staphylococcus aureus

Your Response :

Correct Answer :

Exp:

Serratia
Serratia may grow in sputum after collection and may suggest hemoptysis because of the pigment formed
('Pseudohemoptysis').

(Q.173) Which of the following Rickettsia is spread by a mite?


(a)

Rickettsia typhi

(b)

Rickettsia rickettsi

(c)

Rickettsia conorii

(d)

Rickettsia 'akari

Your
Response b
:
Correct
D
Answer :
Exp:

http://dbmci.examonair.com/Result/ShowAllQuestionInHtml.aspx?testid=5615

63/111

12/25/2014

Dr Bhatia Medical Coaching Institute:Online Test Platform


Rickettsia akari
Vector for Rickettsia akari is gamasid mite.
Refer to the table in explanation to Q. no :
95.
(Q.174) Which of the following serotype of Legionella is associated with hospital acquired infection and has a poor prognosis?
(a)

Serotype 1

(b)

Serotype 2

(c)

Serotype 3
Serotype 6

(d)
Your Response :

Correct Answer :

Exp:

Serotype 6
Diseases and specific associated serotypes
(B) cereus food poisoning

Emetic 1,3,5

diarrhoeal- 2,6,8,9,10,12

Human botulism

A,B,E and rarely F. toxin production is determined by


bacteriophage in type C&D

Clostrial food poisoning

Cl. Perfringes type A

Enteritis necroticans / pigbel

Cl. Perfringes type C (beta toxin)

Listeria monocytogens human


infection

1/2a, 1/2b, 4b

Yersinia pseudotuberculosis

01

Yersinia enerocolitica

03,08,09

S. pyogens glomerulonephritis

49,53-55,59-64,1,12

Late onset neonatal meningitis by


group b streptococcus

Capsular type III

Pneumococcal pneumonia

Type 1 to type 8. Most virulent- type 3

Meningococcal meningitis

A,B,C,W,Y

Legionella pneumophilla

Most common serotype- 1.


Hospital acquired
disease and poor prognosis- serotype 6

Chlamydia trachomatis

Endemic blinding trachoma- (A)(B)Bo,C Inclusion


conjunctivitis, genital infection, infant pneumonia- D to
R
lymphogranuloma venerum- L1,L2,L3

Cryptococcus neoformans

Immunocompromised= var-grubii (serotype


A)
immunocompetent= var- gatti
(serotype B and C)

Polio virus

Most common and most epidemic- type 1 most antigenic and


endemic- type 2
vaccine induced paralysis-mutated type
3

Influenza virus

Antigenic variation- A>B


variation- C

Parainfluenza virus

Most common group in children- type1 most common


group in lower respiratory tract infection in infants- type 3

Rotavirus

Human illness = A>B>C adult diarroheal rota virus= grp


B small population of pediatric diarrhoeal diseases= group
c most common serotypes of group A in humans= G1 to G4 ,
G9

Astrovirus

Most common =1

Rhabdoviridea

Rabies virus= 1, rabies related virus= 2,3,4

Adenovirus

Follicular (swimming pool) conjunctivitis=


3,7
epidemic
keratoconjunctivitis (shipyard eye) =
8,9,37
respiratory disease in
children=1,2,5,6 sorethroat,
pneumonia=3,4,7,14,21 acute respiratory distress in
military recruits= 4,7,21 diarrhoea = 40,41 hemmorhagic
cystitis= 11,21

H. influenza

meningitis = Biotype 1 invasive disease= capsular subtype b

Coxsackie virus

Acute hemmorhagic cystitis= A24 generalised disease of


newborn= B2 to 5 hand-foot-mouth disease and herpangina=
A > B pleurodynia= B>A juvenile diabetes = B4 post viral
fatigue syndrome= B

Enterovirus

Acute hemmorhagic cystitis= ent 70 hand-foot-mouth


disease and herpangina= ent 71

HPV

Common warts / verruca vulgaris =


1,2,3,4
condyloma acuminata=
6,11 CIN (LOW RISK) = 6,11
Carcinoma (HIGH
RISK) = 16,18,31,33,45
epidermodyspl.
Verruciformis= 5,8

no Antigenic

(Q.175) Which of the following is the mechanism of action of Pentasaccharide?


(a)

Factor Xa inhibition, Thrombin inhibition

(b)

Factor Xa inhibition

http://dbmci.examonair.com/Result/ShowAllQuestionInHtml.aspx?testid=5615

64/111

12/25/2014

Dr Bhatia Medical Coaching Institute:Online Test Platform


(c)

Antithrombin inhibitor

(d)

Thrombin inhibition

Your
Response :
Correct
Answer :
Exp:

B
Factor Xa inhibition
Pentasaccharide
Mechanism of action: Factor Xa inhibition
Example: Fondaparinux, Idraparinux
Extra Edge: Newer factor Xa inhibitor :
a. Rivaroxaban
b. Apixaban (H-18th Pg-1000) Both are given orally.

(Q.176) Which is not true regarding Bernard soulier syndrome?


(a)

Ristocetin aggregation is normal

(b)

Aggregation with collagen and ADP is normal

(c)

Large platelets

(d)

Thrombocytopenia

Your Response :

Correct Answer :

Exp:

Ristocetin aggregation is normal


Intrinsic platelet defects to adhesion and aggregation.
Platelet adhesion and aggregation are modulated by glycoprotein receptors located on platelet surface.
GpIb / Ix
Mediates platelet adhesion

GpIIb / IIIa complex


Mediates platelet aggregation

VWF facilitates platelet adhesion by binding to this Fibrinogen facilitates platelet aggregation via sites
receptor
on this receptor.
Loss or defect in above glycoprotein receptors leads to rare platelet disorders causing bleeding
Bernard Soulier syndrome

Glanzmanns thrombasthenia

1. Autosomal recessive disorder

1. Autosomal recessive disorder

2. Deficiency / dysfunction of Gp Ib / Ix Receptor

2. Deficiency / dysfunction of Gp lIb / III a complex

3. Platelets cannot adhere to subendothelium


3. Platelets cannot aggregate because of lack of
because of lack of above receptors for VWF which above receptors for fibrinogen which form the
mediates platelet adhesion
bridges between platelets during aggregation.
4. Platelet aggregation to largely normal platelet
aggregation is normal in response to standard
agonists (collagen ADP, thrombin / but platelet fail
to aggregate in response to ristocetin as it acts by a
different mechanism
5. Other laboratory parameters
a. Thrombocytopenia may be present
b. Platelets on smears are abnormally large
c. Bleeding time is abnormally prolonged
d. VWF factor levels in plasma are normal
6. Clinical presentation
Recurrent episodes of severe mucosal
haemorrhage

4. Platelet aggregation is largely abnormal. Platelet


fail to aggregate in response to standard agonists,
(ADP, collagen, thrombin) as these require
fibrinogen binding, but aggregate normally to
ristocetin as it cause platelet clumping by a
different mechanism.
5. Other laboratory parameters
a. Platelet number is usually normal
b. Platelet morphology is usually normal
c. Bleeding time is abnormally prolonged
d. VWF factor level in plasma are normal
6. Clinical presentation
Recurrent episodes of severe mucosal
hemorrhage

(Q.177) A woman who pricked her finger while pruning some rose bushes develops a local pustule that progresses to an ulcer. Several

nodules then develop along the local lymphatic drainage. The most likely agent is :
(a)

Cryptococcus neoformans

(b)

Candida albicans

(c)

Sporothrix schenckii

(d)

Aspergillus fumigatus

Your Response :

Correct Answer :

Exp:

Sporothrix schenckii
The conidia or hyphal fragments of S. schenckii are introduced into the skin by trauma. Patients frequently recall
a history of trauma associated with outdoor activities and plants. The initial lesion is usually located on the
extremities but can be found anywhere (children often present with facial lesions). About 75% of cases are
lympho-cutaneous; ie, the initial lesion develops as a granulomatous nodule that may progress to form a
necrotic or ulcerative lesion. Meanwhile, the draining lymphatics become thickened and cord-like. Multiple
subcutaneous nodules and abscesses occur along the lymphatics.

(Q.178) Which of the following drugs crosses blood brain barrier by the process of pinocytosis?
(a)

Thyroxin

http://dbmci.examonair.com/Result/ShowAllQuestionInHtml.aspx?testid=5615

65/111

12/25/2014

Dr Bhatia Medical Coaching Institute:Online Test Platform


(b)

Phenobarbitone

(c)

Penicillin

(d)

Insulin-l

Your Response :

Correct Answer :

Exp:

Insulin
Insulin is a high molecular weight protein which is inactive orally and is unable to cross intact cell membrane. It is
transferred into the CNS by the process of pinocytosis.

(Q.179) Which of the following is a wrong match with respect to the anticancer drug and agents used against its toxicity as a preventive

measure?
(a)

Amifostine : Cisplatin

(b)

Calcium leucovorin : Cyclophosphamide

(c)

Dexzoxazone : Adriyamycin

(d)

Defibrotide : Busulphan

Your Response :
Correct Answer :

Exp:

Calcium leucovorin : Cyclophosphamide


TOXICITY

PROTECTIVE AGENT

Anthracyclines
(Doxorubicin/Adriamycin/Daunarubicin)

Cardiotoxicity

Dextrazoxane

Cisplastin

Nephrotoxicity

Amifostine

Cisplatin

8th nerve damage

Glutathione

Cisplatin

Emesis

Ondansetron

Cisplatin

Cardiotoxicity

Dextrazoxane

Cyclophosphamide

Hemorrhagic cystitis

MESNA

Methotrexate

Marrow suppression

Folinic acid / Leucovorin

Busulphan

Veno-occlusive disease of liver Defibrotide

ANTICANCER DRUG

(Q.180) Which of the substance increases the permeability of blood-brain barrier and improves penetration of drugs in the brain?
(a)

Substance P

(b)

Neurokinin A

(c)

Enkephalins
Histamine

(d)
Your Response :

Correct Answer :

Exp:

Enkephalins
Bradykinin and enkephalins increase the permeability of blood brain barrier by increasing pinocytosis. This
approach is being evaluated as a mean to improve penetration of anticancer drugs in the brain during
management of brain tumors.

(Q.181) t (2- 8) is associated with


(a)

CML

(b)

AML

(c)

T cell ALL

(d)

Burkitt lymphoma

Your Response :

Correct Answer :

Exp:

Burkitt lymphoma is acancerof thelymphatic system(in particular,B lymphocytes).


Genetics
Burkitt's lymphoma is associated withc-mycgenetranslocation. This gene is found at8q24.
The most common variant is t(8;14)(q24;q32). This involves c-myc and IGH@.
A rare variant is at t(2;8)(p12;q24). This involves IGK@ and c-myc.
Another rare variant is t(8;22)(q24;q11). This involves IGL@ and c-myc.
Types of Burkitts Lymphoma
TYPE

AGE

SEX

SITE ( most common to least common)

a. Endemic

8-15

M>>>F

Face, abdomen, CNS, LN,BM

b. Sporadic

8-15

M>>>F

Abdomen > face

c. AIDS associated

Young Adults

M=F

LN,BM

PBF - Starry sky appearance


Highly proliferative lymphoma- Doubling time 60 hours ( most rapid)
Malignant B cell characteristics
1. Normal B cells possess rearranged immunoglobulin heavy and light chain genes and each isolated B-cell

http://dbmci.examonair.com/Result/ShowAllQuestionInHtml.aspx?testid=5615

66/111

12/25/2014

Dr Bhatia Medical Coaching Institute:Online Test Platform


possesses a unique IgH gene rearrangement.
2. Since Burkitt lymphoma and other B-cell lymphomas are a clonal proliferative process, all tumor cells from one
patient are supposed to possess identical IgH genes.
3. When the DNA of tumor cells is analyzed usingelectrophoresis, a clonal band can be demonstrated since
identical IgH genes will move to the same position.
Treatment
Chemotherapy
a. Cyclophosphamide
c. Vincristine
e. Cytarabine
g. Etoposide

b. Doxorubicin

d. Methotrexate
f. Ifosfamide
h. Rituximab

Other treatments areimmunotherapy,bone marrow transplants,surgeryto remove the tumor, andradiotherapy.


List of the most important translocations in peculiar conditions :
TRANSLOCATION
t (8 ; 14)

SEEN IN
Burkitts lymphoma
ALL FAB Type L3
Immunoblastic B cell lymphoma

t (11;14)

Mantle cell lymphoma


CLL
Multiple myeloma

t (14;18)

Follicular lymphoma and some B cell lymphomas

t (15;17)

Promyelocytic leukemia M3

t (4;11)

ALL FAB : L1, L2 = Pre B ALL

t (6;14)

Cystadenocarcinoma of Ovary

t (3;8)

Renal adenocarcinoma
Mixed parotid tumour

t (8;21)

AML with maturation M2

t (11;22)

Desmoplastic Small Round Cell Tumours (DSRCTs):


Classical Ewings sarcoma
PNET
Askins tumour

t (12;22)

Clear cell sarcoma


Malignant melanoma of soft tissues

(Q.182)

t (12;16)

Myxoid liposarcoma

t (2;13)

Alveolar rhabdomyosarcoma

t (x;18)

Synovial cell sarcoma

t (11;18)

MALToma

t (14;15)

CLL / SLL

t (9;22)

CML and Pre B ALL

t (9;22, t (4;11) and t (1;19)

Pre B ALL

t (8;14), t (8;22) and t (2;8)

B cell ALL

t (2;5)

CD30 +ve Anaplastic large cell lymphoma

t (9;14)

Lymphoplasmacytoid lymphoma

t (8;14) and t (10;14)

T cell ALL

t (10;17)

Papillary thyroid carcinoma

Which of the following drug produces its effect by blocking GABA gated CI- channels?
(a)

Diazepam

(b)

Amiloride

(c)

Picrotoxin

(d)

Lignocaine

Your Response : a
Correct Answer : A
Exp:
(Q.183) In addition to liver, biotransformation of drugs can also occur in all of the following tissues except:
(a)

Lungs

(b)

Kidney

(c)

Skin
Spleen

(d)
Your Response :

Correct Answer :

Exp:

Spleen
Liver is the principal organ of drug metabolism.
Other tissues that display drug metabolizing activity include the GIT, the lungs, the skin
and the kidneys.

http://dbmci.examonair.com/Result/ShowAllQuestionInHtml.aspx?testid=5615

67/111

12/25/2014

Dr Bhatia Medical Coaching Institute:Online Test Platform


(Q.184) The ability of protamine to counteract the effects of heparin is an example of;
(a)

Competitive antagonism

(b)

Physiological antagonism

(c)

Chemical antagonism

(d)

Pharmacokinetic antagonism

Your Response :

Correct Answer :

Exp:

Chemical antagonism
In chemical antagonism one drug antagonizes the action of a second drug by binding to and inactivating it.
Protamine, a protein that is positively charged at physiological pH interacts with heparin that is negatively
charged and counteracts its effects.

(Q.185) A male patient, 25 years of age was operated by using the technique of balanced anesthesia. In the post operative period he

developed abdominal distension and retention of urine. Which of the following drug can be used to relieve his symptoms? .
(a)

Oral bethanechol

(b)

Injection bethanechol

(c)

Injection neostigmine

(d)

Oral physostigmine

Your Response :

Correct Answer :

Exp:

Injection neostigmine
The patient is having paralytic ileus with urinary retention. Oral administration of a drug is not likely to be very
effective as its absorption will be unpredictable. Bethanechol has a more selective effect on the urinary bladder
but less effect on GIT. Injection neostigmine will relieve both paralytic ileus and urinary retention.

(Q.186) The non-depolarizing skeletal muscle relaxant with a short duration of effect is:
(a)

Mivacurium

(b)

Succinyl choline

(c)

Atracurium

(d)

Vecuronium

Your Response :

Correct Answer :

Exp:

Mivacurium
Mivacurium has a short duration of action as it is destroyed by plasma cholinesterase.
Atracurium is inactivated by a form of spontaneous breakdown known as Hoffman's elimination and vecuronium
is eliminated by hepatic metabolism- both are intermediate acting. Succinyl choline is also destroyed by plasma
Acetyl choline-esterase but is a persistent depolarizing agent.

(Q.187) Estimated average glucose is based on which parameter?


(a)

Fasting glucose

(b)

Post prandial blood glucose

(c)

HbA1C

(d)

Random blood sugar

Your Response :

Correct Answer :

Exp:

HbA1C
eAG (Estimated Average Glucose)

(Ref. Hari-18th ed. Pg.-2992)

HBA1C should be measured in all individuals with DM during their initial evaluation and as part of their
comprehensive diabetes care.
As the primary predictor of long-term complications of DM, the HBA1C should mirror, to a certain extent,
the short-term measurements of Self monitoring of blood sugar (SMBG).
These two measurements are complementary in that recent intercurrent illnesses may impact the SMBG
measurements but not the HBA1C.
Likewise, postprandial and nocturnal hyperglycemia may not be detected by the SMBG of fasting and preprandial capillary plasma glucose but will be reflected in the HBA1C.
In standardized assays, the HBA1C approximates the following mean plasma glucose values:
HBA1C (%) to eAG (mg/dl)
1. 6.0% = 126 mg/dl

4. 7.0% = 154 mg/dl

2. 9.0% = 212 mg/dl

5. 10.0% = 240 mg/dl

3. 7.5% = 169 mg/dl

6. 8.5% = 197 mg/dl

7. 8.0% = 183 mg/dl


8. 6.5% = 140 mg/dl
9. 9.5% = 226 mg/dl

(Q.188) Anti-HIV drug known to cause myopathy resembling Mitochondrial myopathy with ragged red Fibers?
(a)

Zidovudine

(b)

Enfuvirtide

http://dbmci.examonair.com/Result/ShowAllQuestionInHtml.aspx?testid=5615

68/111

12/25/2014

Dr Bhatia Medical Coaching Institute:Online Test Platform


Nevirapine

(c)

Tenofovir

(d)
Your Response :

Correct Answer :

Exp:

Zidovudine.
Culprit AntiHIV drug

Adverse effect
Anti-HIV drug causes severe hepatotoxicity with steatohepatitis

Zidovudine.

Anemia is the most common side effect of

Zidovudine.

Anti-HIV drug known to cause myopathy resembling Mitochondrial


myopathy with ragged red Fibers

Zidovudine.

Local injection reactions, hypersensitivity reactions, increased rate of


bacterial pneumonia

Enfuvirtide
(fusion
inhibitor).

Anti-retroviral drug is contraindicated in pregnancy

Efavirenz,
Abacavir.

Anti-HIV drug that should never be given as re-challenge once history of


producing allergic reaction with it is known

Abacavir.

Peripheral neuropathy in common side effect of

Didanosine,
zalcitabine,
and
stavudine,
indinavir.

Non-nucleoside reverse transcriptase inhibitor anti-retroviral drug with


mild side effects like rash and hepatotoxicity

Nevirapine.

Pancreatitis

Didanosine,
Zalcitabine.

Nephrotoxic

Tenofovir

Abnormal dreams

Efavirenz

Hyperglycemia, fat redistribution, and lipid abnormalities

Saquinavir,
Ritonavir,
Nelfinavir,
Indinavir.

Nephrolithiasis

Indinavir

Drug-Induced Myopathies
Drugs
Lipid-lowering agents
Fibric acid derivatives
HMG-CoA reductase
inhibitors
Niacin (nicotinic acid)

Major Toxic Reaction


Drugs belonging to all three of the major classes of lipidlowering agents can produce a spectrum of toxicity:
asymptomatic serum creatine kinase elevation, myalgias,
exercised-induced pain, rhabdomyolysis, and
myoglobinuria.

Glucocorticoids

Acute, high-dose glucocorticoid treatment can cause


acute quadriplegic myopathy. These high doses of steroids
are often combined with nondepolarizing neuromuscular
blocking agents but the weakness can occur without their
use. Chronic steroid administration produces
predominantly proximal weakness.

Nondepolarizing
neuromuscular blocking
agents

Acute quadriplegic myopathy can occur with or without


concomitant glucocorticoids.

Zidovudine

Mitochondrial myopathy with ragged red fibers.

Drugs of abuse
Alcohol
Amphetamines
Cocaine

All drugs in this group can lead to widespread muscle


breakdown, rhabdomyolysis, and myoglobinuria.
Local injections cause muscle necrosis, skin induration,
and limb contractures.

Heroin
Phencyclidine
Meperidine
Autoimmune toxic myopathy Use of this drug may cause polymyositis and myasthenia
gravis.
D-Penicillamine
Amphophilic cationic drugs
Amiodarone
Chloroquine

All amphophilic drugs have the potential to produce


painless, proximal weakness associated with autophagic
vacuoles in the muscle biopsy.

Hydroxychloroquine
Antimicrotubular drugs
Colchicine

http://dbmci.examonair.com/Result/ShowAllQuestionInHtml.aspx?testid=5615

This drug produces painless, proximal weakness especially


in the setting of renal failure. Muscle biopsy shows
autophagic vacuoles.

69/111

12/25/2014

Dr Bhatia Medical Coaching Institute:Online Test Platform


(Q.189) Which of the following drug is given orally?
(a)

Dalteparin

(b)

Rivaroxaban

(c)

Lepirudin

(d)

All of the above

Your Response :

Correct Answer :

Exp:

Rivaroxaban

(Ref: Harrison, 18th edition, page 992)

Extra Edge: Dabigatran etexilate, an oral thrombin inhibitor, and Rivaroxaban &, Apixaban both are oral Factor
Xa inhibitor, are used for short-term thromboprophylaxis after elective hip or knee replacement surgery.
Dabigatran etexilate is used as an alternative to warfarin for stroke prevention in patients with atrial fibrillation.
(Q.190) The plasma protein binding of acidic drugs, like sulfonamides, is reduced in which of the following conditions?
(a)

Trauma

(b)

Rheumatoid arthritis

(c)

Celiac disease

(d)

Jaundice

Your Response :

Correct Answer :

Exp:

Jaundice
Jaundice is associated with hyperbilirubinemia which reduces binding of acidic drugs to plasma proteins. In
trauma, rheumatoid arthritis and celiac disease there is an increase in plasma a1 acid glycoprotein which
increases the binding of basic drugs like chlorpromazine, imipramine etc. to plasma proteins.

(Q.191) Inheritance of nephrogenic diabetic insipidus is


(a)

AR

(b)

AD

(c)

X-link recessive

(d)

X-link dominant

Your Response :

Correct Answer :

Exp:

X-link recessive
Usually, the hereditary form of nephrogenic DI is the result of an X-linked genetic defect which causes the
vasopressin receptor (also called the V2 receptor) in the kidney to not function correctly.
I am providing you with an Over-comprehensive list of various diseases and their mode of inheritance.
Remembering all of them is impractical but revise it again and again and you will not miss out on it in the exam.
AUTOSOMAL DOMINANT

AUTOSOMAL RECESSIVE

Hyperlipoprotinemias (except type I)

Abetalipoprotinemia

Achondroplasia

Cystic fibrosis

Neurofibromatosis

Hirshsprung disease

Adenomatous polyposis coli

Phenylketonuria

Marfans syndrome

Albinism

Retinoblastoma

Tay-sachs disease

Hereditary Spherocytosis

Alkaptonuria

Tuberous sclerosis

Galactosemia

Myotonic dystrophy

Maple Syrup urine disease

Von Willebrand disease

Beta thallasemia

Osteogenesis imperfecta

Homocystinuria

Acute intermittent porphyria

Lysosomal storage disorders

Holt oram syndrome

Alpha 1 Antitrypsin deficiency

MEN

Wilsons disease

VHL syndrome

Hemochromatosis

Wilms tumour

Glycogen storage disorders

Otosclerosis

Sickle cell disease

BRCA 1 and BRCA II Breast cancers

Congenital adrenal hyperplasia

Gardner,Peutz zeiger syndrome

Freidrichs ataxia

Retinitis pigmentosa

Spinal muscular atrophy

ALS

Hereditary vitamin D Resistant rickets

Osler Weber Rendu disease

Kartagener syndrome

Huntingtons chorea

Turcots syndrome

ABO Blood group system

Polycystic liver disease

Crouzon syndrome

Fanconi syndrome

Myotonic dystrophy

Gaucher syndrome

http://dbmci.examonair.com/Result/ShowAllQuestionInHtml.aspx?testid=5615

70/111

12/25/2014

Dr Bhatia Medical Coaching Institute:Online Test Platform


Hypophosphatemic rickets (FGF23 mutation)

FAX B (Fanconi anemia,Ataxia telangiectasia,Xeroderma


pigmentosum,Bloom syndrome)

Hemiplegic migraine

Swiss type Agammaglobulinemia

Charcot-Marie-Tooth disease

Nonketotic hyperglycinemia

Pigmentary glaucoma

Acrodermatitis enteropathica

Hypokalemic periodic paralysis


Diaphysial aclasia
X Linked Recessive Inheritance

X Linked Dominant Inheritance

Hemophila A and B

Vitamin D Resistant rickets

Duchenne muscular dystrophy

Familial Hypophosphatemia (PHEX )

G6PD deficiency

Blood group Xg

Hydrocephalus

Retts syndrome

Retinitis pigmentosa (most severe- less common form) Oro-Facio-Digital syndrome


Brutons Agammaglobulinemia

Incontinentia pigmentii

Hunters syndrome
Chronic Granulomatous disease
Wiskott-Aldrich syndrome
Nephrogenic Diabetes insipidus
Lesch-Nyhan syndrome
Fragile X syndrome
Dents disease
Kallman syndrome
Androgen insensitivity syndrome
Fabrys disease
Alport syndrome
MITOCHONDRIAL INHERITANCE (1 Question was asked in AIIMS NOV 09)
Mitochondrial myopathy
Lebers Hereditary Optic Neuropathy (LHON)
MELAS : Metabolic Encephalopathy + Lactic Acidosis + Stroke like episodes
NARP : Neuropathy + Ataxia + Retinitis Pigmentosa
Pearson syndrome : Bone marrow and Pancreatic failure
Kearns-Sayre syndrome : Ophthalmoplegia + Pigmentary degeneration of retina + Cardiomyopathy
MERRF syndrome : Myoclonic Epilepsy + Ragged Red Fibres
MMC : Maternally inherited Myopathy and Cardiomyopathy
CEOP : Progressive External Ophthalmoplegia
ADMIMY : Autosomal Dominant inherited Mitochondrial Myopathy and Mitochondrial Deletion
Leighs disease : movement disorder + regression + respiratory dyskinesia
Navajo Neurohepatopathy and MDS
(Q.192) The drug which exerts its CYP 3 A4 inhibiting effect by a mechanism based inactivation is:
(a)

Erythromycin

(b)

Ciprofloxacin

(c)

Amiodarone

(d)

Diltiazem

Your Response :

Correct Answer :

Exp:

Erythromycin
Most of the drugs cause CYP enzyme inhibition by competitive inhibition wherein two drugs compete with each
other for the same active site on the enzyme. A second type of enzyme inhibition is mechanism based or suicide
inactivation in which the effector compound is metabolized by the enzyme to a reactive form which irreversibly
binds to the enzyme and prevents any further metabolism of the drug. The drug which cause such suicide
inhibition are erythromycin, ticlopidine and disulfiram.
SOME SPECIFIC CYTP450s IN THE BODY :
Most Important Cytochrome in the body
Cytochrome metabolizing Ethanol

Cyt P 3A
Cyt P 2E1

Cytochrome induced in chronic alcoholism


Cytochrome metabolizing Aromatic Hydrocarbons

Cyt P 1A1

Cytochrome invoved in activation of Procarcinogens


Aromatic Hydrocarbon Hydroxylase(AHH)
Cytochrome involved in Warfarin metabolism

Cyt P 2C9

Cytochrome involved in Nicotine metabolism

Cyt P 2A6

(Q.193) Which of the following alfa adrenoceptor antagonists is relatively more selective for 1A adrenoceptors ?
(a)

Tamsulosin

(b)

Doxazosin

http://dbmci.examonair.com/Result/ShowAllQuestionInHtml.aspx?testid=5615

71/111

12/25/2014

Dr Bhatia Medical Coaching Institute:Online Test Platform


(c)

Phentolamine

(d)

Yohimbine

Your Response :

Correct Answer :

Exp:

Tamsulosin
Alfa adrenoceptors are broadly classified as alpha1 postsynaptic and alpha2 presynaptic. At least three subtypes
of both alpha1 and alpha2 receptors have been identified Viz. alpha1 A, (B) & D and alpha2 A, B & (C) Tamsulosin
exhibits some selectivity for alpha1 A receptors which are rich in the prostate, while alpha1B receptors are
plentiful in vascular smooth muscle. Tamsulosin is used as an agent of choice in the treatment of Benign
prostatic hypertrophy.

(Q.194) Which of the following antihypertensive agent produces its effect by causing release of nitric oxide and opening of K+ channels?
(a)

Hydralazine

(b)

Minoxidil

(c)

Sodium nitroprusside

(d)

Diazoxide

Your Response :

Correct Answer :

Exp:

Hydralazine
The vasodilatation produced by hydralazine depend in part on the presence of an intact blood vessel
endothelium, implying that it causes the release of nitric oxide which acts on the vascular smooth muscle to
cause relaxation. In addition, it may produce vasodilatation by activating K+ channels. Diazoxide and minoxidil
cause relaxation of vascular smooth muscle by opening of potassium channels, while sodium nitroprusside acts
by releasing nitric oxide.

(Q.195) Heal pad thickness is increase in which condition?


(a)

Acromegaly

(b)

Prolactinoma

(c)

Cushings syndrome

(d)

Hyperparathyroid

Your Response :

Correct Answer :

Exp:

Acromegaly
Test
Increase heal pad thickness seen on X-ray lateral view of the heal
Most common cause of increase heal pad thickness is heal injury (AIIMS Nov 2010).
Elevated IGF-I. Usually > 5 times normal.
Serum GH not suppressed following oral glucose. OGTT test (75 gms of glucose given . After 1 hour if GH>1
ng/ml- suggestive of GH tumor.) It is the most definite test.
Confirmation- MRI/CT

(Q.196) Which of the following diuretic does not require access to the tubular lumen for exerting its diuretic effect?
(a)

Spironolactone

(b)

Triamterine

(c)

Hydrochlorothiazide

(d)

Furosemide

Your Response :

Correct Answer :

Exp:

Spironolactone
Epithelial cells in the LDT and CD contain cytosolic mineralocorticoid receptors (MR) that have high affinity for
aldosterone. Aldosterone enters the epithelial cell from the basolateral membrane and binds to MRs the MR
aldosterone complex is translocated to the nucleus where it regulates the expression of aldosterone induced
proteins (ATP) which exert following effects:
(1) Activation of "silent" Na+ channels and Na+ pumps that pre-exist in the cell membrane
(2) Cycling of Na+ channels and Na+ pumps between the cytosol and cell membrane such that more channels
and pumps are located in the membrane.
(3) Increased expression of Na+ channels and Na+ pumps and
(4) Increased activity of enzymes in the mitochondria that are involved in ATP production.
This results in enhanced transepithelial NaCI transport and increased excretion of K+ and H+. Spironolactone and
eplerenone competitively inhibit the binding of aldosterone to the MRs, blocking the biological effects of
aldosterone. MRs antagonists are the only diuretics that do not require access to the tubular lumen to induce
diuresis.
Triamterine is a basic drug that is transferred by the organic base secreting mechanism in the proximal tubule,
while thiazides and furosemide are acidic drugs that gain access to the tubular lumen by organic acid secretory
mechanisms in the proximal tubule.

(Q.197) Release of which of the following hormones is an example of neuroendocrine secretion?


(a)

Growth hormone

http://dbmci.examonair.com/Result/ShowAllQuestionInHtml.aspx?testid=5615

72/111

12/25/2014

Dr Bhatia Medical Coaching Institute:Online Test Platform


(b)

Cortisol

(c)

Oxytocin

(d)

Prolactin.

Your Response :

Correct Answer :

Exp:

Oxytocin
The secretion of chemical messengers (neurohormones) from neurons into the blood is referred to as
neuroendocrine secretion. Thus, in contrast to the local actions of neurotransmitters at nerve endings,
neurohormones circulate in the blood before producing biological effects at target tissues. Oxytocin is
synthesized from magnocellular neurons whose cell bodies are located in the paraventricular and supraoptic
nuclei and whose nerve terminals terminate in the posterior pituitary gland. Target tissues for circulating
oxytocin are the breast and uterus, where the hormone plays a role in lactation and parturition, respectively.

(Q.198) All of the following are inducers of the microsomal enzyme system except:
(a)

Carbamazepine

(b)

Phenytoin

(c)

Phenobarbitone

(d)

Ticlopidine

Your Response :

Correct Answer :

Exp:

Ticlopidines
This is a very very important and frequently asked topic hence just by-heart the following list once and for all
:
MICROSOMAL ENZYME INDUCERS

MICROSOMAL ENZYME INHIBITORS

Phenobarbitone

Cimetidine

Phenytoin

Erythromycin

Carbamazepine

Ciprofloxacin

Rifampicin

INH

Phenylbutazone

Ketoconazole

Griseofulvin

Metronidazole

Glucocorticoids

Disulphiram

Chronic alcoholic intake

Allopurinol

Clofibrate

Ticlopidine

Meprobamate

OC pills

Smoking

Omeprazole

Chloral hydrate
DDT
Charcoal boiled meat
(Q.199) A cross-sectional view of a skeletal muscle fiber through the H zone would reveal the presence of what?
(a)

Actin, but no myosin

(b)

Actin and myosin

(c)

Myosin, but no actin

(d)

Actin and titin

Your Response :

Correct Answer :

Exp:

Myosin, but no actin


The H zone is the region in the centre of the sarcomere composed of the lighter bands on either side of and
including the M line. In this region, the myosin filaments are centered on the M line, and there are no
overlapping actin filaments. Therefore, a cross section through this region would reveal only myosin.

(Q.200) Which of the following conditions at the A-V node causes a decrease in heart rate?
(a)

Decreased acetylcholine levels

(b)

Increased norepinephrine levels

(c)

Increased potassium permeability

(d)

Increased calcium permeability

Your Response :

Correct Answer :

Exp:

Increased potassium permeability


An increase in potassium permeability causes a decrease in the membrane potential of the atrioventricular node.
Thus, it will be extremely hyperpolarized, making it much more difficult for the membrane potential to reach its
threshold level for conduction. This results in a decrease in heart rate. Increases in sodium and calcium
permeability and norepinephrine levels increase the membrane potential, which tends to increase the heart
rate.

(Q.201) Which of the following conditions is normally caused by sympathetic stimulation of the heart?

http://dbmci.examonair.com/Result/ShowAllQuestionInHtml.aspx?testid=5615

73/111

12/25/2014

Dr Bhatia Medical Coaching Institute:Online Test Platform


(a)

Acetylcholine release at the sympathetic endings

(b)

Decreased rate of conduction of the cardiac impulse

(c)

Decreased force of contraction of the atria

(d)

Increased force of contraction of the ventricles

Your Response :

Correct Answer :

Exp:

Increased force of contraction of the ventricles


Sympathetic stimulation of the heart normally causes an increased heart rate, increased rate of conduction of
the cardiac impulse, and increased force of contraction in the atria and ventricles. However, it does not cause
acetylcholine release at the sympathetic endings, because they contain norepinephrine. Parasympathetic
stimulation causes acetylcholine release. The sympathetic nervous system firing increases the permeability of
the cardiac muscle fibers, the sinoatrial node, and the atrioventricular node to sodium and calcium.

(Q.202) A decrease in which of the following would be expected to occur in response to a direct increase in renal arterial pressure?
(a)

Water excretion

(b)

Sodium excretion

(c)

Extracellular fluid volume

(d)

Glomerular filtration rate

Your Response :

Correct Answer :

Exp:

Extracellular fluid volume


An increase in renal arterial pressure results in pressure natriuresis and diuresis. The loss of sodium and water
tends to decrease extracellular fluid volume. Glomerular filtration rate would be normal or slightly increased in
response to an increase in renal artery pressure.

(Q.203) In normal kidneys, which of the following is true of the osmolarity of renal tubular fluid that flows through that early distal tubule in

the region of the macula densa ?


(a)

Usually tonic compared with plasma

(b)

Usually hypotonic compared with plasma

(c)

Usually hypertonic compared with plasma

(d)

Hypertonic, compared with plasma, in antidiuresis

Your Response :

Correct Answer :

Exp:

Usually hypotonic compared with plasma


As water flows up the ascending limb of the loop of Henle, solutes are reabsorbed, but this segment is relatively
impermeable to water progressive dilution of the tubular fluid occurs so that the osmolarity decreases to
approximately 100 mOsm/L by the time the fluid reaches the early distal tubule. Even during maximal
antidiuresis, this portion of the renal tubule is relatively impermeable to water and is therefore called the
diluting segment of the renal tubule.

(Q.204) In a patient with very high levels of aldosterone and otherwise normal kidney function, approximately what percentage of the

filtered load of sodium would be reabsorbed by the distal convoluted tubule and collecting duct?
(a)

40 to 60 percent

(b)

20 to 40 percent

(c)

10 to 20 percent

(d)

Less than 10 percent

Your Response :

Correct Answer :

Exp:

Less than 10
Although aldosterone is one of the body's most potent sodium-retaining hormones, it stimulates sodium
reabsorption only in the late distal tubule and collecting tubules, which together reabsorb much less than 10
percent of the filtered load of sodium therefore, the maximum percentage of the filtered load of sodium that
could be reabsorbed in the distal convoluted tubule and collecting duct, even in the presence of high levels of
aldosterone, would be less than 10 percent.
Na+ Reabsorption
The reabsorption of Na+ and Cl plays a major role in body electrolyte and water homeostasis. In addition, Na+
transport is coupled to the movement of H+, glucose, amino acids, organic acids, phosphate, and other
electrolytes and substances across the tubule walls. In the proximal tubules, the thick portion of the ascending
limb of the loop of Henle, the distal tubules, and the collecting ducts, Na+ moves by cotransport or exchange
from the tubular lumen into the tubular epithelial cells down its concentration and electrical gradients, and is
then actively pumped from these cells into the interstitial space. Na+ is pumped into the interstitium by Na, K

ATPase in the basolateral membrane. Thus, Na+ is actively transported out of all parts of the renal tubule except
the thin portions of the loop of Henle. It extrudes three Na+ in exchange for two K+ that are pumped into the
cell.

Transport Proteins Involved in the Movement of Na and Cl Across the Apical Membranes of Renal Tubular
Cells.

http://dbmci.examonair.com/Result/ShowAllQuestionInHtml.aspx?testid=5615

74/111

12/25/2014

Dr Bhatia Medical Coaching Institute:Online Test Platform

Apical Transporter

Function

Na/glucose CT

Na+ uptake, glucose uptake

Na+/Pi CT

Na+ uptake, Pi uptake

Na+amino acid CT

Na+ uptake, amino acid uptake

Na/lactate CT

Na+ uptake, lactate uptake

Na/H exchanger

Na+ uptake, H+ extrusion

Cl/base exchanger

Cl uptake

NaK2Cl CT

Na+ uptake, Cl uptake, K+ uptake

Na/H exchanger

Na+ uptake, H+ extrusion

K+ channels

K+ extrusion (recycling)

Distal convoluted tubule

NaCl CT

Na+ uptake, Cl uptake

Collecting duct

Na+ channel (ENaC)

Na+ uptake

Site
Proximal tubule

Thick ascending limb

Normally about 60% of the filtered Na+ is reabsorbed in the proximal tubule, primarily by NaH exchange.
Another 30% is absorbed via the Na2ClK cotransporter in the thick ascending limb of the loop of Henle, and

about 7% is absorbed by NaCl cotransporter in the distal convoluted tubule. The remainder of the filtered Na+,
about 3%, is absorbed via the ENaC channels in the collecting ducts, and this is the portion that is regulated by
aldosterone in the production of homeostatic adjustments in Na+ balance.
(Q.205) Warthin Finkeledy bodies are seen in ?
(a)

Measles

(b)

Mumps

(c)

Small pox
Chicken-pox

(d)
Your Response :

Correct Answer :

Exp:

Measles
REMEMBER : List of various BODIES seen in various diseases :
BODIES

DISEASE

Hematoxylin body

Libman Sacs endocarditis

Meyers Kouvenaars body

Filariasis

Rice bodies

Fibrous loose bodies in joints in TB arthritis

Michalis-Guttman body

Malakoplakia

Herring body

Posterior pituitary bodies

Halberstaedler Prowazek body

Trachomatosis

Picks body

Fronto-Temporal dementia / Picks disease

Moosers body

Endemic typhus

Lofora body

Familial myoclonic epilepsy

Tuftstone body

Metachromatic leukodystrophy

Cystoid body

CMV retinitis

Paschen body

Elementary body of variola(Smallpox)

Guarneri body

Inclusion body of variola-vaccinia

Molluscum body

Inclusion body of molluscum contagiosum

LD body

Leishmaniasis (It is a amastigote within a


macrophage)

Negri body

Rabies (Intra-cytoplasmic)

Bollinger body

Fowlpox

Henderson-Peterson body

Molluscum contagiosum

Torres body

Yellow fever

Warthin Finkeledy body

Measles

Miyagawa body

LGV

Levinthal Cole Lilly body

Psittacosis

Copper-penny body

Histologic characteristic of Chromoblastomycosis

Parappenheimer body

Sideroblastic anemia

Gamma gandy body

Sickle cell anemia

Masson body

Cryptogenic organizing pneumonia

Asteriod body, Schaumann body

Sarcoidosis

Russel and Dutcher body

Multiple myeloma

Globoid body

Krabbes disease

Aschoff body

Rheumatic heart disease

http://dbmci.examonair.com/Result/ShowAllQuestionInHtml.aspx?testid=5615

75/111

12/25/2014

Dr Bhatia Medical Coaching Institute:Online Test Platform


(Q.206) All of the following are Type I hypersensitivity reactions except ?
(a)

Eczema

(b)

Casoni test / reaction.

(c)

Spring catarrh

(d)

Chronic graft rejection

Your Response :

Correct Answer :

Exp:

Chronic graft rejection is a type IV reaction. Rest all options are type I reactions.
All types of HYPERSENSITIVITY REACTIONS :
TYPE II

TYPE I
Local :
Eczema
Hay fever
Asthama {atopy}
Systemic :
Anaphylaxis

Change in cellular
function :
Increase : graves
disease

TYPEIII
Local : Arthus reaction.

TYPE IV
Tuberculin test

Systemic : Serum sickness Frei test


Lepromin test
Fairley test

Decrease : myasthenia
gravis

Patch test

Theobald smith
phenomenon

Antibody dependant
cell mediated toxicity

Schicks test

Jones Mote reaction

Praustnitz Kustner
reaction

Phagocytosis

PSGN

Chronic graft rejection

Casoni test

Blood transfusion
reactions

Erythema nodosum
leprosum and Lucios
phenomenon

Hypersensitivity
pneumonitis/Farmers
lung{both III and IV

Spring catarrh

Hemolytic anemias in
IM and mycoplasmal
infection

Hypersensitivity
pneumonitis/Farmers
lung{both III and IV}

Type I lepra reaction

ABPA{both I and III}

Agranulocytosis

ABPA {both I and III}

Beryliosis

Erythema multiforme

Histoplasmosis

Dialysis reaction type ITP


A
Allergic
rhinitis/sinusitis

Goodpasture syndrome SLE

Phlyctenular
conjunctivitis

Giant papillary
conjunctivitis{both I
and IV}

Pernicious anemia

Diabetes mellitus type I

urticaria

Dialysis reaction type B PAN

GBS

Steven-Johnson
syndrome

Hyperacute rejection of
renal transplamt

Sarcoidosis

Rheumatic fever

HSP

Poison Ivy dermatitis

Rheumatoid arthritis

Diabetes mellitus type SABE


II

Giant papillary
conjunctivitis{both I and
IV}

Blood transfusion
rection

Reactive arthritis

Schistosomiasis
granuloma formation

Optic neuritis

Erythema multiforme

Sympathetic
ophthalmitis.

(Q.207) Kegel exercises are advised in which condition?


(a)

JRA

(b)

Over active bladder

(c)

Duchenne myopathy

(d)

Talar dome fracture

Your Response :

Correct Answer :

Exp:

Over active bladder


Ref: Nelson, 18th edition, page 2252
Another treatment is biofeedback, in which children are taught pelvic floor exercise (Kegel exercise), because
there is evidence that daily performance of these exercise may reduce or eliminate unstable bladder
contractions.
TREATMENT OPTIONS
Both surgical and non-surgical treatment options are available for overactive bladder.
Non-surgical Treatments
Kegel exercises
Kegel exercises strengthen the pelvic floor muscles. These exercises can help control stress urinary continence,
urge urinary incontinence, overactive bladder, fecal incontinence and can help slow the progression of vaginal
prolapsed. Kegel exercises must be done correctly and regularly to work.
Pelvic floor therapy
Pelvic floor therapy consists of a series of visits to a physical therapist with specialized training in the treatment
of pelvic floor problems. The physical therapist uses a combination of the techniques listed below depending on
the type of urogynecological condition present.

http://dbmci.examonair.com/Result/ShowAllQuestionInHtml.aspx?testid=5615

76/111

12/25/2014

Dr Bhatia Medical Coaching Institute:Online Test Platform


Behavioral modification:education on diet, fluid intake and other lifestyle changes to manage various
bothersome symptoms
Bladder training:learning to use the pelvic floor muscles to suppress overactive bladder symptoms (urinary
urgency, frequency, noturia and urge urinary incontinence)
Biofeedback:an intravaginal device is used to train the pelvic floor muscles to contract or relax correctly
Functional electrical stimulation:a device that can be used intravaginally or externally that delivers a gentle
electrical current to activate or relax the nerves and muscles in the pelvis
Manual therapy:pressure applied to and released from muscles in spasm to relax them and increase blood flow
to the area for healing
Joint and tissue mobilization:gentle manipulation to help calm the muscles and nerves of the pelvis
Low-dose vaginal estrogen
Low-dose vaginal estrogen replaces declining estrogen in vaginal and urethral tissues easing symptoms of vaginal
dryness, itching, and irritation as well as urinary urgency, frequency and incontinence. Low-dose vaginal
estrogen comes in a cream (Estrace or Premarin), suppository (Vagifme) or a ring (Estring).
Various overactive bladder medications
There are several brands of overactive bladder medications on the market to treat urge urinary incontinence.
These medications work by relaxing the bladder muscle. Side effects are usually mild and include dry mouth, dry
eyes, blurred vision, urinary retention, constipation, dizziness or drowsiness. Changing the brand or dose of
medication can decrease side effects. The different brands of overactive bladder medications include:
(tolterodine tartrate) Detrol
Surgical Treatments
InterStim Therapy
InterStim Therapy is an FDA-approved treatment for urinary urgency, frequency, urge incontinence and
retention. The InterStim is a small device that is implanted under the skin of one of the upper buttocks. It
works by gently stimulating the sacral nerves to help the bladder function more normally.
BOTOX (botulinum toxin type A)
Currently Botox is considered investigational in the management of urge urinary incontinence. It can be very
effective in patients with neurogenic bladder or with urge incontinence not responsive to anticholinergic
medications.
(Q.208) AMES classification is used for ?
(a)

Thyroid cancer

(b)

Laryngeal cancer

(c)

Salivary gland cancer

(d)

Tongue calcification

Your Response :

Correct Answer :

Exp:

Thyroid cancer
REF Schwartz 8th Ed Chapter 45
Thyroid Surgery
The prevalence of thyroid disease increases with advancing age. The etiologies, risk factors, and presentations of
thyroid disease are similar across all ages and, therefore, are not discussed in detail. Of note, however, is that
elderly patients more often present with cardiac manifestations of hyperthyroidism such as atrial fibrillation than
do their younger counterparts. A common finding requiring evaluation in elderly patients is the presence of a
thyroid nodule, usually detected by physical examination. These nodules are usually single and four times more
common in women, making them a particular concern for postmenopausal elderly women. Indications for
surgical intervention for thyroid nodules are dependent on the characteristics of the nodule (i.e., whether it is
benign or malignant, or whether the patient is euthyroid or thyrotoxic). In addition, surgical intervention
becomes necessary if the nodule enlarges, producing compressive symptoms.
Papillary carcinoma in elderly patients tends to be sporadic with a bell-shape distribution of age at presentation,
occurring primarily in patients age 30 to 59 years. The incidence of papillary carcinoma decreases in patients
older than 60 years of age. 18 However, patients older than 60 years of age have increased risk of local
recurrence and for the development of distant metastases. Metastatic disease may be more common in this
population secondary to delayed referral for surgical intervention because of the misconception that the
surgeon will be unwilling to operate on an elderly patient with thyroid disease. Age is also a prognostic indicator
for patients with follicular carcinoma. There is a 2.2 times increased risk of mortality from follicular carcinoma
per 20 years of increasing age. 19 Therefore, prognosis for elderly patients with differentiated thyroid
carcinomas is worse when compared to younger counterparts. The higher prevalence of vascular invasion and
extracapsular extension among older patients is in part responsible for the poorer prognosis in geriatric patients.
Advancing age leads to increased mortality risk for patients with thyroid cancer and is demonstrated by the
AMES (age, metastases, extent of primary tumor, and size of tumor) classification system developed by the
Lahey Clinic (Table 45-4).

AMES Classification of Thyroid Cancers

A:

Age

Low Mortality Risk

High Mortality Risk

Men: <41 years of age

Men: >41 years of age

Women: <51 years of age

Women: >51 years of age

M: Metastases

Absence of distant metastases

Presence of distant metastases

E:

Intrathyroidal papillary cancer


(confined to thyroid)

Extrathyroidal papillary cancer

Follicular cancer with minor


capsular involvement

Follicular cancer with major capsular


involvement

Extent of primary
tumor

http://dbmci.examonair.com/Result/ShowAllQuestionInHtml.aspx?testid=5615

77/111

12/25/2014

Dr Bhatia Medical Coaching Institute:Online Test Platform


S:

Size of tumor

Primary tumor <5 cm in diameter

Primary tumor

5 cm in diameter

(regardless of extent of disease)

(Q.209) True statement regarding surfactant is ?


(a)

Mixture of dipalmitoylphosphatidylcholine (DPPC), other lipids, and proteins

(b)

Produced by type II alveolar epithelial cells

(c)

Surface tension is inversely proportional to the surfactant concentration per unit area

(d)

All of above

Your Response :

Correct Answer :

Exp:

All of the above


Surfactant
The low surface tension when the alveoli are small is due to the presence in the fluid lining the alveoli of
surfactant, a lipid surface-tension-lowering agent. Surfactant is a mixture of dipalmitoylphosphatidylcholine
(DPPC), other lipids, and proteins If the surface tension is not kept low when the alveoli become smaller during
expiration, they collapse in accordance with the law of Laplace. In spherical structures like the alveoli, the
distending pressure equals two times the tension divided by the radius (P = 2T/r); if T is not reduced as r is
reduced, the tension overcomes the distending pressure. Surfactant also helps to prevent pulmonary edema. It
has been calculated that if it were not present, the unopposed surface tension in the alveoli would produce a 20
mm Hg force favoring transudation of fluid from the blood into the alveoli.
Approximate Composition of Surfactant.

Component

Percentage
Composition

Dipalmitoylphosphatidylcholine 62
Phosphatidylglycerol

Other phospholipids

10

Neutral lipids

13

Proteins

Carbohydrate

Surfactant is produced by type II alveolar epithelial cells Typical lamellar bodies, membrane-bound organelles
containing whorls of phospholipid, are formed in these cells and secreted into the alveolar lumen by exocytosis.
Tubes of lipid called tubular myelin form from the extruded bodies, and the tubular myelin in turn forms the
phospholipid film. Following secretion, the phospholipids of surfactant line up in the alveoli with their
hydrophobic fatty acid tails facing the alveolar lumen. Surface tension is inversely proportional to their
concentration per unit area. The surfactant molecules move further apart as the alveoli enlarge during
inspiration, and surface tension increases, whereas it decreases when they move closer together during
expiration. Some of the proteinlipid complexes in surfactant are taken up by endocytosis in type II alveolar cells
and recycled.
Formation of the phospholipid film is greatly facilitated by the proteins in surfactant. This material contains four
unique proteins: surfactant protein (SP)-A, SP-B, SP-C, and SP-(D) SP-A is a large glycoprotein and has a collagenlike domain within its structure. It has multiple functions, including regulation of the feedback uptake of
surfactant by the type II alveolar epithelial cells that secrete it. SP-B and SP-C are smaller proteins, which
facilitate formation of the monomolecular film of phospholipid. A mutation of the gene for SP-C has been
reported to be associated with familial interstitial lung disease. Like SP-A, SP-D is a glycoprotein. Its full function
is uncertain. However, SP-A and SP-D are members of the collectin family of proteins that are involved in innate
immunity in the conducting airway as well as in the alveoli.
(Q.210) False statement regarding LASIK is ?
(a)

Stable refraction for 18 months prior to procedure is essential.

(b)

Can be used to treat keratoconus.

(c)

Can be used to treat myopia upto 30D.

(d)

None of the above

Your Response :

Correct Answer :

Exp:

Can be used to treat keratoconus.


Keratoconus is one of the contraindications for performing LASIK procedures.
LASIK [Laser assisted i-n situ keratomileusis] Indications
Age minimum is 18 years
Range of correction - 1 to 30 Diopter.
Refraction should be stable for 18 months (or 1 years) .
Central corneal thickness> 500 microns
Contraindication of LASIK
Monocular patients

. Keratoconus

Infections eg conjunctivitis, keratitis

. Poor endothelial cell count in cornea

Glaucoma

Autoimmune disease

. Dry eye

Thin cornea 450 micron)

http://dbmci.examonair.com/Result/ShowAllQuestionInHtml.aspx?testid=5615

1500)

Diabetic retinopathy

78/111

12/25/2014

Dr Bhatia Medical Coaching Institute:Online Test Platform


Currently LASIK procedure is being considered the refractory surgery of choice for myopia
upto-30 D
Advantage

Disadvantage

1.

Minimal or no post operative pain

1. More expensive

2.

Recovery of vision is very early

2. Require greater surgical skill

3.

No risk of perforation during

3. There is potential risk of flap related

4.

Surgery

complications include

No residual haze

(I) Intra operative flap amputation


(II) Wrinkling of flap on repositioning
(III) Post operative flap dislocations!
subluxation
(IV) Epithelization of flap bed interface
(V) Irregular astigmatism

(Q.211) Stinger is seen in which condition?


(a)

Neck Injury

(b)

Head injury

(c)

Brachial plexus injury

(d)

Knee injury

Your Response :
Correct Answer :

Exp:

Brachial plexus injuries


The brachial plexus includes nerves originating from C5T1 and emerging from the spinal column in the deep
triangle of the neck. The upper trunk (C56) can be contused or stretched during football when tackling with the
shoulder or having the head forcefully flexed laterally. Manifestations include unilateral burning (known as a
burner or stinger), paresthesia, and weakness in the arm, usually in a C56 distribution manifested as the
inability to forward flex or abduct the shoulder. These symptoms often resolve spontaneously within minutes.
Bilateral symptoms, such as transient quadriplegia, are an indication to curtail participation until the patient is
evaluated by MRI. If a patient has recurrent stingers, an MRI of the cervical spine is indicated.
(Q.212) Bones injured in a 'Side-Swipe fracture' are:
(a)

Humerus and forearm bones

(b)

Femur and leg bones

(c)

Femur only

(d)

Leg bone only

Your Response :

Correct Answer :

Exp:

Humerus and forearm bones


Side - Swipe fracture is an elbow injury sustained when one's elbow projecting out of a car, is 'side-swept' by
another vehicle. It has a combination of the humerus with fractures of proximal end of radius and/or ulna. It is
also called baby-car fracture.

(Q.213) A 30 year old male Deepak was having fracture shaft of right femur. After 2 days of injury he was having restlessness and dyspnoea.

On examination he was having tachypnoea and tachycardia & petechial rash. Which of the investigation is not of much use?
(a)

Urinary fat globules

(b)

Pulmonary function test

(c)

Blood oxygen saturation

(d)

Fundus examination

Your Response :

Correct Answer :

Exp:

Pulmonary function test


This is case of fat embolism syndrome. Its features are:
- Occlusion of small vessels by fat globules
- More common with fractures of long bones and multiple fractures
Cerebral type - patient is drowsy, restless and disoriented and gradually goes into a state of coma.
- Pulmonary type - Tachypnoea and tachycardia.
- Sputum and urine fat globules seen.
- Striate hemorrhages and exudates in retina
- Snow - storm appearance in chest X- ray.
- Treatment - respiratory support, heparinization intravenous low - molecular weight dextran and
corticosteroids.

(Q.214) Which of the following is not a common site for avascular necrosis after trauma?
(a)

Head of the femur

(b)

Distal pole of scaphoid

(c)

Body of the talus

http://dbmci.examonair.com/Result/ShowAllQuestionInHtml.aspx?testid=5615

79/111

12/25/2014

Dr Bhatia Medical Coaching Institute:Online Test Platform


Proximal pole of lunate

(d)
Your Response :

Correct Answer :

Exp:

Distal pole of scaphoid


Proximal pole of scaphoid (not distal pole) has high probability to undergo avascular necrosis after fracture
through the waist of the scaphoid.

(Q.215) Which of the following nerve is most frequently damaged in musculo-skeletal injuries?
(a)

Radial nerve

(b)

Axillary nerve

(c)

Median nerve

(d)

Common peroneal nerve

Your Response :

Correct Answer :

Exp:

Radial nerve
The radial nerve is the most frequently damaged nerve in musculo - skeletal injuries.
Most common nerve injured in supracondylar fracture of

ANTERIOR INTEROSSEOUS NERVE

humerus
Tardy ulnar nerve palsy

CUBITUS VALGUS

Most Common Nerve involved in anterior shoulder


dislocation

AXILLARY (CIRCUMFLEX HUMERAL)


NERVE

Most Common Nerve involved in surgical neck humerus


fracture

AXILLARY NERVE

Most Common Nerve involved in fracture shaft of


Humerus

RADIAL NERVE

Most Common Nerve involved in fracture medial condyle ULNAR NERVE


of humerus
Most Common Nerve involved in Monteggia fracturedislocation

POSTERIOR INTEROSSEOUS NERVE

Most Common Nerve involved in scapholunate dislocation MEDIAN NERVE


(Q.216) Most frequent indication for internal fixation of a fracture is:
(a)

Unstable fractures

(b)

Early mobility of the patient

(c)

Associated neurological injury

(d)

Associated vascular injury

Your Response :

Correct Answer :

Exp:

Unstable fractures
Most frequent indication for internal fixation is, when a fracture is so unstable that it is difficult to maintain it in
an acceptable position by conservation meAns:
Even in the third millennium, the use of nonoperative treatment is still appropriate for certain types of fractures.
The benefits of IF must be balanced against its associated risks. The main advantages of IF are precise restoration
of the osseous anatomy and early mobilization with at least partial weight-bearing. The principal disadvantages
are the increased risk of infection and the fact that healing may be impaired if this demanding technique is not
used properly. Generally speaking, patients are more comfortable after IF than with cast treatment or external
fixation.
Displaced intra-articular fractures are best treated by IF, as it is the only fixation method that allows anatomic
restoration of the articular surface. Early joint motion is possible, with improvement in the range of movement
and the condition of the articular cartilage. Even in the hands of experienced surgeons, displaced fractures of the
diaphysis are often unstable after closed reduction and the application of a cast. Comminuted fractures are not
suitable for nonoperative treatment. Some fractures, such as displaced fractures of the femoral neck, the
femoral head, or the talus, require immediate compression osteosynthesis to reduce the risk of osteonecrosis.
Increasingly, many patients elect to have IF even for nondisplaced fractures, as they may return to their work or
sporting activities at an earlier stage. Nevertheless, the decision to operate or not is often based on the
surgeon's preference and experience.

(Q.217) Knuckle - bender splint is used for:


(a)

Ulnar nerve palsy

(b)

Radial nerve palsy

(c)

Median nerve palsy

(d)

Axillary nerve palsy

Your
Response a
:
Correct A
Answer :
Exp:

Ulnar nerve palsy


Splint Name
Cock - up splint

Use
Radial nerve

http://dbmci.examonair.com/Result/ShowAllQuestionInHtml.aspx?testid=5615

80/111

12/25/2014

Dr Bhatia Medical Coaching Institute:Online Test Platform


palsy
Knuckle - bender splint
palsy

Ulnar nerve

Aero plane splint


injury

Brachial plexus

Toe raising splint

For foot drop

(Q.218) Injury to lateral condyle of humerus is included in which type epiphyseal injury according to Salter and Harris classification?
(a)

Type II

(b)

Type III

(c)

Type IV
Type V

(d)
Your Response :

Correct Answer :

Exp:

Type IV
Injury to lateral condyle of humerus is type IV injury. Open reduction and internal fixation is it's treatment of
choice. Growth disturbance is common even after open reduction.
This fracture is the second most common distal humerus fracture in children. They occur between the ages of 4
and 10 years. These fractures occur when a varus force is applied to the extended elbow. They tend to be
unstable and become displaced because of the pull of the forearm extensors.
Since these fractures are intra-articular they are prone to nonunion because the fracture is bathed in synovial
fluid.
Lateral condyle fractures are classified according to Milch. They are Salter-Harris IV epiphysiolysis fractures.
Most are Milch II fractures that travel from the lateral humeral metaphysis above the epiphysis and exit through
the lateral crista of the trochlea leading to an unstable humeral ulnar articulation.
Milch Classification
Type I
Type II

Fracture line is lateral to trochlear groove (considered a SH IV fracture)


Fracture line into trochlear groove (considered a SH II fracture)

Fracture Displacement Classification


Type 1

<2mm, indicating intact cartilaginous hinge

Type 2

2-4mm, displaced joint surface

Type 3

>4mm, joint displaced and rotated

(Q.219) After a fracture at the junction of middle and outer - thirds of the clavicle, the outer fragment is displaced:
(a)

Upwards & medially

(b)

Upwards & laterally

(c)

Downwards & medially

(d)

Downwards & laterally

Your Response :

Correct Answer :

Exp:

Downwards & medially


Fracture of the clavicle:
- Common fracture at all age groups
- Common site is junction of middle & outer third
- Outer fragment displaces medially and downwards because of the gravity and pull by the pectoralis major
muscle attached to it
- Figure - of - 8 bandage is applied for treatment - Shoulder stiffness is a common complication

(Q.220) A 17 year old child hyper-extended the fingers of a 13 year old child while quarreling. Which metacarpo-phalangeal joint is most

likely to dislocate?
(a)

Second

(b)

Third

(c)

Fourth
Fifth

(d)
Your Response :

Correct Answer :

Exp:

Second
The metacarpo-phalangeal joint of the index finger is affected most commonly. Open reduction is required in
most cases.

(Q.221) Which of the following correctly describes a component used as a bone graft substitute that induces differentiation of stem cells

into osteogenic cells ?


(a)

Osteoconduction

(b)

Osteoinduction

http://dbmci.examonair.com/Result/ShowAllQuestionInHtml.aspx?testid=5615

81/111

12/25/2014

Dr Bhatia Medical Coaching Institute:Online Test Platform


(c)

Osteogenesis

(d)

None of the above

Your Response :

Correct Answer :

Exp:

Osteoinduction
REF : Canale & Beaty: Campbell's Operative Orthopaedics, 11th ed.
Chapter 1.
Classification of Bone Graft Substitutes
Property

Description

Classes

Osteoconduction Provides a passive porous scaffold to


support or direct bone formation

Calcium sulfate, ceramics, calcium phosphate


cements, collagen, bioactive glass, synthetic
polymers

Osteoinduction

Induces differentiation of stem cells into


osteogenic cells

Demineralized bone matrix, bone morphogenic


proteins, growth factors, gene therapy

Osteogenesis

Provides stem cells with osteogenic


potential, which directly lays down new
bone

Bone marrow aspirate

Combined

Provides more than one of the above


mentioned properties

Composites

(Q.222) A 30 year male presents with fever, malaise, sore throat. On examination patient has multiple blisters and crusting on the muco-

cutaneous surface. Membrane formation is seen on the conjunctiva. Early Conjunctival scarring is also present. Diagnosis of Steven
Johnson syndrome is made. Following is true about the disease except:
(a)

Males more common affected than females

(b)

Papillary conjunctivitis is most common feature in eye

(c)

Dry eyes is common sequelae

(d)

Steroids are contraindicated in acute stage.

Your Response :

Correct Answer :

Exp:

Steroids are contraindicated in acute stage.


Topical steroids are mainly used in acute stage to prevent scarring and decrease inflammation. Some times oral
steroids are used SJS is an acute severe mucocutaneous blistering disease.
Males> females.
Probably etiology is immunological Hypersensitivity due to drugs and viral illness is known
The disease is self limiting
Symptoms: Fever, malaise, sore throat, cough, arthralgia
Signs: crusty eyelids, papillary conjunctivitis, membranous conjunctivitis Complications: symblepharon, dry eyes,
limbal stem cell deficiency
Treatment: topical steroids, lubricants, antibiotics

(Q.223) True about various stages of cataract ?


(a)

4th Purkinje image not seen in Immature cataract

(b)

Iris shadow not seen in mature cataract.

(c)

Deep anterior chamber in hypermature cataract

(d)

All of above

Your Response :

Correct Answer :

Exp:

All of above
Differentiating Various Stages of Cataract
Immature

Mature

Hypermature

Vision

6/9 - FC

HM - PL

HM FC

Anterior Chamber

Normal (shallow
inintumescent)

Normal (shallow
inintumescent)

Normalto deep

Color of Lens

Grayish white

Pearly white

Milky white (with


brown crescent of
nucleus) or chalky
white

Iris shadow

Seen

Not seen

Not seen

Distant
DirectOphthalmoscopy

Black patches against No red glow seen


red glow

No red glow seen

Purkinje-SansonImages

4thimage not seen

4, 5 & 6thimages not


seen

Features

http://dbmci.examonair.com/Result/ShowAllQuestionInHtml.aspx?testid=5615

4, 5 & 6thimages not


seen

82/111

12/25/2014

Dr Bhatia Medical Coaching Institute:Online Test Platform


(Q.224) Amsler grid test is used to diagnose :
(a)

Diabetic retinopathy

(b)

Age related macular degeneration

(c)

Retinoblastoma

(d)

Choroidal metastatic of malignancy

Your Response : b
Correct Answer : B
Exp:

Age related macular degeneration


Amsler grid test is a test chart consisting of multiple squares like a square
page book
It has a dot at centre of the chart
In macular degeneration disease patient has micropsia, macropsia and
metamorphopsia
So the parallel lines appear distorted to the patient
It is a simple home based test for patient.

(Q.225) Stochastic effect is seen in which condition?


(a)

Cancer chemo therapy

(b)

Radio therapy

(c)

Post transplant immuno suppressive therapy

(d)

After coronary artery bypass surgery

Your Response :

Correct Answer :

Exp:

Radio therapy
Ref: Nelson, 18th edition, page 2899
BIOLOGIC EFFECTS OF RADIATION
Stochastic (random)effects are of greater concern because they can occur at any dose; i.e., there is no threshold,
with the probability of an effect increasing with increasing dose. These effects can be caused by any radiation
striking vulnerable tissue (most importantly DNA, but cytoplasm also may be at risk) and causing irreversible
damage. These effects lead to the linear no dose threshold (LNT) concept, which states that radiation damage
increases with increasing dose in a linear fashion. This concept stresses that no level of radiation exposure can be
considered to be absolutely safe.

(Q.226) A 45 yr old lady presents to casualty with pain, redness, watering and photophobia. O/E perilimbal congestion +, slight cornea

edema +, AC flare +. Pupil is mid dilated. Provisional diagnosis is :


(a)

Atropine toxicity

(b)

Acute uveitis

(c)

Severe conjunctivitis

(d)

Acute angle closure glaucoma

Your Response :

Correct Answer :

Exp:

Acute angle closure glaucoma (acute ACG)


These are typical features of acute angle closure glaucoma.
Slight corneal edema and mid dilated pupil (instead of fully dilated pupil)
The patient is 45 female. Its more common in females
No history of drug use and atropine has systemic side effects too: flushing of body, fever, tachycardia etc. No
Keratitic precipitates were found so uveitis is unlikely Conjunctivitis: congestion is mainly in fornix, no cells, flare.

(Q.227) The following is true about adult chlamydial conjunctivitis except:


(a)

Caused by D- K serotypes

(b)

Transmission by autoinoculation

(c)

Superior conjunctiva commonly involved

(d)

Essential to identify and treat sexual partner

Your Response :

Correct Answer :

Exp:

Superior conjunctiva is common is false.


In Adult Chlamydia conjunctivitis inferior conjunctiva is commonly involved.
Clinical feature: Mucopurulent discharge, discomfort
Signs: Large follicles esp in inferior fornix Peripheral corneal infiltrates Tender
lymphadenopathy
Inv.

Fluorescent antibody test

ELISA test
PCR
Treatment:
Topical- tetracycline point QID * 6 Wks
Oral

- Azithromycin I gm single dose

http://dbmci.examonair.com/Result/ShowAllQuestionInHtml.aspx?testid=5615

83/111

12/25/2014

Dr Bhatia Medical Coaching Institute:Online Test Platform


- Doxycycline 100mg BD * 2 wks
(Q.228) The predictive factor for retinoblastoma metastasis is:
(a)

Endophytic variety

(b)

Exophytic variety

(c)

Bilateral RB

(d)

Tumour volume > l cu. CM

Your
Response d
:
Correct
D
Answer :
Exp:

Tumour volume> lcu. CM


The predictive factors are:
Tumour volume> 1 cu. CM
Orbital invasion
Optic nerve invasion
Massive choroidal invasion

(Q.229) Familial polyposis coli is associated with which malignancy in eye:


(a)

Congenital hypertrophy of RPE

(b)

Retinoblastoma

(c)

Choroidal melanoma

(d)

Familial exudative vitreo retinopathy (FEVR)

Your Response :

Correct Answer :

Exp:

Congenital hypertrophy of RPE


Congenital hypertrophy of the retinal pigment epithelium (CHRPE) is a benign lesion of the fundus that most
often appears as a flat, pigmented spot.Its prevalence is highly variable and difficult to determine- studies quote
a highly variable prevalence ranging from 0.3-40% of the population.It is more common in females by a ratio of
2:1.
CHRPE can be broken down into three subtypes which include solitary CHRPE (which is most often referred to
with the term CHRPE), grouped CHRPE (bear tracks), and multiple CHRPE.The latter 2 subtypes are often
considered distinct entities, and this module will focus primarily on the solitary type. Typically, CHRPE has no
clinical significance, however the recent development of multiple CHRPE is often associated with the cancer
syndromefamilial adenomatous polyposis(FAP).
CHRPE is the most common extra-colonic manifestation of FAP, occurring in 70-75% of patients.CHRPE lesions in
FAP are atypical in that they are multiple, bilateral, and familial (i.e. the name multiple CHRPE). Studies have
also shown that the lesions differ in both histology and appearance on fluorescein angiography from standard
solitary CHRPE.As such, many consider these lesions to be a different entity altogether, using the term
pigmented ocular fundus lesions of familial adenomatous polyposis or POFLs

(Q.230) True about ECCE is all except?


(a)

Lens removed as single piece within its capsule

(b)

Difficult technique

(c)

A routine procedure for all forms of cataract (except where contra-indicated)

(d)

Dislocated lens is a contra-indication.

Your Response :

Correct Answer :

Exp:

Lens removed as single piece within its capsule


ECCE vs. ICCE
ECCE

ICCE

Lens removal

Nucleus removed out of the


capsule and cortex sucked out

Lens removed as single piece within its


capsule

Posterior capsule
&zonules

Intact

Removed

Incision

Smaller (8 mm)

Larger (10 mm)

Peripheraliridectomy

Not performed

Required to avoidpupillaryblock
glaucoma

Sophisticated equipment Required

Not required

Time taken

More

Less

IOL Implantation

Posterior chamber

Anterior chamber (with its associated


risk of damage to corneal endothelium
leading to PseudophakicBullousKeratopathy)

Expertise required

Difficult technique

Easier to learn

http://dbmci.examonair.com/Result/ShowAllQuestionInHtml.aspx?testid=5615

84/111

12/25/2014

Dr Bhatia Medical Coaching Institute:Online Test Platform


Cost

More

Less

Complications which are


increased

After-Cataract

Vitreousprolapse& loss
CystoidMacular Edema
Endophthalmitis
AphakicGlaucoma
Fibrous & Endothelialingrowth
NeovascularGlaucoma
inProliferativeDiabetic Retinopathy

Compicationswhich are
decreased

All the complications mentioned


for ICCE

After-Cataract

Indications

A routine procedure for all forms Dislocated Lens


of cataract (except where contraSubluxatedLens (>1/3rdzonulesbroken)
indicated)
Chronic Lens InducedUveitis
HypermatureShrunken Cataract with
thick anterior capsule
Intra-lenticularForeign Body when
integrity of posterior capsule is
compromised.

Contraindications

Dislocated lens
Subluxatedlens
(>1/3rdzonulesbroken)

Young Patients (<35 years) who have


strong attachment between lens and
vitreous (Ligament ofWeigert)

(Q.231) True about pediatric cataract is ?


(a)

Cataract is usually discovered in children by their parents in advanced stages when the pupil appears white or the child
becomes severely visually handicapped.

(b)

If macula is deprived of clear image in the first 6-8 weeks of life then it can lead to failure of development of fixation and
causesnystagmus.

(c)

It is recommended that the child be made hyperme-tropicby about 3dioptre, so that as thechild growthe eye becomes
nearlyemmetropic.

(d) All of above


Your Response :

Correct Answer :

Exp:

All of above
Pediatric Cataract
Cataract occurring in a child assumes special importance because of some characteristic differences in the way a
childs developing eye behaves. Following are the main problems to be tackled while managing pediatric
cataract:
Visual Assessment: It is very difficult to assess visual acuity of a pre-verbal child and also young children do not
complain of diminished vision. Therefore, cataract is usually discovered in children by their parents in advanced
stages when the pupil appears white or the child becomes severely visually handicapped.
Vision DeprivationAmblyopia: The young developing visual system of a child needs constant stimulation by a
clear image on the retina in order to develop normally. If visual stimulus is absent or disturbed by poor vision
due to cataract or any other cause then the development of visual system is arrested leading toamblyopia. If
macula is deprived of clear image in the first 6-8 weeks of life then it can lead to failure of development of
fixation and causesnystagmus. Thisproblemsis even worse if only one eye has the problem and the other is
normal.
Therefore, if cataract is detected in a young child it has to be treated urgently and the postoperative visual
rehabilitation also involves treatment ofamblyopia.
Postoperative Inflammation and Fibrosis: The immune system of the child being very active responds violently
to surgery and thereby, leads to severe postoperativeuveitisand fibrosis.This severely marthe results of
operation. To deal with this problem some surgeons advocateParsPlanaLensectomyas the technique of choice
for removal of cataract in children. Butthis technique makeimplantation of IOL difficult.
After-cataract: The epithelialcells of the anterior capsule being very actively dividing a child isvery prone to
develop dense and thick after-cataract. This can be very difficult to treat because the children do not cooperate
for YAG Laser treatment and being thick the laser sometimes fail to penetrate it. Therefore, some surgeons make
an opening in the posterior capsule at the end of operation for cataract (Primary PosteriorCapsulotomy).
IOL Power Calculation: As the eyeball of the child is still growing it poses a problem in calculating the power of
the IOL to be implanted as the later is based on the axial length of the globe. IfaIOL ofemmetropicpower is
implanted this leads to myopia as the child grows up. Therefore, it is recommended that the child be
madehypermetropicby about 3dioptre, so that as thechild growthe eye becomes nearlyemmetropic.

(Q.232) Antithrombin III levels in disseminated intravascular coagulation:


(a)

Are decreased

(b)

Are increased

(c)

Remain same

(d)

Are variable

Your Response :

Correct Answer :

Exp:

Are decreased
Disseminated intravascular' coagulation.
- Coagulation is usually confined to a localized area by the combination of blood flow and circulating inhibitors of
coagulation, especially antithrombin III. If the stimulus to coagulation is too great, these control mechanisms can
be

http://dbmci.examonair.com/Result/ShowAllQuestionInHtml.aspx?testid=5615

85/111

12/25/2014

Dr Bhatia Medical Coaching Institute:Online Test Platform


overwhelmed, leading to the syndrome of disseminated intravascular coagulation (DIC).
Diagnosis :
- Antithrombin III levels may be low in DIC due to the combination of increased consumption and decreased
synthesis.
- Hypofibrinogenemia, thrombocytopenia, fibrin degradation products, and prolonged prothrombin time.
(Q.233) The role of angiogenesis-converting enzyme inhibitors in protecting against deterioration of renal function in diabetic nephropathy

is most closely related to their ability to


(a)

Reduce renal plasma flow

(b)

Prevent enzymatic glycosylation in the glomerular capillaries

(c)

Decrease the effect of angiotensin II on efferent arteriolar tone

(d)

Increase Transcapillary hydrostatic pressure in the glomerulii

Your Response :

Correct Answer :

Exp:

Angiotensin-converting enzyme (ACE) inhibitors block the conversion of angiotensin I into angiotensin II.
Angiotensin II is a powerful vasoconstrictor that preferentially affects the vascular tone of the efferent arterioles.
Removal of the tonic vasoconstrictor effect of angiotensin II on the efferent arterioles lowers the glomerular
intercapillary pressure, while preserving renal plasma flow.
Glomerular efferent arteriolar tone is increased in diabetes. Hyaline arteriolosclerosis, the small vessel disease of
diabetes mellitus, is commonly present in both the afferent and efferent arterioles. Increased efferent arteriolar
tone results in increased transcapillary hydrostatic pressure, which interferes with the functional integrity of the
glomerular capillary wall. In addition, nonenzymatic glycosylation contributes to the permanent damage in the
glomerular capillary walls.

(Q.234) All are true about Pallid spells except?


(a)

Can be initiated by painful experience

(b)

Tachycardia

(c)

Normal interictal EEG

(d)

Unconsciousness

Your Response :
Correct Answer :

Exp:

Tachycardia
Ref. Nelson, 18th edition, page 2477
BREATH-HOLDING SPELLS.
A breath-holding spell can be a frightening experience for parents because the infant becomes lifeless and
unresponsive owing to cerebral anoxia at the height of the attack. There are two major types of breath-holding
spells: the more common cyanotic form and the pallid form.
Cyanotic Spells.
A cyanotic breath-holding spell is usually predictable and is always provoked by upsetting or scolding an infant.
The episode is heralded by a brief, shrill cry followed by forced expiration and apnea. There is rapid onset of
generalized cyanosis and a loss of consciousness that may be associated with repeated generalized clonic jerks,
opisthotonos, and bradycardia. Results of an interictal electroencephalogram (EEG) are normal. A breath-holding
spell can occur repeatedly within a few hours or it can recur sporadically, but it is always stereotyped. Breathholding spells are rare before 6 mo of age, peak at about 2 yr of age, and abate by 5 yr of age. The management
of breath-holding spells concentrates on the support and reassurance of the parents. Some parents feel that
whatever the physician recommends, they must splash cold water on the face, turn the child upside down, or
initiate mouth-to-mouth resuscitation and even cardiopulmonary resuscitation. A thorough examination
followed by an explanation of the mechanism of breath-holding spells is reassuring for most parents. The
counseling session should emphasize the need for both parents to be consistent and not reinforce the child's
behavior after the child recovers from the spell. This may be accomplished by placing the child safely in bed and
by refusing to cuddle, play, or hold the child for a given period of time until recovery is complete.
Pallid Spells
These spells are much less common than cyanotic breath-holding spells, but they share several characteristics.
Pallid spells are typically initiated by a painful experience, such as falling and striking the head or a sudden
startle. The child stops breathing, rapidly loses consciousness, becomes pale and hypotonic and may have a tonic
seizure. Bradycardia with periods of asystole of>2sec may be recorded. The interictal EEG is normal. Pallid spells
can in some cases be induced spontaneously in the laboratory by ocular compression that produces the
oculocardiac reflex, afferent stimulation of the trigeminal nerve, and efferent inhibition of the heart by way of
the vagus nerve. This procedure should not be attempted by an inexperienced physician, and appropriate
resuscitation equipment should be readily available. Most children respond to conservative measures as outlined
for cyanotic spells, but a trial of an anticholinergic, oral atropine sulfate 0.01 mg/kg/24hr in divided doses with a
maximum daily dose of 0.4 mg, which increases the heart rate by blocking the vagus nerve, may be considered in
refractory cases.

(Q.235) What is Retinal dialysis?


(a)

Retinal tear

(b)

Retinal infarct

(c)

Retinal hemorrhage

(d)

Retinal changes after hemodialysis

Your Response :

Correct Answer :

Exp:

Retinal tear

http://dbmci.examonair.com/Result/ShowAllQuestionInHtml.aspx?testid=5615

86/111

12/25/2014

Dr Bhatia Medical Coaching Institute:Online Test Platform


Ref: Baily & Love 25th edition, page 651
Retinal dialysis is a retinal tear at the ora serrata. It usually results from trauma, although some tears occur
spontaneously. If the trauma is intense there may also be a retinal break at the optic disc but the most frequent
location is in the lower temporal quadrant. The condition is most typically asymptomatic. However, as it often
gives rise to retinal detachment, the patient may report some of the symptoms associated with the latter.
Perimetry and binocular indirect ophthalmoscopy are essential in the examination of this condition. In some
cases the tear does not progress, but because of the risk of retinal detachment, patients must be referred to a
retinal specialist.
(Q.236) Cytokine release syndrome is seen with:
(a)

Mycophenolate Mofetil

(b)

Muromonab CD3

(c)

Antithymocyte Globulin

(d)

Methotrexate

Your Response :

Correct Answer :

Exp:

Muromonab CD3
Muromonab CD3 is a murine monoclonal antibody against CD3 glycoprotein on helper T cells initial dose of this
drug can produce cytokine release syndrome with flu like symptoms. This drug is used in induction therapy for
organ transplantation.
As immune-based therapies for cancer become potent, more effective, and more widely available, optimal
management of their unique toxicities becomes increasingly important. Cytokine release syndrome (CRS) is a
potentially life-threatening toxicity that has been observed following administration of natural and bispecific
antibodies and, more recently, following adoptive T-cell therapies for cancer. CRS is associated with elevated
circulating levels of several cytokines including interleukin (IL)-6 and interferon , and uncontrolled studies
demonstrate that immunosuppression using tocilizumab, an anti-IL-6 receptor antibody, with or without
corticosteroids, can reverse the syndrome. However, because early and aggressive immunosuppression could
limit the efficacy of the immunotherapy, current approaches seek to limit administration of immunosuppressive
therapy to patients at risk for life-threatening consequences of the syndrome.
Risks associated with cancer immunotherapy can be broadly classified into autoimmune toxicity and cytokineassociated toxicity. Autoimmune toxicity, so-called on target, off-tumor toxicity, results from antigen-specific
attack on host tissues when the targeted tumor associated antigen is expressed on nonmalignant tissue.
Autoimmune toxicity occurs not uncommonly after treatment with checkpoint inhibitorsand has resulted in fatal
toxicities after infusion of genetically engineered T cells targeting MAGE-A3.Cytokine-associated toxicity, also
known as cytokine release syndrome (CRS), is a nonantigen-specific toxicity that occurs as a result of high-level
immune activation. The magnitude of immune activation typically required to mediate clinical benefit using
modern immunotherapies exceeds levels of immune activation that occurring in more natural settings, and as
immune-based therapies have become more potent, this syndrome is becoming increasingly recognized.
CRS revised grading system
Grade

Toxicity

Grade 1

Symptoms are not life threatening and require symptomatic treatment only, eg, fever,
nausea, fatigue, headache, myalgias, malaise

Grade 2

Symptoms require and respond to moderate intervention


Oxygen requirement <40% or
Hypotension responsive to fluids or low dose2of one vasopressor or
Grade 2 organ toxicity

Grade 3

Symptoms require and respond to aggressive intervention


Oxygen requirement 40% or
Hypotension requiring high dose*or multiple vasopressors or
Grade 3 organ toxicity or grade 4 transaminitis

Grade 4

Life-threatening symptoms
Requirement for ventilator support or
Grade 4 organ toxicity (excluding transaminitis)

Grade 5

Death

Grades 2-4 refer to CTCAE v4.0 grading.

(Q.237) True about Cilomilast are all except:


(a)

Phosphodiesterase 4 inhibitor

(b)

Used in bronchial asthma

(c)

Used in allergic rhinitis

(d)

Safe when injected intravenously

Your Response :

Correct Answer :

Exp:

Safe when injected intravenously


Cilomilast is a new drug found effective in the treatment of allergic conditions. It is an orally used preparation
hence can be given only in mild symptoms or for prevention.
Cilomilast is a drug which was developed for the treatment of respiratory disorders such asasthmaandChronic
Obstructive Pulmonary Disease(COPD). It is orally active and acts as a selectivePhosphodiesterase-4 inhibitor.
Phosphodiesterase(PDE) inhibitors, such astheophylline, have been used to treatChronic Obstructive Pulmonary
Disease (COPD)for centuries; however, the clinical benefits of these agents have never been shown to out-weigh
the risks of their numerous adverse effects. Four clinical trials were identified evaluating the efficacy of

http://dbmci.examonair.com/Result/ShowAllQuestionInHtml.aspx?testid=5615

87/111

12/25/2014

Dr Bhatia Medical Coaching Institute:Online Test Platform


cilomilast, the usual randomized, double-blind, and placebo-controlled protocols were used. It showed
reasonable efficacy for treating COPD, but side effects were problematic and it is unclear whether cilomalast will
be marketed, or merely used in the development of newer drugs.
Cilomilast is a second-generation PDE4 inhibitor withantiinflammatoryeffects that
targetbronchoconstriction,mucus hypersecretion, and airway remodeling associated with COP(D)
PDE-IV inhibition
The phosphodiesterases are enzymes responsible for hydrolysis of cyclic adenosine monophosphate (cAMP), a
naturally occurring substance that mediates suppression of inflammatory cell activation. PDE-IV is the
predominant phosphodiesterase isoenzyme in respiratory tissues and is therefore a suitable target for therapy of
inflammatory airways diseases such as COP(D)
By inhibiting PDE-IV, drugs increase levels of cAMP and reduce inflammatory cell activation in the respiratory
tract. Results from pre-clinical investigations with Ariflo suggest it may:
Reduce bronchoconstrction
Reduce inflammation and subsequent damage to respiraotry tissues
Reduce cough and dyspnoea (breathlessness)
Selective PDE-IV inhibitors, have potential to not only target symptoms of the disease but to also ameliorate the
underlying components of airway obstruction and inflammation. Thus, they represent a completely new
approach to the treatment of COP(D)
(Q.238) A 30 year male having received 2 months therapy as DOTS for pulmonary tuberculosis discontinues on improvement in symptoms. A

few months later he returns with increasing cough and dyspnea. Which treatment category he should be put on now:
(a)

One

(b)

Two

(c)

Three
Four

(d)
Your Response :

Correct Answer :

Exp:

Two
Following are the categories in DOTS to decide the anti tubercular regimen:
Cat.1 new smear + new smear -with extensive involvement; new severe extrapulmonary disease.
Cat.2 smear+ relapse treatment failure; treatment after interruption
Cat.3 new smear- other than category 1 new less severe extra pulmonary disease.
Cat.4 not used these days.

(Q.239) Ramu 53 yrs old, Hindu male, presented to the emergency, complaining severe crushing pain in epigastrium and breathlessness for

the past 20 minutes. On examination, pulse and BP are normal; ECG shows ST segment elevation in leads V2 to V6. All of the
following should be given in the management of the patient, except:
(a)

Aspirin

(b)

LV Beta blockers

(c)

Tissue plasminogen activator

(d)

Hydrocortisone

Your Response :

Correct Answer :

Exp:

Hydrocortisone
The history and examination of the patient suggest that, he is suffering from an attack of acute myocardial
infarction.
Aspirin: Aspirin is essential in the management of a patient of suspected STEM.
I. V blockers: they are useful in a patient of AMI
Due to the following reasons:
1. They are helpful in reducing pain
2. They reduce in hospital mortality, particularly in the high-risk patients.
TP A: TPA helps in limitation of the infarct size by early restoration of blood flow to the ischaemic areas in pts
with STEM!.
Glucocorticoids: glucocorticoids and other NSAIDS except aspirin should be avoided in the setting of STEM. They
can impair infarct healing and increase the risk of myocardial rupture and may also lead to larger infarct
Scar. In addition, they cause, increased coronary vascular resistance, thereby, potentially reducing flow to
ischaemic myocardium.

(Q.240) Which of the following is the mechanism of action of Low-Molecular-Weight Heparin?


(a)

Factor Xa inhibition, Thrombin inhibition

(b)

Factor Xa inhibition

(c)

Antithrombin inhibitor

(d)

Thrombin inhibition

Your Response : b
Correct Answer : A
Exp:

Factor Xa inhibition, Thrombin inhibition


Low-Molecular-Weight Heparin

http://dbmci.examonair.com/Result/ShowAllQuestionInHtml.aspx?testid=5615

88/111

12/25/2014

Dr Bhatia Medical Coaching Institute:Online Test Platform


Mechanism of action: Factor Xa inhibition, Thrombin inhibition
Example: Enoxaparin, Tinzaparin, Dalteparin
(Q.241) Following are the indications for aortic valve replacement in a patient of Aortic stenosis, except:
(a)

Severe AS who are symptomatic

(b)

Patients exhibiting LV dysfunction

(c)

Asymptomatic patients with expanding post stenotic aortic root

(d)

Asymptomatic patients with ejection fraction> 50%

Your Response :

Correct Answer :

Exp:

Asymptomatic patients with ejection fraction >50%


Aortic valve replacement is indicated in patients with:
(1) Severe AS (valve area <1.0 cm2 or 0.6 cm2/m2 body area) Who are symptomatic.
Those who exhibit left ventricular dysfunction
Those with expanding post stenotic aortic root dilation even if they are asymptomatic.
In most cases it is prudent to postpone operation in patients with severe calcific aortic stenosis, who are
asymptomatic and have ejection fraction>50%.
The risk of surgical mortality, exceeds that of sudden death in an asymptomatic patient.
Asymptomatic patients with ejection fraction >50%
Aortic valve replacement is indicated in patients with:
(1) Severe AS (valve area <1.0 cm2 or 0.6 cm2/m2 body area) Who are symptomatic.
Those who exhibit left ventricular dysfunction
Those with expanding post stenotic aortic root dilation even if they are asymptomatic.
In most cases it is prudent to postpone operation in patients with severe calcific aortic stenosis, who are
asymptomatic and have ejection fraction>50%.
The risk of surgical mortality, exceeds that of sudden death in an asymptomatic patient.

(Q.242) Normally, fractional excretion of sodium in prerenal ARF is <1.0%. It can be > 1.0% in all of the following conditions, except:
(a)

Patient receiving diuretics

(b)

Patient with pre-existing chronic renal failure, complicated by salt Wasting.

(c)

Patient with adrenal insufficiency

(d)

Patient with pre-renal ARF due to blood loss

Your Response :

Correct Answer :

Exp:

Patient with pre-renal ARF due to blood loss


Many caveats apply when interpreting biochemical renal failure Indices.
Normally, fractional excretion of sodium for pre-renal ARF is <1.0%, but it may be > 1.0% in the following
conditions:
(1) Patients receiving diuretics or having bicarbonaturia
(2) Patients with pre-existing chronic renal failure complicated by Salt wasting
(3) Patients with adrenal insufficiency Likewise, fractional excretion of sodium <1 % may be seen in 15% cases of
non-oliguric ischaemic or nephrotoxic ARF. Remember, fractional excretion of sodium, is the most useful index
to differentiate between pre-renal and ischaemic or nephrotoxic intrinsic renal ARF.

(Q.243) On ECG a convex elevation of J-point indicates which of the following?


(a)

Hypocalcemia

(b)

Hypothermia

(c)

Digitalis toxicity

(d)

Hypokalemia

Your Response :

Correct Answer :

Exp:

Hypothermia
A convex elevation of J-point on E.(C)G. is called an OSBORN WAVE. It is seen in cases of
hypothermia.
This is an extremely high yield topic having atleast 1 question in every exam.
Here is a list of almost all important conditions causing specific ECG abnormalities.
ECG ABNORMALITIES :
DIAGNOSTIC ECG FEATURES
QT shortening(Earliest feature)

CONDITION
Therapeutic Digitalization

Increased PR interval
Slowing of heart rate
Decreased amplitude/inverted T wave
ST depression
Prolonged PR interval,

Digitalis toxicity

VPCs,VT,VF,AV block

http://dbmci.examonair.com/Result/ShowAllQuestionInHtml.aspx?testid=5615

89/111

12/25/2014

Dr Bhatia Medical Coaching Institute:Online Test Platform


Sino-atrial block,
Ventricular bigeminy/trigeminy
Flattening/inversion of T wave

Early hypokalemia

ST depression
Prominent U wave
Prolonged QU interval
Prolonged PR

Severe hypokalemia

Decreased voltage and widening of QRS


Increased risk of ventricular arrhythmias
Rarely SA block
Peaked T waves

Hyperkalemia

Loss of P wave
SINE WAVE PATTERN
Widening of QRS and prolonged PR
Sinus arrest
Cardiac arrest-asystole
Ventricular fibrillation
Bradycardia

Hypercalcemia

AV block
Short QT
Prolonged QT

Hypocalcemia

Widespread ST elevation with upward concavity STAGE I Acute Pericarditis


involving 2-3 standard limb leads and V2-V6
Reciprocal depression only in aVR and sometimes
in V1
PR depression s/o Atrial involvement
ST segment returns to normal after several days STAGE II Acute Pericarditis
Only then or even after it T wave inversion occurs STAGE III Acute Pericarditis
Weeks to months after it-ECG returns to normal STAGE IV Acute Pericarditis
J wave or Osborn wave

Hypothermia

High voltage tall QRS with short PR interval

Pompes disease

CVA-T pattern :

Intracranial blleds esp SAH.

Marked QT prolongation with deep wide T wave


inversions
Total Electrical Alternans

Pericardial Effusion with cardiac tamponade


d/t swinging motion of heart in the pericardial
cavity at a frequency exactly half the hrt rate

Replarisation Alternans

Sign of electrical instability and may suggest


impending ventricular tachyarrythmias

Peaked T waves(d/t hyperkalemia)

ECG Triad of chroniv renal failure

Long QT d/t St segment


lengthening(hypocalcemia)
Left ventricular hypertrophy(d/t systemic
hypertension)
Right axis deviation

Tetralogy of Fallot

Right ventricular hypertrophy with inverted T


waves and P Pulmonale
Normal P wave with short PR

WPW syndrome

A Slurred thickened initial upstroke of QRS


DELTA wave
Main QRS deflection
Secondary ST and T wave changes
T wave suppression/inversion(most
characteristic change)

Imipramine (TCA) toxicity

Prolonged QRS >100ms


R wave greater than S wave

http://dbmci.examonair.com/Result/ShowAllQuestionInHtml.aspx?testid=5615

90/111

12/25/2014

Dr Bhatia Medical Coaching Institute:Online Test Platform


Polymorphic VT preceeded by marked QT
prolongation >0.6 sec

Torsades-de-pointes

Bizzare and multiform QRS


Transient ST depression

Classical angina

Transient St elevation without Q wave

Non-infarction transmural
ischemia(Prinzmetals angina)

Paradoxical T wave normalization


ST depression or T wave inversion without Q
wave

NSTEMI

ST elevation

Hyperacute phase MI

Tall widened T waves


Pathological Q wave

Fully evolved MI

ST elevation
Inverted symmetric T wave
Pathological Q wave only

Old infarct

(Q.244) Iridodonesisis seen in which condition?


(a)

Anterior uveitis

(b)

Posterior uveitis

(c)

Lens subluxation

(d)

Acute glaucoma

Your Response :

Correct Answer :

Exp:

Lens subluxation
Ref: Baily & Love 25th edition, page 650
Iridodonesisis the vibration or agitated motion of the iris (wobbles) with eye movement.This may be caused
bylens subluxation,the incomplete or partial dislocation of the lens; or byaphakia, the absence of a lens.

(Q.245) A 25-year-old male cigarette smoker has a history of respiratory infections and has also been found to have hematuria. A high value

for diffusing capacity is noted during pulmonary function testing. This finding is consistent with which of the following disorders?
(a)

Cystic fibrosis

(b)

Emphysema

(c)

Intrapulmonary hemorrhage

(d)

Anemia

Your Response :

Correct Answer :

Exp:

Intrapulmonary hemorrhage
Carbon monoxide (CO) diffusing capacity provides an estimate of the rate at which oxygen moves by diffusion
from alveolar gas to combine with hemoglobin in the red blood cells. It is interpreted as an index of the surface
area engaged in alveolar-capillary diffusion. Measurement of diffusing capacity of the lung is done by having the
person inspire a low concentration of carbon monoxide. The rate of uptake of the gas by the blood is calculated
from the difference between the inspired and expired concentrations. The test can be performed during a single
10-second breath holding or during 1 minute of steady-state breathing. The diffusing capacity is defined as the
amount of carbon monoxide transferred per minute per millimeter of mercury of driving pressure and correlates
with oxygen transport from the alveolus into the capillaries. Primary parenchymal disorders, anemia, and
removal of lung tissue decrease the diffusing capacity. Conversely, polycythemia, congestive heart failure, and
intrapulmonary hemorrhage tend to increase the value for diffusing capacity. In this patient, the possibility of
Goodpasture syndrome should be considered.

(Q.246) The most specific blood test for vitamin-D deficiency :


(a)

Serum calcium

(b)

Serum 1- hydroxyl cholecalciferol

(c)

Serum 25 - hydroxycholecalciferol

(d)

Serum 1, 25 - dihydroxycholecalciferol

Your Response :

Correct Answer :

Exp:

25 - hydroxycholecalciferol
The most specific screening test for Vitamin D deficiency in otherwise healthy individual is a serum 25 (OH) D(25hydroxy cholecalciferol) with levels < 15 ng/ML are associated with increasing Parathyroid hormone (PTH) levels
and lower bone density. Rise in PTH levels maintains plasma calcium at the expense of skeleton. Similarly PTH is
a major stimulus for the renal vitamin D hydroxylase, there is increased synthesis of the active hormone. 1, 25 dihydroxycholecaliferol. Therefore measurements of 1, 25 (OH)2D, therefore do not provide an accurate index of
vitamin D store and should not be used to diagnose vitamin D deficiency in patients with normal renal function.
I-hydroxycholecalciferol {l(OH)D} is a synthetic derivative of vitamin D used in treatment of renal
osteodystrophy.

http://dbmci.examonair.com/Result/ShowAllQuestionInHtml.aspx?testid=5615

91/111

12/25/2014

Dr Bhatia Medical Coaching Institute:Online Test Platform


(Q.247) A 60 year old man presented to emergency room with altered sensorium and left sided hemiparesis. He is having urinary

incontinence. Best way is to deal with incontinence is use of :


(a)

Foley's catheter

(b)

Caude latex catheter

(c)

Condom catheter

(d)

Incontinence pad

Your Response :

Correct Answer :

Exp:

Condom catheter
Indwelling catheters should not be used for routine management of urinary incontinence. If feasible, less
invasive measures such as incontinence pads (usually in females), intermittent catheterization or penile sheath
(condom) catheterization should be used. Caude catheters has a semi rigid curved tip that facilitate placement in
patients with prostatic enlargement.

(Q.248) A 35 year-old alcoholic male is admitted for nausea, vomiting, and abdominal pain that radiates to the back. The laboratory value

that suggests a poor prognosis in this patient is:


(a)

Elevated serum lipase

(b)

Elevated serum amylase

(c)

Leukocytosis of 20,000/mm3

(d)

Diastolic blood pressure greater than 90 mmHg

Your Response :

Correct Answer :

Exp:

Leukocytosis of 20,000/mm3
The RANSON CRITERIA are used to determine prognosis in acute pancreatitis.
FACTORS THAT ADVERSELY AFFECT SURVIVAL INCLUDE
AT PRESENTATION
Age greater than 55 years
Leukocytosis greater than 16,000/mm3
Glucose greater than 200 mg/dL
LDH greater than 400 IU
AST greater than 250 IU/L
AFTER THE INITIAL 48 H
A fall in hematocrit
Hypocalcemia
Hypoxemia
An increase in BUN
Hypoalbuminemia
Hypotension with systolic BP less than 90 mmHg is also a poor prognostic sign; diastolic hypertension is not
correlated with prognosis.

(Q.249) A 43-year-old female presents with hypertension, edema, hyperlipidemia, and a deep venous thrombosis in her left leg. Which of

the following is not necessary to diagnose the nephrotic syndrome?


(a)

Edema

(b)

Hypertension

(c)

24-hr urine albumin 3g

(d)

Hyperlipidemia

Your Response :

Correct Answer :

Exp:

Hypertension
While hypertension may occur in diseases causing the nephrotic syndrome, its presence is not necessary for the
diagnosis of this syndrome. Renal loss and catabolism of albumin lead to hypoalbuminemia and edema.
Increased hepatic synthesis of lipoproteins leads to markedly elevated lipid levels.

(Q.250) Hemoptysis is said to be massive when it exceeds what amount in 24 hours ?


(a)

400Ml

(b)

500mL

(c)

600mL
800mL

(d)
Your Response :

Correct Answer :

Exp:

Massive hemoptysis is generally defined as expectoration of over 600 mL of blood within a 24-hour period. It is a
medical emergency associated with a mortality rate of 30 to 50%. Most clinicians would agree that losing over a
liter of blood via the airway within 1 day is significant, yet use of an absolute volume criterion presents
difficulties. First, it is difficult for the patient or caregivers to quantify the volume of blood being lost. Second,
and most relevant, the rate of bleeding necessary to incite respiratory compromise is highly dependent on the
individual's prior respiratory status. For example, the loss of 100 mL of blood over 24 hours in a 40-year-old male

http://dbmci.examonair.com/Result/ShowAllQuestionInHtml.aspx?testid=5615

92/111

12/25/2014

Dr Bhatia Medical Coaching Institute:Online Test Platform


with normal pulmonary function would be of little immediate consequence, because his normal cough would
ensure his ability to clear the blood and secretions. In contrast, the same amount of bleeding in a 69-year-old
male with severe COPD, chronic bronchitis, and an FEV1 of 1.1 L may be life-threatening.
Anatomy
The lungs have two sources of blood supply: the pulmonary and bronchial arterial systems. The pulmonary
system is a high-compliance, low-pressure system, and the walls of the pulmonary arteries are very thin and
delicate. The bronchial arteries, part of the systemic circulation, have systemic pressures and thick walls; most
branches originate from the proximal thoracic aorta. Most cases of massive hemoptysis involve bleeding from
the bronchial artery circulation or from the pulmonary circulation pathologically exposed to the high pressures
of the bronchial circulation. In many cases of hemoptysis, particularly those due to inflammatory disorders, the
bronchial arterial tree becomes hyperplastic and tortuous. The systemic pressures within these arteries,
combined with a disease process within the airway and erosion, lead to bleeding.
Causes
Significant hemoptysis has many causes. Most are secondary to inflammatory processes. An acute necrotizing
pneumonic infection can lead to destruction and erosion of vascular structures and bleeding. Chronic
inflammatory disorders (i.e., bronchiectasis, cystic fibrosis, tuberculosis, and others) lead to localized bronchial
arterial proliferation, and with erosion, bleeding of these hypervascular areas occurs.
(Q.251) Which of the following is a risk factor for carcinoma of the bladder?
(a)

Exposure to alcohol intake

(b)

Use of cyclophosphamide

(c)

Positive family history

(d)

Infestation with Schistosoma mansoni

Your Response :

Correct Answer :

Exp:

Use of cyclophosphamide
Carcinoma of the bladder typically affects older men. Transitional cell carcinoma is the most common histologic
subtype and is associated with a more favorable prognosis than is adenocarcinoma or squamous carcinoma.
Squamous carcinomas occur more frequently in Egypt and are associated with S. haematobium and not S.
mansoni, which typically causes an infection of the intestines or biliary tract. Risk factors for carcinoma of the
bladder include exposure to the aromatic amines, which result from cigarette smoke or products of the dye,
rubber, and chemical industries, but it is not associated with positive family bistory or a prior diagnosis of renal
carcinoma. Chronic bladder irritation, such as that produced by the metabolites of cyclophosphamide or
ifosfamide as well as by recurrent bladder stones or infections, also leads to a higher incidence of carcinoma of
the bladder.

(Q.252) In acute renal failure, dietary protein should be restricted in which of the following?
(a)

All patients.

(b)

All patients with BUN>100.

(c)

All patients with creatinine> 10.

(d)

If azotemia is advanced and dialysis is not an option.

Your Response :

Correct Answer :

Exp:

If azotemia is advanced and dialysis is not an option


Years before dialysis was routinely available, it was well established that protein restriction (prescribed or selfimposed) could alleviate some of the symptoms of uremia; unfortunately, prolonged protein restriction led to
the development of malnutrition and its associated complications. In the setting of chronic renal failure, a
number of clinical studies have suggested that modest protein restriction may slow the rate of progression of
renal failure, particularly in patients with glomerular disease and daily protein excretion rates> 1 g/d. There are
insufficient data in the setting of acute renal failure to adequately assess the importance of protein intake.
However, in view of the hypercatabolism that accompanies many cases of acute renal failure, most practitioners
provide adequate protein to patients (e.g., 1.0 to 1.2 g protein per kg per day) and provide dialysis if uremia
ensues. There are no set laboratory "cutoffs" (BUN> 1 00) that indicate the need for dialysis.

(Q.253) Why do patients with the carcinoid syndrome develop pellagra (loss of appetite, generalized weakness, abdominal pain, vomiting,

stomatitis and bright red glossitis, and a skin rash that is pigmented and scaling in sunlight- exposed areas)?
(a)

Failure to elaborate serotonin

(b)

Thiamine deficiency

(c)

Niacin deficiency

(d)

Pyridoxine deficiency

Your Response :

Correct Answer :

Exp:

Niacin deficiency
The amino acid tryptophan is converted to the niacin derivatives nicotinic acid and nicotinamide, coenzymes
required for numerous oxidative and reductive enzymes. Niacin deficiency causes pellagra, which is a
complicated syndrome involving loss of appetite, weakness, irritability, abdominal pain, bright red tongue and
stomatitis, and a skin rash characterized by pigmentation and scaling, particularly in areas exposed to sunlight.
Pellagra can be seen in those whose diet consists primarily of corn in parts of China, Africa, and India; in chronic
alcoholics; in those with congenital defects of tryptophan absorption; and in patients with carcinoid syndrome,
in which there is an increased conversion of tryptophan into serotonin.

(Q.254) Spur cell anemia is a feature of which disorder?

http://dbmci.examonair.com/Result/ShowAllQuestionInHtml.aspx?testid=5615

93/111

12/25/2014

Dr Bhatia Medical Coaching Institute:Online Test Platform


(a)

Liver disease

(b)

Renal failure

(c)

Myelofibrosis

(d)

Acute blood loss

Your Response :

Correct Answer :

Exp:

Liver disease
Spur cells, or acanthocytes, are large erythrocytes covered with spikelike projections that vary in width, length,
and distribution (see image below). Spur cells can be encountered in acquired or inherited disorders.
Spur cells are characterized by diminished deformability, which is responsible for their entrapment and
destruction in the spleen.
Historically, spur cell anemia has been described with advanced alcoholic liver cirrhosis, but it can also be
observed in other severe liver diseases.The inherited disorders associated with significant acanthocytosis are
characterized by an association with neuromuscular disorders. These diseases are presented together in this
article because of the common hematologic feature of peripheral blood acanthocytosis
Acquired acanthocytosis is associated with advanced liver disease regardless of the primary cause. Although
alcohol abuse is the most common cause of chronic liver disease in Western societies, other entities have been
recognized, including nonalcoholic steatohepatitis (NASH) that may progress to cirrhosis.[2]Anorexia nervosa,
hypothyroidism, and myelodysplasia are rare causes of this disorder.

Neuroacanthocytosis is the term used for acanthocytosis associated with inherited disorders. Autosomalrecessive disorders, abetalipoproteinemia/aprebetalipoproteinemia (chromosome 2), chorea-acanthocytosis
syndrome (band 9q21), and the X-linked McLeod phenotype are among the conditions linked with
neuroacanthocytosis.
Formation of acanthocytes
Most acanthocytic disorders are associated with acquired abnormalities of the outer leaflet of the lipid bilayer.
However, some rare conditions have normal lipids and abnormal membrane proteins.
In severe liver disease, free cholesterol in red blood cells equilibrates with abnormal lipoproteins containing a
high free cholesterol-to-phospholipid ratio, resulting in the preferential expansion of the outer leaflet and the
development of the spur cell shape.
A decrease occurs in polyunsaturated versus saturated and monounsaturated fatty acid content in red blood
cells of patients with cirrhosis. This abnormality is more pronounced in patients with spur cell anemia, resulting
in the alteration of the red blood cell shape and a decrease of the cells fluidity.
An increase in the proteolytic activity of the erythrocyte membrane is also reported in spur cell anemia. The
significance and role of this abnormality in changing the shape of the red blood cell and in hemolysis are
unknown.[8]
The plasma of some patients exhibits decreased activity of lecithin cholesterol acyltransferase, resulting in
increased free cholesterol in the outer layer of the red blood cell membrane as a direct consequence of its
increased concentration in the plasma. After acquiring these abnormalities in the plasma, the red blood cells
undergo a remodeling process in the spleen, which gives them the spheroidal shape with longer and more
irregular projections.
Chorea-acanthocytosis
Alteration of band 3, the anion exchange protein, is thought to play a role in the formation of acanthocytes in
chorea-acanthocytosis.[9]According to this hypothesis, the red blood cell shape is controlled by the ratio of the
outward-facing (band 3o) and inward-facing (band 3i) conformations of band 3. Depending on this ratio, there
will be contraction (leading to echinocytosis) or relaxation (leading to stomatocytosis) of the membrane
skeleton.
Abetalipoproteinemia
In abetalipoproteinemia, B-apoproteincontaining lipoproteins (chylomicrons, very low-density lipoproteins
[VLDL], low-density lipoproteins [LDL]) are nearly absent in the plasma. Plasma cholesterol and phospholipids are
decreased, with a relative increase of sphingomyelin at the expanse of lecithin. At equilibrium, the
sphingomyelin concentration in the outer leaflet increases, resulting in its expansion and acanthocytosis.
McLeod phenotype
The expression of the Kell antigen (the product of a single gene on band 7q23) on red blood cells, white blood
cells, and monocytes is under the control of the Kx antigen encoded for by theXKgene on band Xp21.[10]Both

antigens are transmembrane proteins bound by a single disulfide bond. In the McLeod phenotype, theXKgene is
deleted and the Kell antigen cannot be expressed, whereas in the Kell null phenotype, the Kell antigen is missing
and the Kx antigen is present at a normal level. The Kell null phenotype is not associated with hematologic
disorders.
The close proximity on the short arm of band Xp21 of the genes responsible for chronic granulomatous disease
(CGD) of childhood, retinitis pigmentosa (RP), and Duchenne muscular dystrophy (DMD) explains the variable
association of the McLeod phenotype with these diseases. Red blood cells from patients with choreaacanthocytosis syndrome and McLeod phenotype do not show measurable abnormalities of the lipid bilayer
Focal membrane skeleton heterogeneity has been described as characterized by decreased compactness of the
filamentous meshwork in the areas underlying the spikes. This focal weakness allows limited detachment of the
lipid bilayer that does not result in membrane loss. The nature of the membrane skeleton abnormality is not
known.

(Q.255) Uses of TRUS in prostatic evaluation ?


(a)

Obtaining prostate biopsies

http://dbmci.examonair.com/Result/ShowAllQuestionInHtml.aspx?testid=5615

94/111

12/25/2014

Dr Bhatia Medical Coaching Institute:Online Test Platform


(b)

Local staging

(c)

Estimation of prostate volume

(d)

All of above

Your Response :

Correct Answer :

Exp:

All of above
REF SMITHS UROLOGY 17th Ed Chapter22
1. TRUSTRUS is useful in performing prostatic biopsies and in providing some useful local staging information if
cancer is detected. Almost all prostate needle biopsies are performed under TRUS guidance. This allows uniform
spatial separation and sampling of the regions of the prostate and also makes lesion-directed biopsies possible. If
visible, Carcinoma Prostate tends to appear as a hypoechoic lesion in the peripheral zone.
TRUS provides more accurate local staging than does DRE. The sonographic criteria for extracapsular extension
are bulging of the prostate contour or angulated appearance of the lateral margin. The criteria for seminal
vesicle invasion are a posterior bulge at the base of the seminal vesicle or asymmetry in echogenicity of the
seminal vesicle associated with hypoechoic areas at the base of the prostate. TRUS also enables measurement of
the prostate volume, which is needed in the calculation of PSA density.
Typically, a prostate ellipsoid formula is used: (/6) (anterior- posterior diameter) (transverse diameter)
(sagittal diameter). TRUS is also used in the performance of cryosurgery and brachytherapy (see below). Color
or power Doppler TRUS assesses blood flow through prostatic vessels. As cancers may have increased
vascularity, such technology may improve the sensitivity and specificity of U.S. imaging. 3D color Doppler permits
a three-dimensional image to be constructed from a series of 2D images by a computer algorithm. Use of a
microbubble, intravenous contrast agent may also improve visualization of cancers.

(Q.256) The LEAST common site for diverticular disease is the


(a)

Upper esophagus

(b)

Mid esophagus

(c)

Small intestine

(d)

Rectum

Your Response :

Correct Answer :

Exp:

The rectum is the least common site for diverticula because it is surrounded by muscle and has no areas of
weakness in the bowel wall.
Diverticula are subdivided into true diverticula and false (pulsion) diverticula. True diverticula have all layers
present, including the mucosa, submucosa, and muscle wall. False diverticula are created by a weakness in the
underlying muscle wall, such that only the mucosa and submucosa are present in the diverticulum.
In the esophagus, diverticula are three times more common in men than women. A Zenker's diverticulum is the
most common type. It is a false diverticulum located in the upper esophagus. A traction diverticulum is a true
diverticulum that is located in midesophagus at the level of the tracheal bifurcation. It is due to retraction of the
esophagus by scar tissue in the hilar nodes secondary to tuberculosis (mnemonic traction, true, TB = 3Ts).
A Meckel's diverticulum is a true diverticulum located on the antimesenteric border of the ileum 2 feet from the
ileocecal valve. It represents the persistence of the omphalomesenteric duct. Peptic ulceration due to gastric
mucosa (40% to 50%) with bleeding is the most common complication and leads to iron deficiency. It is one of
the most common causes of gastrointestinal bleeding in the newborn (other than swallowed maternal blood
during delivery) and in children. A good mnemonic is the rule of twos: 2 inches long, 2% incidence, and 2 feet
from the cecum. Other small-bowel diverticula are false diverticula. They are frequently the site of bacterial
overgrowth, which, in turn, may predispose to bile salt deficiency (malabsorption) and B12 deficiency
(megaloblastic anemia).
Diverticula involving the colon are false diverticula. They occur as a double row of diverticula near the taenia coli,
where the vessels penetrate the bowel wall. They are most commonly located in the sigmoid colon, which is the
most common site for diverticula in the entire gastrointestinal tract. Diets that are low in fiber and high in fats
have been implicated because intraluminal pressures increase when stool transit time is slow and constipation is
present. The sacs often be come filled with stool, which hardens to form fecaliths, which predispose patients to
mucosal ischemia, damage, and inflammation (similar to appendicitis). Wide mouthed diverticula are a
characteristic feature of progressive systemic sclerosis.

(Q.257) A 32 yr old female asymptomatic not requiring blood transfusion presents with Hb 13g/dl. Her Hbf levels are 95%, HbA2 1.5%.

Which is the most likely diagnosis?


(a)

Hereditary persistence of fetal hemoglobin

(b)

Beta homozygous thalassemia

(c)

Thalassemia intermedia

(d)

Beta heterozygous thalassemia

Your Response :

Correct Answer :

Exp:

Hereditary persistence of fetal hemoglobin


Persistence of 95% of Hb in the fetal form in an adult who is asymptomatic and requires and requires no blood
transfusion suggests a diagnosis of hereditary persistence of fetal hemoglobin. Combination of 95% HbF and
1.5% HbA2 may also be seen in thalassemia intermedia. Although it does not require transfusion, it is not
asymptomatic.
HEREDITARY PERSISTENCE FETAL HEMOGLOBIN
CHARACTERISTICS
Continued synthesis of high level of HbF in adult life.
A mutation lead to decreased synthesis of and globins
Haemoglobin electophoresis reveals only Hbf Clinically pt is asymptomatic
Mild anemia and slight microcytosis may be preserved.

http://dbmci.examonair.com/Result/ShowAllQuestionInHtml.aspx?testid=5615

95/111

12/25/2014

Dr Bhatia Medical Coaching Institute:Online Test Platform


(Q.258) The deductive approach is done in which of the following condition?
(a)

Treatment of septicemia

(b)

Looking for hidden injury

(c)

Treatment of acute pancreatitis

(d)

Perforation of intestine

Your Response :

Correct Answer :
Exp:

Looking for hidden injury


Baily and Love 25th edition, Page 278
Covert injuries
Looking for the hidden injury can follow three methods:
1 the deductive approach;
2 the look everywhere approach;
3 the focused exclusion approach.
The deductive approach patterns of injury
One can apply common sense and experience to the search for hidden injuries. Injuries occur in patterns along
the lines of energy transmission. Finding an obvious injury may therefore lead to one that is hidden. A good
example is the child restrained by a lap seatbelt in the rear of a car involved in a frontal collision.
There may be an obvious flexion spinal injury, e.g. a Chance fracture. What may be less obvious but should be
sought is damage to the immobile retroperitoneal structures, the pancreas or duodenum. Certain mechanisms
are associated with particular patterns of injury. Thus, if the mechanism and perhaps some of the obvious
injuries are known, one can seek the less obvious injury. An unrestrained passenger in a head-on car crash will
hit their knee on the dashboard. If the tibia is the site of impact the patient has to be evaluated for a potential
rupture of the posterior cruciate ligament and a posterior hip dislocation. A dislocation of the knee should
immediately raise concerns about damage to the popliteal artery and injury to it should be excluded. Table 21.1
gives some examples of patterns of associated injuries

(Q.259) For patients with Parkinson's disease, which one of the following drugs is most useful in the management of tremors?
(a)

Apomorphine

(b)

Cabergoline

(c)

Amantadine
Benzhexol

(d)
Your Response :

Correct Answer :

Exp:

Benzhexol
Currently accepted practice in the management of patients is to delay treatment until the onset of disabling
symptoms and then to introduce a dopamine receptor agonist. If patient is elderly, levodopa is sometimes used
as an initial treatment
Dopamine receptor agonists :Bromocriptine, apomorphine
NEWER AGENTS:
- Ropinirole, cabergoline, pergolide
LEVODOPA
- Usually combined with a decarboxylase inhibitor (e.g. carbidopa or benserazide) to prevent peripheral
metabolism of levodopa to dopamine
- reduced effectiveness with time (usually by 2 years)
- unwanted effects: dyskinesia, 'on-off effect - no use in neuroleptic induced parkinsonism
SELEGILINE : - MAO-B inhibitor
- Reduces dopamine metabolism
ANTIMUSCARINICS
- Useful for tremor
- Reduces inhibition of excitatory cholingeric neurons
- e.g. procyclidine, benztropine, benzhexol
AMANTADINE
- Prevents reuptake of dopamine

(Q.260) A 54-year-old male with no past medical history is found to be in atrial fibrillation during a consultation regarding a sprained ankle.

If the patient remains in chronic atrial fibrillation what is the most suitable form of anticoagulation?
(a)

Aspirin

(b)

Warfarin, target INR 2-3

(c)

No anticoagulation

(d)

Warfarin, target INR 3-4

Your Response :

Correct Answer :

Exp:

Aspirin
Atrial fibrillation: Anticoagulation
The guidelines on the management of atrial fibrillation (AF) suggest a stroke risk stratification approach when
determining how to anticoagulated a patient, as detailed below:
Low - annual risk of stroke = 1 %

http://dbmci.examonair.com/Result/ShowAllQuestionInHtml.aspx?testid=5615

96/111

12/25/2014

Dr Bhatia Medical Coaching Institute:Online Test Platform


Age < 65 years
No history of TIA/stroke or other risk factor.
Use aspirin
Moderate - annual risk of stroke = 4%
Age 65-75 years and no high risk factors, or Age < 65 years with diabetes, hypertension, ischaemic heart disease,
peripheral vascular disease.
Use aspirin or warfarin depending on individual circumstances
High - annual risk of stroke = 8-12% Age> 75 years with diabetes or hypertension
Previous TIA/stroke
Valve disease/prosthetic valve
Thyrotoxicosis
Use warfarin
(Q.261) Yeoman biopsy forceps is used most commonly in which condition?
(a)

CA prostate

(b)

CA rectum

(c)

CA testis
CA penis

(d)
Your Response :

Correct Answer :

Exp:

CA rectum
Baily and Love 25th edition, Page 1232
Proctosigmoidoscopy
Proctosigmoidoscopy will always show a carcinoma, if present, provided that the rectum is emptied of faeces
beforehand.
Biopsy
Using biopsy forceps via a sigmoidoscope, a portion of the edge of the tumour can be removed. If possible,
another specimen from the more central part of the growth should also be obtained.
Novo Surgical Yeoman Rectal Biopsy Forceps are ideal for obtaining small samples from the rectum. The forceps
feature ring-sized handles for precise and accurate maneuvering of the bite and basket. The forceps are available
with oval or square baskets. The shaft is slightly angled from the handles is available in various lengths to
accommodate a wide range of cases.

(Q.262) Visor procedure is done in surgery of which place?


(a)

Brain

(b)

Mouth

(c)

Heart
Lungs

(d)
Your Response :

Correct Answer :

Exp:

Mouth
Baily and Love 25th edition, Page 743
FLOOR OF MOUTH
Carcinoma of the floor of the mouth can spread to the ventral surface of the anterior tongue or encroach upon
the lower anterior alveolus (Fig. 46.19). Surgical excision may include a partial anterior glossectomy and anterior
mandibular resection. Only very small tumours of the floor of mouth can be managed by simple excision. The
visor procedure provides excellent access.
LIST OF IMPORTANT NAMED OPERATIONS IN GENERAL SURGERY :
NAME

USED FOR

Swensons operation, Duhamels operation, Soave


operation

Hirschsprung disease

Nissens total fundoplication, Toupets partial


fundoplication, Hills procedure, Belsey mark IV
operation,Angel chick prosthesis

GERD

Dohlmans operation

Zenkers diverticulum

Delormes procedure

Perineal repair of adult rectal prolapse

Anderson-Hynes dismembered pyeloplasty

Cong. PUJ obstruction

Nesbitts operation

Peyronies disease of penis

Jaboulays operation

Small and medium sized hydrocele

Lords operation

Large hydrocele

Hadfields operation

Periductal mastitis / Duct ectasia

Trendelenberg operation

GSV varicosity

Cokkets and Dodds operation

Subfascial ligation of varicosities

Ivor lewis operation, Mckeown 3 stage


operation,Orringers transhiatal esophagectomy

Ca Esophagus

Fredet- Ramsted operation

CHPS

Thiersch operation,Lockheart mummery rectopexy

Childhood rectal prolapse

Thiersch operation, Delormes operation,Altemiers


operation

Adult rectal prolapse Perineal approaches

http://dbmci.examonair.com/Result/ShowAllQuestionInHtml.aspx?testid=5615

97/111

12/25/2014

Dr Bhatia Medical Coaching Institute:Online Test Platform


Ripsteins operation, Wells operation, Lahauts
operation

Adult rectal prolapse Abdominal approaches

Miles operation/APR

Ca Lower rectum

Hartmanns procedure

Left ca colon

Burrhenne technique

Removal of residual stone in CBD

Grahams repair

Perforated duodenal ulcer

Kocks pouch

Chronic ulcerative colitis

Puestows procedure

Drainage procedure for chronic pancreatitis with duct


dilataion

Thomsons procedure,Le Muserier operation,Tennisson Cleft lip


Randell operation, Millers rotation advancement
technique
Allisons repair

Hiatus hernia

Commandos operation

Ca Tongue

Kraske procedure

Ca rectum (post proctotomy-sphincter sparing)

Miligan Morgan operation

Hemorrhoidectomy

Mayos operation

Umbilical hernia

Psoas hitch operation

Ureteric reconstruction

Nesovics operation

Sports hernia

Cheatle henry operation

Obturatr hernia

Sebrocks operation

Parotid duct fistula

Bentalls operation

Aortic root aneurysm repair

Heller dors operation

Achalasia cardia

Ladds procedure

Midgut volvulus

Lockwoods operation Low repair

Femoral hernia

Mc Evedy operation -- High repai with vertical incision


Lotheissen operation -- Open inguinal canal
Rovsings operation

Polcystic kidney dis

Brunschwings operation

Pelvic exenteration in ca rectum

(Q.263) Imatinib mesylate is a novel molecularly targeted therapy in cancer treatment. All are true statements about imatinib except :
(a)

It is a specific RNA polymerase inhibitor

(b)

First line drug in the management of chronic phase of CML .

(c)

Also effective in gastrointestinal stromal tumors.

(d)

Pancytopenia and hepatotoxicity are dose limiting side effects.

Your Response :

Correct Answer :

It is a specific RNA polymerase inhibitor

Exp:

Imatinib inhibits tyrosine kinase an enzyme produced due to DNA translocation of Ph chromosome.
Clinical uses
GIST (Gastrointestinal stromal tumours)
Philadelphia positive ALL
Clonal eosinophilic disorders.
Effective in PDGF platelet derived growth factor mutations.
Usual dosage 400-600 mg per day in adults
Side effects - GI upset. Pancytopenia, arthralgias, myalgias, hepatotoxicity.
(Q.264) Oseltamivir the drug available for the treatment of ' Avian flu' belongs to which pharmacological class?
(a)

Reverse transcriptase inhibitor

(b)

Protease inhibitor

(c)

Neuraminidase inhibitors

(d)

Ribonuclease inhibitor

Your Response :

Correct Answer :

Exp:

Neuraminidase inhibitors
Oseltamivir and Zanamivir are two drugs of neuraminidase inhibitor group which are known to reduce the
severity and duration of illness caused by avian influenza. Oseltamivir is given 75 mg orally bid for 5 days, to be
started within 48 hours of symptom onset. Zanamivir is to be given by inhalation.

(Q.265) A 23-year-old woman presents to the OPD with complaint of lethargy. The blood investigations are as follows:

Hb-10 g, TLC 6300, Platlets-2,78000,


MCV-65 and HbA2- 4.5%.
What - is the most likely diagnosis?
(a)

Beta-thalassanemia major

(b)

Sickle cell anemia

(c)

Beta-thalassanemia trait

http://dbmci.examonair.com/Result/ShowAllQuestionInHtml.aspx?testid=5615

98/111

12/25/2014

Dr Bhatia Medical Coaching Institute:Online Test Platform


(d)

Hereditary spherocytosis

Your Response :

Correct Answer :

Exp:

Beta-thalassanemia trait
A microcytic anaemia in a female should raise the possibility of either gastrointestinal blood loss or menorrhagia.
However, there is no history to suggest this and the microcytosis is disproportionately low for the haemoglobin
level. This combined with a raised HbA2 points to a diagnosis of beta-thalassanemia trait
Beta-thalassanemia trait
The thalassaemias are a group of genetic disorders characterised by a reduced production rate of either alpha or
beta chains. Betathalassaemia trait is an autosomal recessive condition characterised by a mild hypochromic,
microcytic anaemia. It is usually asymptomatic
Features
Mild hypochromic, microcytic anaemia microcytosis is characteristically disproportionate to the anaemia
HbA2 raised (> 3.5%)

(Q.266) In a patient with mild hypertension which is not affecting his functional activity, will be classified as:
(a)

ASA I

(b)

ASA II

(c)

ASA III
ASA IV

(d)
Your Response :

Correct Answer :

Exp:

ASA II
American society of anesthesiologist has classified patients in five categories.
ASA Physical Status Classification System
ASA Physical Status 1- A normal healthy patient
ASA Physical Status 2- A patient with mild systemic disease
ASA Physical Status 3- A patient with severe systemic disease
ASA Physical Status 4- A patient with severe systemic disease that is a constant threat to life
ASA Physical Status 5- A moribund patient who is not expected to survive without the operation
ASA Physical Status 6- A declared brain-dead patient whose organs are being removed for donor purposes
If the surgery is an emergency, the physical status classification is followed by E (foremergency) for example
3E. Class 5 is usually an emergency and is therefore usually "5E". The class "6E" does not exist and is simply
recorded as class "6", as all organ retrieval in brain-dead patients is done urgently. The original definition of
emergency in 1940, when ASA classification was first designed, was "a surgical procedure which, in the surgeon's
opinion, should be performed without delay.This gives an opportunity for a surgeon to manipulate the schedule
of elective surgery cases for personal convenience. An emergency is therefore now defined as existing when
delay in treatment would significantly increase the threat to the patient's life or body part.With this definition,
severe pain due to broken bones, ureteric stone or parturition (giving birth) is not an emergency.

(Q.267) Untrue about Magill circuit is :


(a)

Type A Mapleson

(b)

Circuit of choice for controlled ventilation

(c)

Fresh gas flow should be equal to the minute volume

(d)

Expiratory valve is near to patient

Your Response :
Correct Answer :

Exp:

Circuit of choice for controlled ventilation


Magill circuit is type A Mapleson circuit.
Fresh gas flow should be equal to minute volume to prevent rebreathing (that means that patient should not
reinhale his expired gases). It is the circuit of choice for spontaneously breathing patient.
The expiratory valve should be as near to the patient to allow maximum exhalation of expired gases.

(Q.268) Jackson Rees is modification of :


(a)

Type A

(b)

Type B

(c)

Type D

(d)

Type F

Your Response :
Correct Answer :

Exp:

Type F
Anesthetic breathing systems are classified as open, semiopen, semiclosed, and closed according to the presence
or absence of (1) a gas reservoir bag in the circuit, (2) rebreathing of exhaled gases, (3) means to chemically
neutralize exhaled carbon dioxide, and (4) unidirectional valves. The most commonly used anesthetic breathing
systems are the (1) Mapleson F (Jackson-Rees) system, (2) Bain circuit, and (3) circle system.
Mapleson F (Jackson-Rees) System
The Mapleson F (Jackson-Rees) system is a T-piece arrangement with a reservoir bag and an adjustable pressurelimiting overflow valve on the distal end of the gas reservoir bag.The degree of rebreathing when using this

http://dbmci.examonair.com/Result/ShowAllQuestionInHtml.aspx?testid=5615

99/111

12/25/2014

Dr Bhatia Medical Coaching Institute:Online Test Platform


anesthetic breathing system is influenced by the method of ventilation (spontaneous versus controlled) and
adjustment of the pressure-limiting overflow valve (venting). Fresh gas flow equal to two to three times the
patients minute ventilation is recommended to prevent rebreathing of exhaled gases.
FLOW CHARACTERISTICS
During spontaneous ventilation, exhaled gases pass down the expiratory limb and mix with fresh gases.The
expiratory pause allows the fresh gas to push the exhaled gases down the expiratory limb. With the next
inspiration, the inhaled gas mixture comes from the fresh gas flow and from the expiratory limb, including the
reservoir bag.
CLINICAL USES
The Mapleson F system is commonly used for controlled ventilation during transport of tracheally intubated
patients. Because of no moving parts except the pressure-limiting overflow valve, minimum dead space and
resistence exist. This is ideal for pediatric anesthesia. The Mapleson F system may be used for both spontaneous
and controlled ventilation. It is inexpensive, can be used with a facemask or endotracheal tube, is lightweight,
and can be repositioned easily. Pollution of the atmosphere with anesthetic gases when using this system can be
decreased by adapting it to scavenging systems.
DISADVANTAGES
Disadvantages of the Mapleson F system include (1) the need for high fresh gas inflow to prevent rebreathing,
(2) the possibility of high airway pressure and barotrauma should the overflow valve become occluded, and (3)
the lack of humidification. Lack of humidification can be offset by allowing the fresh gas to pass through an inline heated humidifier.

(Q.269) All of the following are suitable anaesthetic circuits for both controlled and assisted ventilation except ?
(a)

Mapleson A

(b)

Mapleson B & C

(c)

Mapleson D

(d)

Mapleson E

Your Response :
Correct Answer :

Exp:

Mapleson A
Mapleson B & C are no more used in anaesthesia practice but if the characteristic of these circuits are seen they
are almost equally effective for controlled and spontaneous ventilation.
Mapleson E & F (although E is primarily used for spontaneous but controlled ventilation is possible by
intermittently occluding and sealing the end of tube) can be used for controlled and spontaneous ventilation.
Type D (Bain) requires high flow for spontaneous but does work efficiently for spontaneous also.
Mapleson A (Magill) is circuit of choice for spontaneous respiration, for controlled ventilation it may require very
high flows (may be > 3 times minute volume or> 20 L by some studies) and in spite of that, prevention of
rebreathing is unpredictable, so Mapleson A should not used for controlled ventilation.

(Q.270) Commonest post operative complication of intubation is :


(a)

Sore throat

(b)

Aspiration pneumonitis

(c)

Vocal cord granuloma

(d)

Tracheal stenosis

Your Response :

Correct Answer :

Exp:

Sore throat
Most common post operative complication of intubation is sore throat which usually subsides in 2-3 days
without any specific treatment.
Complications of intubation
A. Perioperative
1. Esophageal intubation
3. Aspiration

2. Ischemia, edema and necrosis at local site

4. Bronchial intubation & collapse of other lung.

5. Tracheal tube obstruction by secretions


7. Trauma to gums
B.

6. Accidental extubation

8. Cardiac arrhythmias

Postoperative

1. Sore throat

2. Laryngeal nerve palsies

C. Delayed complications
1. Vocal cord granuloma
3. Tracheal stenosis

2. Laryngotracheal web

4. Tracheal collapse

(Q.271) All of the following features distinguish infant larynx from adult EXCEPT:
(a)

Epiglottis is long and leafy

(b)

Subglottic region is narrowest laryngeal portion

(c)

Large tongue

(d)

Lower placement of the larynx

Your Response :

Correct Answer :

http://dbmci.examonair.com/Result/ShowAllQuestionInHtml.aspx?testid=5615

100/111

12/25/2014

Dr Bhatia Medical Coaching Institute:Online Test Platform


Following features of infant larynx in contrast to adult larynx:

Exp:

1.

Epiglottis is long and leafy.

2.

Subglottic region is the narrowest portion of larynx.

3.

Larynx is placed at a higher level (in adults, it is placed at the level of C3-C6 vertebrae).

Anatomic Differences between the Child's and Adult's Larynx


Child's larynx

Adult's larynx

Size

Smaller

Larger

Shape

Lumen is funnel-shaped with the narrowest part below the


vocal cords and within the cricoid ring

Narrowest part of lumen is


at the vocal cords

Location

Higher, closer to the tongue base; vertical extent is opposite Vertical extent is lower,
C3, C4, C5 vertebrae; more anterior
opposite C4, C5, C6
vertebrae

Epiglottis

Longer, narrower, and "U" shaped; the angle between


glottis and epiglottis is more acute; increased chance of
airway obstruction (see Figure 1-8)

Shorter and wider

Vocal cords

Angled in relation to the axis of trachea; shorter; more


cartilaginous; more distensible; more likely to be injured

Perpendicular to the axis of


trachea

Rigidity

The laryngeal cartilages are softer and more pliable

More rigid

Response to
trauma

Mucous membrane is more loosely attached and swells


more readily when traumatized or infected

Less vulnerable to trauma


and infection

(Q.272) The laryngeal mask airway used for securing the airway of a patient in all of the following conditions except:
(a)

In a difficult intubation

(b)

In cardiopulmonary resuscitation

(c)

In a child undergoing an elective/routine eye surgery

(d)

In a patient with a large tumor in the oral cavity

Your Response :

Correct Answer :

Exp:

In a patient with a large tumor in the oral cavity


The indications for laryngeal mask airway are:
Difficult intubation.
Emergency airway management especially during difficult or failed intubation like in CPR. As elective procedure.
where anaesthetist wants to avoid intubation (like for minor cases) As a conduit for endotracheal tubes,
bronchoscope etc.
Contraindications are:
Full stomach patient or patients who are at high risk of aspiration.
Patients vulnerable for laryngospasm and broncho-spasm.
Mass and abscess in oral cavity, so large tumour in oral cavity is contraindication.

(Q.273) Dibucaine number is utilized for diagnosing:


(a)

Atypical pseudocholinesterase

(b)

Degree and intensity of block after succinyl choline

(c)

Dual block

(d)

Duration of block

Your Response :

Correct Answer :

Exp:

Atypical pseudocholinesterase
Atypical pseudocholinesterase is a genetic disease in which the patient's pseudocholinesterase can not
metabolize succinylcholine and there can be very prolonged block.
Dibucaine a local anaesthetic that can inhibit 80% of normal enzyme and 20% of abnormal enzyme.
So normal dibucaine number is 80.
The dibucaine number (DN) is the percent of pseudocholinesterase (PChE) enzyme activity that is inhibited by
dibucaine. Together, the DN and the PChE enzyme activity results can help to identify individuals at risk for
prolonged paralysis following the administration of succinylcholine. Decreased PChE enzyme activity in
conjunction with a DN less than 30 suggests high risk for prolonged paralysis. Normal to decreased PChE enzyme
activity in conjunction with a DN 30-79 suggests variable risk. Although decreased PChE activity in conjunction
with DN greater than or equal to 80 suggests variable risk, these results may be caused by exposure to
organophosphates, the presence of liver disease, pregnancy, or circulating succinylcholine. Specimens should be
collected 48 hours after the administration of succinylcholine.

(Q.274) First muscle to be blocked by muscle relaxants:


(a)

Laryngeal muscles

(b)

Diaphragm

(c)

Thenar muscles

(d)

Intercostals

Your Response :

Correct Answer :

http://dbmci.examonair.com/Result/ShowAllQuestionInHtml.aspx?testid=5615

101/111

12/25/2014

Dr Bhatia Medical Coaching Institute:Online Test Platform


Laryngeal muscles

Exp:

First muscles to be blocked by muscle relaxants (both depolarizing and non depolarizing) are central muscles i.e.,
muscles of head and neck (face, jaw, pharynx, larynx), respiratory, abdominal muscles and muscles of trunk.
After the central muscles, muscles of limbs (peripheral muscles) are blocked.
The sequence of recovery is in the same way as it goes i.e., first to recover are central muscles like larynx and
diaphragm and then limb muscles.
(Q.275) In Erb-Duchenne palsy all of the following Imuscle are paralyzed except :
(a)

Supraspinatus

(b)

Biceps brachii

(c)

Pectoralis major

(d)

Deltoid

Your Response :

Correct Answer :

Exp:

Pectoralis major
Erb-Duchenne palsy
It results from injury to upper trunk of brachial plexus (C5,6).
Muscles paralysed: Mainly Biceps, Deltoid, Brachialis & Brachioradialis; Partly Supraspinatus, Infraspinatus &
Supinator.
The pectoralis major is not paralyzed because it is supplied both by the lateral and medial pectoral nerves and
not merely by branches from the upper trunk of the lateral cord.

(Q.276) Which of the following structures pass between the External and Internal carotid arteries ?
(a)

Styloglossus

(b)

Stylopharyngeus

(c)

IXth Cranial nerve

(d)

All of above

Your Response :

Correct Answer :

Exp:

All of above
ORDER OF ARRANGEMENT AT SPECIFIC SITES
FEMORAL TRIANGLE

Within femoral sheath from medial to lateral


side- VAN (vein artery - nerve )
Femoral canal- lymph node of cloquet/
Rosenmuller
Middle compartment- femoral vein
Lateral compartment- femoral artery with
femoral branch of genitofemoral nerve
Outside of sheath and lateralmost- femoral
nerve

AT APEX OF FEMORAL TRIANGLE

From anterior to posterior


Femoral artery
Femoral vein
Profunda femoris artery and vein

POPLITEAL FOSSA

Upper part: medial to lateral artery- vein- tibial


nerve
Middle part: superior to deep- nerve-vein-artery
Lower part: medial to lateral- nerve-vein-artery

ANTERIOR ASPECT OF ANKLE THROUGH


EXTENSOR RETINACULUM

Medial to lateral (The Himalayas Are Never Dry


Places )
Tibial anterior muscle
Extensor Hallucis longus
Anterior tibial artery
Deep Peroneal nerve
Extensor Digitorum longus
Peroneus tertius

THROUGH FLEXOR RETINACULUM BEHIND


MEDIAL MALLEOUS

Medial to lateral(Anterior to posterior) -( The


Doctors Are Not Here)
Tibialis posterior
Flexor Digitorum longus
Posterior tibial artery
Tibial nerve
Flexor Hallucis longus

CUBITAL FOSSA

Medial to lateral (MBBR)


Median nerve
Brachial artery
Biceps tendon
Radial nerve

STRUCTURES WITHIN THE PAROTID GLAND

Medial to lateral
Arteries- Maxillary and Superficial temporal

http://dbmci.examonair.com/Result/ShowAllQuestionInHtml.aspx?testid=5615

102/111

12/25/2014

Dr Bhatia Medical Coaching Institute:Online Test Platform


Retromandibular vein
Facial nerve
Parotid lymph node
THYROID GLAND

Dense capillary plexus lies deep to the capsule


Thyroid is removed along with the true capsule

PROSTATE

Capillary plexus is present between true and


false capsule
Prostate removed and both the capsules are left
behind

At aortic opening of Diaphragm(T12)

Left to RightAorta
Thoracic duct
Azygous vein

ROOT OF LUNG

From before backwards (SVAB) (same on both


sides)
Superior Pulmonary Vein
Pulmonary Artery
Bronchus
From Above downwards
RIGHT SIDE
Eparterial Bronchus
Pulmonary Artery
Hyparterial branch
Inferior pulmonary vein

LEFT SIDE (ABI)


Pulmonary artery
Bronchus
Inferior pulmonary vein

IN BRONCHOPULMONARY SEGMENT

Pulmonary artery lies dorsolateral to bronchus

AT HILUM

Pulmonary veins are ventromedial to bronchus

NECK OF 1ST RIB

Related anteriorly from medial to lateral side


(SPIT)
Sympathetic chain
1st posterior intercostal vein
Intercostal artery
1st thoracic nerve

CAVERNOUS SINUS

Lateral wall above downwards


3rd nerve
4th nerve
5th nerve
Trigeminal ganglion
Within centre of sinus
ICA with venous and symphatetic plexus
Abducent nerve

STRUCTERS PASSING BETWEEN EXTERNAL AND


INTERNAL CAROTID ARTERY

Styloid process
Styloglossus
Stylopharyngeus
IXth nerve
Pharyngeal branch of vagus nerve
Part of parotid gland

STRUCTERS DEEP TO POSTERIOR BORDER OF


HYOGLOSSUS

From above downwards (NiSHA)


IX th nerve
Stylohyoid ligament
Lingual artery

COSTAL GROOVE OF RIBS

From above downwards(VAN)


Posterior intercostal vein
Posterior intercostal artery
Intercostal nerve

AT COMMON CAROTID ARTERY BIFURCATION

http://dbmci.examonair.com/Result/ShowAllQuestionInHtml.aspx?testid=5615

Internal carotid artery is posterolateral to


external carotid artery

103/111

12/25/2014

Dr Bhatia Medical Coaching Institute:Online Test Platform


IN FRONT OF ALA OF SACRUM

(SLIO)
Symphatetic trunk
Lumbosacral trunk
Iliolumbar artery
Obturator nerve

HILUM OF KIDNEY

Anterior to posterior(VAP)
Vein
Artery
pelvis

STRUCTURES PIERCING CLAVIPECTORAL FASCIA

(TALC)
Thoracoacromial vein
Lateral pectoral nerve
Cephalic vein

AT PORTA HEPATIS

Anterior to posterior (DAV)


CBD
Hepatic artery
Portal vein

IN RADIAL/SPIRAL GROOVE OF HUMERUS

Radial nerve
Profunda brachial artery

INBETWEEN SUPERIOR CONSTRICTOR AND BASE Sinus of Morgagni


OF SKULL
Auditory tube
Levator palatine MUSCLEnerve
INBETWEEN SUPERIOR AND MIDDLE
CONSTRICTOR

Stylopharyngeus muscle
Glossopharyngeal nerve
Ascending palatine artery

INBETWEEN MIDDLE AND INFERIOR


CONSTRICTOR

Internal laryngeal nerve

INBETWEEN INFERIOR CONSTRICTOR AND


ESOPHAGUS

Recurrent laryngeal nerve

Greater sciatic foramen (structures passing


through)

1. Pyriformis

Superior laryngeal vessels

Inferior laryngeal vessels

2. Superior and inferior gluteal nerve and


vessels
3. Sciatic nerve
4. Pudendal nerve
5. Internal pudendal vessels
6. Posterior cutaneous nerve of thigh
7. Nerve to quadratus femoris
8. Nerve to obturator internus.

Lesser sciatic foramen (structures passing


through)

1. Tendon of obturator internus


2. Nerve to obturator internus
3. Pudendal nerve
4. Internal pudendal vessels
5. Origin of gemelli

(Q.277) All of the following tissues are derived from the dorsal mesentery EXCEPT
(a)

The splenorenal ligament

(b)

The visceral peritoneum of the spleen

(c)

The greater omentum

(d)

The anterior border of the epiploic foramen

Your Response :

Correct Answer :

Exp:

The anterior border of the epiploic foramen is formed by the free edge of the lesser omentum. The lesser
omentum is derived from the ventral mesentery. The parietal peritoneum to the right of the descending colon is
derived from the dorsal mesentery of this part of the hindgut, but fuses to the posterior wall of the peritoneal
cavity during the process of retro peritonealization of the descending colon. The greater omentum is the
expanded portion of the dorsal mesogastrium. After the spleen develops in the dorsal mesentery, the portion of
the mesentery covering the spleen becomes the visceral peritoneum of the spleen; the portion of the mesentery
dorsal to the spleen becomes the splenorenal ligament.

(Q.278) Tingling or painful sensations in the lateral region of the thigh may occur in the older, overweight individual as a result of a bulging

abdomen, compressing a nerve beneath the inguinal ligament. Which of the following nerves is most likely to be involved?
(a)

Anterior femoral cutaneous

(b)

Femoral branch of genitofemoral

(c)

Genital branch of genitofemoral

(d)

Lateral femoral cutaneous

http://dbmci.examonair.com/Result/ShowAllQuestionInHtml.aspx?testid=5615

104/111

12/25/2014

Dr Bhatia Medical Coaching Institute:Online Test Platform


Your Response :

Correct Answer :

Exp:

Lateral Femoral Cutaneous Nerve


Injury or entrapment of the lateral femoral cutaneous nerve is also known as meralgia paresthetica. It is derived
from the Greek wordmeros,meaning thigh, andalgo,meaning pain. It is a syndrome of paresthesia and pain in the
lateral and anterolateral thigh. This syndrome is seen most commonly in individuals aged 20-60 years, but it can
occur in people of all ages.
Anatomy
This nerve arises from the ventral primary rami of L2-4 where they divide into anterior and posterior branches.
The dorsal portions fuse to form the lateral femoral cutaneous nerve in the midpelvic region of the psoas major.
The nerve then courses over the iliacus toward the ASIS. The nerve travels posterior to the inguinal ligament and
superior to the sartorius muscle at the iliac crest region and divides into anterior and posterior branches. The
anterior branch comes off 10 cm distal to the inguinal ligament in line with the ASIS, and supplies cutaneous
sensation to the lateral thigh, including just proximal to the patella. It then communicates with cutaneous
branches of the femoral nerve and saphenous nerve to form the patellar plexus. The posterior branch pierces
the fascia lata posteriorly and laterally, and divides into multiple, small branches that supply the skin from the
greater trochanter to the midthigh.
Etiology
Entrapment usually occurs at the inguinal ligament. The peak incidence for this condition is in middle age.
Differential diagnoses include lumbar radiculopathies and discogenic or nerve root problems at L2 and L3. The
entrapment may be from intrapelvic causes, extrapelvic causes, or mechanical causes. Intrapelvic causes include
pregnancy, abdominal tumors, uterine fibroids, diverticulitis, or appendicitis. Injury has been described in cases
of abdominal aortic aneurysm. Examples of extrapelvic causes include trauma to the region of the ASIS (eg, from
a seatbelt in a motor vehicle accident), tight garments, belts, girdles, or stretch from obesity and ascites.
Mechanical factors include prolonged sitting or standing and pelvic tilt from leg-length discrepancy. Diabetes can
also cause this neuropathy in isolation or in the clinical setting of a polyneuropathy.
Clinical
Symptoms include anterior and lateral thigh burning, tingling, and/or numbness, that increase with standing,
walking, or hip extension. Symptoms may also increase with lying prone. Symptoms usually are unilateral but
may be bilateral in rare cases. The symptoms usually improve with sitting unless compressive forces, such as
tight belts or garments, remain.
Physical examination findings may be completely normal. Findings may include hyperesthesia over the lateral
thigh (usually in a smaller area than the symptoms). Pain can be produced by pressure medial to the ASIS. A
positive Tinel sign may be present over the ASIS or inguinal ligament.
Diagnosis of this entrapment may again be based on an injection of local anesthetic near the inguinal ligament or
ASIS. Spontaneous recovery usually is expected. Electrodiagnostic testing may be performed for diagnosis. With
nerve conduction studies, the technique includes using a bar electrode for recording and reference. This can be
performed with either antidromic (conduction against the direction of sensory fiber conduction) or orthodromic
(conduction in the direction of nerve conduction) methods. The antidromic study is usually easier to perform,
although response may be absent bilaterally on occasion. The response is small and difficult to obtain in obese
patients.
A needle stimulation electrode may needed. The sensory response is absent in 71% of patients with meralgia
paresthetica and is prolonged in 24% of patients with this condition. Electromyographic test results with needle
are normal in patients with this diagnosis, which may help to differentiate it from an upper lumbar
radiculopathy. Technically, the sensory test is a difficult study and a response must be present on the opposite
side to determine entrapment. It may be nearly impossible to obtain a response in an obese patient or a patient
with a large abdomen without using a needle for stimulation. Unfortunately, the test may be difficult for the
patient to tolerate because of the large amount of current (with respect to more peripheral nerves) that is
required to stimulate a nerve that lies under adipose tissue.
Treatment
Treatment may include the injection of local anesthetic agents, as previously noted. A steroid can also be used to
prolong the effects of the local anesthetic and reduce inflammation. Oral medications, such as NSAIDs,
antiseizure medications (gabapentin [Neurontin]), tricyclic antidepressants, and tramadol may be used, as may
capsaicin cream and topical lidocaine. One must also instruct patients on ways to prevent further irritation of the
nerve. These may include avoidance of hip extension, prolonged standing, and compressive garments. The use of
ice and a TENS unit may also be helpful. Surgical exploration may be required if the above treatment options are
not helpful. This includes transection of the nerve, or decompression with or without neurolysis.Anatomical
variations of the nerve and neuromas can occur and lead to recurrence
Upper limb
Nerve
median

Entrapment site

Named as

carpal tunnel

carpal tunnel syndrome

median(anterior interosseous) proximalforearm

anterior interosseous syndrome

median

pronator teres

pronator teres syndrome

median

ligament of Struthers ligament of Struthers syndrome

ulnar

cubital tunnel

cubital tunnel syndrome

ulnar

Guyon's canal

Guyon's canal syndrome

radial

axilla

radial nerve compression

radial

spiral groove

radial nerve compression

radial(posterior interosseous)

proximalforearm

posterior interosseous nerve entrapment

radial(superficial radial)

distalforearm

Wartenberg's Syndrome

suprascapular

suprascapular notch

suprascapular nerve entrapment

Lower limb, abdomen and pelvis[edit]


Nerve

http://dbmci.examonair.com/Result/ShowAllQuestionInHtml.aspx?testid=5615

Entrapment site Named as

105/111

12/25/2014

Dr Bhatia Medical Coaching Institute:Online Test Platform


common peroneal

fibular neck

peroneal nerve compression

tibial

tarsal tunnel

tarsal tunnel syndrome

lateral cutaneous nerve of thigh inguinal ligament meralgia paraesthetica


sciatic

piriformis

piriformis syndrome[not always due to entrapment]

iliohypogastric

lower abdomen

iliohypogastric nerve entrapment

obturator

obturator canal

obturator nerve entrapment

pudendal

pelvis

pudendal nerve entrapment

abdominalcutaneous nerves

abdominal wall

abdominal cutaneous nerve entrapment syndrome

(Q.279) At the knee joint, all of the following structures are intraarticular except :
(a)

Menisci

(b)

Tendon of popliteus muscle'

(c)

Cruciate ligaments

(d)

Oblique popliteal ligament

Your Response :

Correct Answer :

Exp:

Oblique popliteal ligament


The oblique popliteal ligament, being a fibrous expansion of the tendon of semimembranosus muscle, is
extracapsular. The ligament passes obliquely upwards and laterally to strengthen the capsule of the knee joint
from the posterior aspect.
- All the other three structures are intra-articular, the tendon of popliteus being partly so.

(Q.280) A hydrocele can be tapped by inserting a needle into the distended tunica vaginalis. In this procedure the following is not pierced :
(a)

Dartos muscle and superficial fascia

(b)

Tunica albuginea

(c)

Internal spermatic fascia

(d)

Cremasteric fascia

Your Response :

Correct Answer :

Exp:

Tunica albuginea
Hydrocele is collection of fluid between the layers of the tunica vaginalis.
The tunica albuginea is a dense, fibrous capsule covering the testis. In turn it is covered by a layer of peritoneum,
the tunica vaginal is. Thus it is not a part of the scrotal wall.

(Q.281) Which cranial nerve is likely to be damaged if the direct and consensual light reflexes are absent in a patient assumed to have

normal eyesight?
(a)

Optic

(b)

Oculomotor

(c)

Abducent
Trochlear

(d)
Your Response :

Correct Answer :

Exp:

Oculomotor
Light reflexes (direct & consensual) involve the sphincter pupillae supplied by the ciliary ganglion which receives
preganglionic fibres from the Edinger-Westphal part of third nerve (oculomotor) nucleus.

(Q.282) Following tonsillectomy, one of the main sources of prolonged postoperative bleeding is from :
(a)

External palatine vein (paratonsillar vein)

(b)

Pharyngeal veins

(c)

Maxillary artery

(d)

Sphenopalatine artery

Your Response :

Correct Answer :

Exp:

External palatine vein (paratonsillar vein)


The external palatine vein (paratonsillar vein) runs vertically between the superior constrictor (medial aspect)
and the lateral aspect of the fibrous "capsule" of the tonsil. It is the main source of bleeding after tonsillectomy.

(Q.283) Out of the following signs the most significant sign in establishing the diagnosis of rosacea
(a)

Edema

(b)

Central facial erythema

(c)

Papules

(d)

Telangiectasia

Your Response :

http://dbmci.examonair.com/Result/ShowAllQuestionInHtml.aspx?testid=5615

106/111

12/25/2014

Dr Bhatia Medical Coaching Institute:Online Test Platform


Correct Answer :

Exp:

Central facial erythema


Rosacea is a common skin disorder characterized by transient or persistent central facial erythema also called
persistent flushing, visible blood vessels, papules and pustules. The etiology of rosacea is not well understood.
Clinically rosacea is classified into four categories or subtypes: papulopustular, erythematotelangiectatic,
phymatous and ocular. The earliest clinical sign that is also a hallmark of rosacea is central flushing or a
permanent blush.

(Q.284) Each of the following is a feature of rosacea except?


(a)

Flushing

(b)

Rhinophyma

(c)

Styes and chalazia

(d)

Genital ulcerations

Your Response :

Correct Answer :

Exp:

Genital ulcerations
Rosacea is a chronic cutaneous disorder of the central portion of the face such as cheeks, chin, nose and central
forehead. It is characterized by remissions and exacerbations. Rosacea is a syndrome comprising of a
combination of cutaneous signs as flushing, erythema, telangiectasia, edema, papules, pustules, nodules, ocular
lesions and rhinophyma. There are no genital lesions in rosacea.

(Q.285) Which of the following topical agent has the properties of remodeling connective tissue and posses antiinflammatory activity?
(a)

Benzoyl peroxide

(b)

Metronidazole

(c)

Gentamycin

(d)

Tretinoin

Your Response :

Correct Answer :

Exp:

Tretinoin
Topical tretinoin promotes connective tissue remodeling in the papillary and reticular zones of the dermis. Longterm tretinoin therapy reduces the dermal inflammation. Thus, topical tretinoin is useful for the treatment albeit
it takes about 2 months for the clinical response to become apparent.

(Q.286) Drug used in treatment of cerebral salt wasting syndrome is?


(a)

Desmopressin

(b)

Conivaptan

(c)

Fludrocortisone

(d)

All of the above

Your Response :

Correct Answer :

Exp:

Fludrocortisone

Treatment
While CSWS usually appears within the first week after brain injury and spontaneously resolves in 24
weeks,
It can sometimes last for months or years.
While fluid restriction is used to treat SIADH, CSWS requires aggressive hydration and correction of the low
sodium levels using sodium chloride tablets. (volume for volume) (Ref: Nelson 18th edition 2302)
Sometimes,fludrocortisone(amineralocorticoid) improves the hyponatremia.
(Q.287) In case of acute bacterial wound infection after surgical procedure the first and most appropriate action would be ?
(a)

Get culture and sensitivity done

(b)

Start a course of systemic antibiotics

(c)

Start oral antifungal drugs

(d)

Use topical corticosteroids

Your Response :

Correct Answer :

Exp:

Get culture and sensitivity done


Wound infection after surgical procedure can drastically affect the outcome of surgery by completely disrupting
wound healing and scarring. Most of the surgical wound infection occurs due to the introduction of pathogenic
organisms during the surgical procedure such as S. aureus, B-hemolytic streptococcus and Pseudomonas
aeruginosa. Early and precise identification of the causative organism and its sensitivity pattern to antibiotics
from the exudate is imperative for making the right choice of the drug to control and eradicate the infection.
Once the sample is taken and sent to laboratory, an oral antibiotic can be started while awaiting the pathology
report.

(Q.288) Which of the following statement about pyoderma gangrenosum is true?

http://dbmci.examonair.com/Result/ShowAllQuestionInHtml.aspx?testid=5615

107/111

12/25/2014

Dr Bhatia Medical Coaching Institute:Online Test Platform


(a) Pyoderma gangrenosum is a skin disease with no multisystem involvements
(b) Pyoderma gangrenosum is more common, in children than in adults
(c) Culture-negative pulmonary infiltrates may occur in patients with neutrophilic dermatoses
(d)

Unlike cutaneous disease systemic involvement with neutrophilic dermatoses does not clear with steroids or
immunosuppressive agents

Your Response :

Correct Answer :

Exp:

Culture-negative pulmonary infiltrates may occur in patients with neutrophilic dermatoses


Pyoderma gangrenosum is an uncommon ulcerative skin disorder characterized by rapidly enlarging necrotic
ulceration with a mucopurulent base and violaceous undermined border. It is generally associated with
inflammatory bowel disease, arthritis, and immunologic and hematologic disorder; but can also occur in normal
persons. The lesions are generally limited to the skin but there may be multisystem involvement such as lungs,
heart, joints, eyes, gastrointestinal tract and central nervous system. Pyoderma gangrenosum is characterized by
sterile neutrophilic infiltrations of the tissues. The condition is rare in children. Pyoderma gangrenosum is a
multisystem neutrophilic disease which requires aggressive treatment with systemic corticosteroids and other
immunosuppressive agents.

(Q.289) The most common cause of death in pemphigus is ?


(a)

Fluid and electrolyte imbalance

(b)

Infection with Pseudomonas

(c)

Gastrointestinal hemorrhage

(d)

Infection with Staphylococcus aureus

Your Response :

Correct Answer :

Exp:

Infection with Staphylococcus aureus


The blisters of pemphigus vulgaris tend to rupture easily discharging their serum contents on and around the
ulcer. This acts an ideal medium for bacterial growth and subsequent septicemia. Staphylococcus aureus is the
most common organism infecting these lesions. It is virulent organism and extensive septicemia is a common
cause of death in pemphigus cases.

(Q.290) During a biopsy of annular nodular lesion on elbow region it was noticed that the patient neither felt the entry of the needle nor

the pain of anesthetic injection. The most likely diagnosis is?


(a)

Psoriasis

(b)

Lichen planus

(c)

Leprosy

(d)

Granuloma annulare

Your Response :

Correct Answer :

Exp:

Leprosy tuberculoid type


Leprosy or Hansen's disease caused by Mycobacterium leprae is a chronic mycobacterial disease that affects
millions of people. The organism has the ability to invade and survive within nerve cells and during the process
damage the cell resulting anesthetic patches. The disease is classified into two poles tuberculoid and
lepromatous with different grades in between them. In the tuberculoid end of the spectrum the disease is
localized and confined due to good resistance in the patient. Tuberculoid leprosy is clinically characterized by
nerve enlargement, hypopigmented or pink patches or plaques that are usually anesthetic. The histology shows
localized granuloma formation in the dermis.
Clinical, Bacteriologic, Pathologic, and Immunologic Spectrum of Leprosy
Tuberculoid (TT, BT)
Leprosy

Borderline (BB, BL) Leprosy

Lepromatous (LL) Leprosy

Skin lesions

One or a few sharply


defined annular
asymmetric macules or
plaques with a tendency
toward central clearing,
elevated borders

Intermediate between BT
and LL type lesions; illdefined plaques with an
occasional sharp margin; few
or many in number

Symmetric, poorly marginated,


multiple infiltrated nodules and
plaques or diffuse infiltration;
xanthoma-like or dermatofibroma
papules; leonine facies and
eyebrow alopecia

Nerve lesions

Skin lesions anesthetic


Hypesthetic or anesthetic
early; nerve near lesions skin lesions; nerve trunk
sometimes enlarged;
palsies, at times symmetric
nerve abscesses most
common in BT

Hypesthesia a late sign; nerve


palsies variable; acral, distal,
symmetric anesthesia common

Acid-fast bacilli
(BIa)

01+

35+

46+

Lymphocytes

2+

1+

01+

Macrophage
differentiation

Epithelioid

Epithelioid in BB; usually


undifferentiated, but may
have foamy changes in BL

Foamy change the rule; may be


undifferentiated in early lesions

Langhans' giant
cells

13+

Lepromin skin
test

+++

Lymphocyte

Generally positive

110%

12%

Feature

http://dbmci.examonair.com/Result/ShowAllQuestionInHtml.aspx?testid=5615

108/111

12/25/2014

Dr Bhatia Medical Coaching Institute:Online Test Platform


transformation
test
CD4+/CD8+ T-cell 1.2
ratio in lesions

BB (NT); BL: 0.48

0.50

M. leprae PGL-1
antibodies

85%

95%

60%

(Q.291) Electroconvulsive therapy is most effective in reducing


(a)

Affective symptoms

(b)

Memory impairment

(c)

Delusions

(d)

Obsessions

Your Response :

Correct Answer :

Exp:

Affective symptoms
Electroconvulsive therapy is most effective in reducing affective symptoms, i.e. major severe depression
especially when it is associated with suicidal risk, this is the first and most important indication for ECT. It is also
indicated in severe depression associated with stupor, poor food intake, melancholia, psychotic features or
unsatisfactory response to drug therapy.

(Q.292) A patient of schizophrenia taking anti Psychotic drug haloperidol was brought to Psychiatry OPD with complaints of turning of neck

to one side, excessive salivation, decreased body movements and tremors of hands. His diagnosis would be:
(a)

Allergic reaction

(b)

Drug induced extra pyramidal syndrome

(c)

Akathisia

(d)

Tardive dyskinesia

Your Response :

Correct Answer :

Exp:

Drug induced extra pyramidal syndrome


Extra Pyramidal syndrome is extremely common with the use of haloperidol. It is not dose related. The clinical
picture resembles parkinsonian like syndrome. Anti parkinsonian drug ego Trihexyphenidyl is used to treat this
disorder.

(Q.293) A31 yr. old man complaints of a 6 months history of malaise, fatigue, depressive rumination, sleep disturbances and weight loss. He

also complains of difficulty in concentrating on tasks. He admits to occasional intravenous opioid abuse. Psychological testing
reveals mild cognitive deficits. Which of the following is the most appropriate next step for management of this patient:
(a)

Initiate Imipramine

(b)

Initiate drug dependence treatment

(c)

Obtain a blood specimen for HIV screening

(d)

Obtain Urine specimen for screening of opioids.

Your Response :

Correct Answer :

Exp:

Obtain. a blood specimen for HIV screening.


Intravenous drug abuse poses significant risk for HIV infection, which can present with mental symptoms
especially mood disturbance and cognitive deficits. Drug rehabilitation may be indicated later, but the initial
steps for management of this case should be for diagnostic purposes.

(Q.294) All of the followings are feature of delirium except :


(a)

This is the commonest organic disorder in clinical practice

(b)

Motor symptoms like flapping tremor and multifocal myoclonus may present

(c)

Cognition remains unaffected

(d)

Disturbance of sleep wake cycle may be present

Your Response :

Correct Answer :

Exp:

Cognition remains unaffected


Delirium is the commonest organic disorder in clinical practice. It has acute onset. Clouding of consciousness,
disorientation are characteristic features. Motor symptoms including asterixis (flapping tremors) and myoclonus
may be there. Definite diagnosis is made by impairment of consciousness and attention, global disturbance of
cognition, psychomotor disturbance, disturbance of sleep wake cycle, and emotional disturbance.

(Q.295) The psychogenic stupor can be differentiated from organic stupor by all except :
(a)

Protective reflexes are present

(b)

Resistance to eye opening - present

(c)

Doll's head eye phenomenon - present

(d)

Oculovestibular reflex - present

Your Response :

http://dbmci.examonair.com/Result/ShowAllQuestionInHtml.aspx?testid=5615

109/111

12/25/2014

Dr Bhatia Medical Coaching Institute:Online Test Platform


Correct Answer :

Exp:

Doll's head eye phenomenon - present


The psychogenic stupor usually has previous psychiatric illness and absence of medical illness. The onset is
sudden, has short course. The protective reflexes (menace reflex, blepharospasm) are present and there is
resistance to eye opening. The Doll's head eye phenomenon or oculocephalic reflex is absent which is usually
present in organic stupor. The oculovestibular reflex is present. The neurological signs and deficits are absent but
less often an abnormal EEG may be found.

(Q.296) Which type of schizophrenia responds best to electroconvulsive therapy :


(a)

Simple schizophrenia

(b)

Paranoid schizophrenia

(c)

Catatonic schizophrenia

(d)

Hebephrenic schizophrenia

Your Response :

Correct Answer :

Exp:

Catatonic schizophrenia
Among various subtypes of schizophrenia catatonic schizophrenia responds best to electroconvulsive therapy.
With E.(C)T. Pt. shows improvement even before drugs show their effect.

(Q.297) Which of the following antipsychotic drug is completely free of extra pyramidal side effects:
(a)

Thioridazine

(b)

Loxapine

(c)

Clozapine
All of the above

(d)
Your Response :

Correct Answer :

Exp:

Clozapine
Most of the anti psychotics due to blocking dopamine receptors in nigrostriatal tract produce parkinsonian like
side effect known as EPS (extra pyramidal syndrome), whereas clozapine is the drug which does not have any
action on dopamine receptors so it does not produce EPS. It acts via blockade of serotonin receptors.

(Q.298) A Patient who recently suffered myocardial infarction developed symptoms of depression. Which of the following antidepressant

will be best suited for him :


(a)

Imipramine

(b)

Amitriptyline

(c)

Sertraline
Dothiepin

(d)
Your Response :

Correct Answer :

Exp:

Sertraline
Imipramine, Amitriptyline & Doxepin are tricyclic antidepressant drugs. These drugs have cardiac side effects in
the form of tachycardia, conduction time defects and can precipitate arrythmias in post M.I. patients leading to
sudden death. Sertraline is a selective serotonergic reuptake inhibitor and do not have any anti cholinorgic side
effects and so are considered safe for cardiac patients.

(Q.299) Dysthymia is :
(a)

Severe depression

(b)

Chronic mild depression

(c)

Bipolar depression

(d)

Depressive personality disorder

Your Response :

Correct Answer :

Exp:

Chronic mild depression


Dysthymia is a term used for persistent depressive symptoms which last for more than 2 years but are not severe
enough to be labelled as depressive episode so it is called chronic mild depression.

(Q.300) Sampatrilat is used for which condition?


(a)

Hypertension

(b)

CHF

(c)

CAD

(d)

Atrial fibrillation

Your Response :

Correct Answer :

Exp:

CHF

http://dbmci.examonair.com/Result/ShowAllQuestionInHtml.aspx?testid=5615

110/111

12/25/2014

Dr Bhatia Medical Coaching Institute:Online Test Platform


Sampatrilat is a novel vasopeptidase inhibitor that causes a greater benefit than traditional angiotensinconverting enzyme (ACE) inhibitors in the treatment of chronic heart failure (CHF).
It suppresses tissue ACE and neutral endopeptidase (NEP) activities.

Technology Partner:Yoctel Solutions (P) Ltd.

http://dbmci.examonair.com/Result/ShowAllQuestionInHtml.aspx?testid=5615

111/111

You might also like